Pharm Flashcards

1
Q
Tight capillary cell junctions resulting in an
added barrier to the entry of drugs is most
characteristic of which organ or tissue?
A. Adrenal gland
B. Brain
C. Heart
D. Liver
E. Lung
A

B. The brain has especially tight capillary junctions as
well as glial cells that result in a blood–brain
barrier.

How well did you know this?
1
Not at all
2
3
4
5
Perfectly
2
Q

A prescription for which of the following drugs

requires a valid DEA number on the prescrip-
tion?

A. Amoxicillin
B. Carbamazepine
C. Dexamethasone
D. Diphenhydramine
E. Oxycodone
A

E. Only oxycodone is a scheduled drug, requiring

DEA registration on the part of the prescriber.

How well did you know this?
1
Not at all
2
3
4
5
Perfectly
3
Q
  1. What would be the effect of prior administra-
    tion of a competitive drug antagonist on the

concentration–response profile of a drug
agonist on a graded concentration–response
curve? (Assume that both drugs act at the
same receptor.)
A. The agonist curve would shift to the left.
B. The agonist curve would shift to the right.
C. The agonist curve would not change.
D. The agonist curve would not shift but would
reach a lower maximal effect than the curve
with agonist alone.
E. The agonist curve would both shift to the left
and have a lower maximal effect.

A

A. The characteristic response to a competitive
antagonist is a parallel shift to the right of the
agonist curve, with the two curves reaching the
same maximal effect.

How well did you know this?
1
Not at all
2
3
4
5
Perfectly
4
Q

How many human drug testing phases are car-
ried out before a drug is marketed?

A. One
B. Two
C. Three
D. Four

A

C. The fourth phase constitutes postmarketing sur-

veillance.

How well did you know this?
1
Not at all
2
3
4
5
Perfectly
5
Q

In what situation is the postganglionic nerve of
the sympathetic system a cholinergic nerve?
A. The nerves to the eye
B. The nerves to the heart
C. Most nerves to blood vessels
D. Most nerves to sweat glands
E. Most nerves to salivary glands

A

D. This situation for sweat glands is atypical for the

sympathetic nervous system.

How well did you know this?
1
Not at all
2
3
4
5
Perfectly
6
Q

Which is a nicotinic receptor?
A. Receptor for the neurotransmitter at the
skeletal–neuromuscular junction
B. Receptor for the neurotransmitter at the junction
between the postganglionic sympathetic nerve
and sweat glands
C. Receptor for the neurotransmitter at the junction
between the postganglionic parasympathetic
nerve and the parotid gland
D. Receptor for the neurotransmitter at the junction
between the postganglionic sympathetic nerve
and blood vessels
E. Receptor for the neurotransmitter at the junction
between the postganglionic parasympathetic
nerve and the heart

A

A. Nicotinic receptors are located at the

skeletal–neuromuscular junction, ganglia, junc-
tion of the sympathetic nerve to the adrenal

gland and the adrenal chromaffin cells, as well as
in the central nervous system.

How well did you know this?
1
Not at all
2
3
4
5
Perfectly
7
Q
Which of the following effects is a typical effect
of an antimuscarinic drug?
A. Bronchoconstriction
B. Lacrimation
C. Miosis
D. Sweating
E. Urinary retention
A

E. All other choices are typical of muscarinic cholin-

ergic receptor agonists.

How well did you know this?
1
Not at all
2
3
4
5
Perfectly
8
Q

The administration of which compound will

give “epinephrine reversal” (drop in blood pres-
sure from epinephrine) if given prior to admin-
istration of epinephrine?

A. Atropine
B. Guanethidine
C. Propranolol
D. Phenoxybenzamine
E. Tyramine
A

D. α-Adrenoceptor blockers such as phenoxy-
benzamine will inhibit the vasoconstrictor effect of

epinephrine but not the vasodilator effect of epi-
nephrine. Therefore, the administration of

α-blockers will result in epinephrine reversal.
Atropine would have little effect since it does not
act at adrenergic receptors. Propranolol would only
block the vasodilator effect of epinephrine and the
effect of epinephrine on the heart. Guanethidine
and tyramine act largely at prejunctional sites and
don’t block adrenergic receptors.

How well did you know this?
1
Not at all
2
3
4
5
Perfectly
9
Q
Motor adverse effects from phenothiazine
antipsychotic drugs are due to drug effects in
what region of the brain?
A. Chemoreceptor trigger zone
B. Cerebrum
C. Cerebellum
D. Nigro-striatal pathway
E. Mesolimbic pathway
A

D. The nigro-striatal pathway contains dopa-
minergic neurons—important in muscle control.

Many antipsychotic drugs block these, leading to
the motor adverse effects.

How well did you know this?
1
Not at all
2
3
4
5
Perfectly
10
Q

A patient is administered haloperidol. Along
with the haloperidol, the patient also receives
benztropine. What is the most likely reason for
administering the benztropine?
A. To reduce the effects of histamine release
B. To aid in the therapeutic response to
haloperidol
C. To reduce the motor adverse effects of
haloperidol
D. To overcome a decrease in salivary flow
resulting from haloperidol
E. To reduce the rate of kidney excretion of
haloperidol

A

C. The antimuscarinic action of benztropine tends to
reduce the Parkinsonlike symptoms and some
other motor symptoms caused by haloperidol, a
dopamine receptor blocker. It does not improve
the antipsychotic effect of haloperidol. Histamine
release appears to play little role in this interaction.
Benztropine actually reduces salivary flow and

xerostomia can easily result from its admin-
istration. Benztropine has little effect on renal

clearance of haloperidol.

How well did you know this?
1
Not at all
2
3
4
5
Perfectly
11
Q
The benzodiazepine receptors BZ1 and BZ2 are
located on which ion channel?
A. Calcium
B. Chloride
C. Magnesium
D. Potassium
E. Sodium
A

B. The two benzodiazepine receptor subtypes (tar-
gets for drugs such as diazepam) are located on

the same chloride channel as is the GABAA
receptor.

How well did you know this?
1
Not at all
2
3
4
5
Perfectly
12
Q

Methemoglobinemia is an adverse effect associ-
ated with which local anesthetic due to its

metabolism to o-toluidine?
A. Lidocaine
B. Mepivacaine
C. Prilocaine
D. Bupivacaine
E. Benzocaine
A

C. Only prilocaine is metabolized to o-toluidine.

How well did you know this?
1
Not at all
2
3
4
5
Perfectly
13
Q
Which drug poses the greatest risk of a cardiac
arrhythmia when administered at the same
time as epinephrine?
A. Desflurane
B. Halothane
C. Isoflurane
D. Propofol
E. Sevoflurane
A

B. Halothane sensitizes the heart to epinephrine

and other catecholamines.

How well did you know this?
1
Not at all
2
3
4
5
Perfectly
14
Q

Local anesthetics act on what type of receptor?
A. An ion channel receptor
B. A nuclear receptor
C. A 7-membrane domain receptor linked to Gs
D. A 7-membrane domain receptor linked to Gq
E. A membrane receptor with tyrosine kinase
activity

A

A. Inhibiting sodium channels leads to the inhibition
of the nerve action potential and inhibition
of nerve conduction. Sodium channels are
examples of ion channel receptors. Ion channel
receptors contain several subunits arranged in a
barrel shape. Drugs that bind to the channel can
alter conductance to the ion associated with that
channel.

How well did you know this?
1
Not at all
2
3
4
5
Perfectly
15
Q

Which drug lacks the amine group that other
anesthetics have and is used only topically?
A. Procaine
B. Mepivacaine
C. Lidocaine
D. Benzocaine
E. Prilocaine

A

D. Benzocaine lacks the amine group that procaine,
mepivacaine, prilocaine, and lidocaine have.
This amine group can become protonated, thus
making these drugs more water-soluble and
facilitating an injectible form. Benzocaine must be provided in a cream or oil-based preparation
allowing just a topical form. Procaine and
mepivacaine have poor topical anesthetic
properties.

How well did you know this?
1
Not at all
2
3
4
5
Perfectly
16
Q

Injecting a local anesthetic into an area of
inflammation would have which effect?
A. Increase the rate of onset of anesthesia.
B. Decrease the rate of metabolism of the
anesthetic.
C. Reduce the net anesthetic effect of the drug.
D. Reduce the vasodilator effect of the local
anesthetic.
E. Reduce the need for a vasoconstrictor with the
local anesthetic.

A

C. An area of inflammation is an area of low pH. The
acid environment would convert more of the
drug into the charged form, making it less able to
diffuse to the nerve cells. This would reduce the
rate of onset and the net anesthetic effect of the
drug.

How well did you know this?
1
Not at all
2
3
4
5
Perfectly
17
Q

Which two drugs have mechanisms of anal-
gesic action that are most similar?

A. Fentanyl, ibuprofen
B. Aspirin, codeine
C. Oxycodone, acetaminophen
D. Ibuprofen, naproxen
E. Aspirin, ibuprofen
A

D. All of the choices are combinations of an opioid
and an inhibitor of cyclo-oxygenase (COX), except
two: ibuprofen, naproxen and aspirin, ibuprofen.
Ibuprofen and naproxen are both reversible

inhibitors of COX, and are propionic acid deriva-
tives. Aspirin is a salicylate and is an irreversible

inhibitor.

How well did you know this?
1
Not at all
2
3
4
5
Perfectly
18
Q

Your patient is continually taking a small daily
dose of aspirin (82 mg) prescribed by the
patient’s physician. The object of this therapy
is most likely what mechanism?
A. To mimic the effect of endogenous endorphins
B. To inhibit the production of prostaglandin E1
C. To inhibit the production of thromboxane A2
D. To inhibit the production of arachidonic acid
E. To inhibit the production of leukotrienes

A

C. Thromboxane A2 increases platelet aggregation.
Its inhibition is the target of low-dose aspirin which
inhibits cyclo-oxygenase. Inhibition of this enzyme
leads to a reduction in important down stream
products, including thromboxane A2

How well did you know this?
1
Not at all
2
3
4
5
Perfectly
19
Q

Your patient indicates that he is taking medica-
tion for atrial fibrillation. He reports that a

blood test has indicated that he has an INR
number of 4.0. An emergency dental extraction

is now required. Which postoperative medica-
tion would pose the greatest risk for an

adverse effect in this patient?
A. Acetaminophen
B. Amoxicillin
C. Aspirin
D. Codeine
E. Ibuprofen
A

C. The international normalized ratio (INR) value

indicates that the patient has received anticoagu-
lant therapy for his atrial fibrillation. Aspirin

increases the risk of postsurgical bleeding. The
combination of increase in prothrombin time,
surgery, and the antiplatelet effect of aspirin
make aspirin contraindicated in this situation.
Ibuprofen’s effect on the platelet is reversible,
whereas the effect of aspirin on the platelet is
irreversible. Thus, aspirin poses a greater risk
than does ibuprofen in this situation.

How well did you know this?
1
Not at all
2
3
4
5
Perfectly
20
Q
Which drug blocks H1 histamine receptors but
is least likely to cause sedation?
A. Diphenhydramine
B. Hydroxyzine
C. Fexofenadine
D. Albuterol
E. Famotidine
A

C. The first three choices are all H1 histamine recep-
tor blockers. Fexofenadine, however, is largely

excluded from the central nervous system, unlike
diphenhydramine and hydroxyzine. Albuterol is a
β2 adrenergic receptor agonist. Famotidine is a H2
histamine receptor antagonist.

How well did you know this?
1
Not at all
2
3
4
5
Perfectly
21
Q
The use of selective COX-2 inhibitors has
recently been restricted or discontinued
because of what type of adverse effects?
A. Carcinogenesis
B. Cardiovascular disorders
C. Convulsive disorders
D. Striated muscle disorders
E. Skeletal disorders
A

B. The cardiovascular risks may be associated with

adverse hematologic effects, but the exact mech-
anism is not yet known.

How well did you know this?
1
Not at all
2
3
4
5
Perfectly
22
Q

Sodium reabsorption in the thick ascending limb
of the loop of Henle is inhibited by which drug?
A. Bumetanide
B. Chlorthalidone
C. Hydrochlorothiazide
D. Spironolactone
E. Triamterene

A

A. All the drugs listed are diuretics. However, only
bumetanide acts on the ascending limb of the

loop of Henle. It is called a “loop” or “high ceil-
ing” diuretic because of its site of action in the

nephron and maximal effect, respectively.

How well did you know this?
1
Not at all
2
3
4
5
Perfectly
23
Q

Torsades de pointes, or polymorphic ventricu-
lar tachycardia, is linked most closely to what

characteristic of the electrocardiogram?
A. Inverted T wave
B. Shorter P-R interval
C. Shorter P-P interval
D. Longer Q-T interval
E. Normal electrocardiogram
A

D. The long Q-T interval observed as a result of cer-
tain drugs or as a hereditary condition makes the

patient more susceptible to this condition.

How well did you know this?
1
Not at all
2
3
4
5
Perfectly
24
Q
Which antihypertensive drug also increases
bradykinin levels?
A. Candesartan
B. Furosemide
C. Lisinopril
D. Metoprolol
E. Nifedipine
A

C. Lisinopril, by virtue of the fact that it inhibits
angiotensin-converting enzyme (ACE) (also

called peptidyl dipeptidase), inhibits the break-
down of bradykinin.

How well did you know this?
1
Not at all
2
3
4
5
Perfectly
25
Q

Which one of the following drugs enters the
target cell and acts on a nuclear receptor?
A. Diazepam
B. Epinephrine
C. Insulin
D. Prednisone
E. Heparin

A

D. Diazepam, epinephrine, and insulin act at ion
channel receptors, G-protein-linked receptors,

and tyrosine kinase-linked receptors, res-
pectively. These three receptor types are cell

surface receptors. Thyroid hormone and steroid
hormones or drugs, such as prednisone, act on
Sample Exam Answer Key ▼ 411

nuclear receptors, accounting for much of their
action. Heparin’s action is to stimulate
antithrombin III in the plasma. Its action is
extracellular.

How well did you know this?
1
Not at all
2
3
4
5
Perfectly
26
Q
Inhibiting a-glucosidase and reducing glucose
absorption from the gastrointestinal tract is
the mechanism of action of which drug?
A. Acarbose
B. Acetoheximide
C. Glyburide
D. Metformin
E. Pioglitazone
A

A. All of the choices are oral hypoglycemic agents.

Only acarbose inhibits α-glucosidase.

How well did you know this?
1
Not at all
2
3
4
5
Perfectly
27
Q

Which of the following drugs blocks the aldos-
terone receptor?

A. Amiloride
B. Triamterene
C. Losartan
D. Spironolactone
E. Furosemide
A

D. Spironolactone, a potassium-sparing diuretic
useful in treating edema and heart failure, is
a competitive antagonist at the aldosterone
receptor.

How well did you know this?
1
Not at all
2
3
4
5
Perfectly
28
Q

Which drug is most selective as a glucocorticos-
teroid?

A. Aldosterone
B. Dexamethasone
C. Fludrocortisone
D. Hydrocortisone

A

B. Aldosterone and fludrocortisone are selective

mineralocorticosteroids. Hydrocortisone has sig-
nificant mineralocorticoid and glucocorticoid

activity. Dexamethasone has very little mineralo-
corticoid activity.

How well did you know this?
1
Not at all
2
3
4
5
Perfectly
29
Q

Stimulation of gluconeogenesis and lipolysis
are most characteristic of which hormone?
A. Calcitonin
B. Cortisol
C. Insulin
D. Parathyroid hormone
E. Progesterone

A

B. Glucocorticoids characteristically stimulate glu-
coneogenesis and lipolysis. Insulin has the oppo-
site effects. The other hormones have minor or

negligible effects.

How well did you know this?
1
Not at all
2
3
4
5
Perfectly
30
Q
Fanconi syndrome from outdated tetracyclines
affects predominantly which organ?
A. Brain
B. Heart
C. Kidney
D. Pancreas
E. Stomach
A

C. Renal tubular acidosis, aminoaciduria, and

hyperphosphaturia are some of the manife-
stations of proximal tubule damage in Fanconi

syndrome.

How well did you know this?
1
Not at all
2
3
4
5
Perfectly
31
Q
Methicillin-resistant Staphylococci are most
likely to be inhibited by which drug?
A. Amoxicillin
B. Clarithromycin
C. Clindamycin
D. Vancomycin
E. Penicillin V
A

D. Of the choices given, only vancomycin is effec-
tive against many methicillin-resistant Staphy-
lococci. Various penicillins, macrolides, and

clindamycin are ineffective.

How well did you know this?
1
Not at all
2
3
4
5
Perfectly
32
Q

Pick the organism that is usually clinically sen-
sitive to clarithromycin but not to penicillin V.

A. Streptococcus viridans
B. Leptotrichia buccalis
C. Mycoplasma pneumoniae
D. Streptococcus pneumoniae
E. Streptococcus pyogenes
A

C. Because it lacks a cell wall, Mycoplasma pneumo-
niae is not sensitive to cell wall inhibitors such as

penicillin V. The macrolides, such as clar-
ithromycin, are ribosomal protein synthesis

inhibitors that are effective against Mycoplasma

pneumoniae. Streptococcus viridans, Strep-
tococcus pneumoniae, and Streptococcus pyo-
genes are gram-positive cocci. Leptotrichia

buccalis is a gram-negative oral bacillus.

How well did you know this?
1
Not at all
2
3
4
5
Perfectly
33
Q
What is the approximate elimination half-time
for penicillin V?
A. 0.5 hour
B. 2 hours
C. 4 hours
D. 8 hours
E. 12 hours
A

A. The short elimination half-time for penicillin V is
due to rapid excretion of penicillin in the urine.
About 90% of this renal excretion is a result of
active tubular transport, a rapid and efficient
process. (Very little metabolism of penicillin
occurs.)

How well did you know this?
1
Not at all
2
3
4
5
Perfectly
34
Q
Which drug has an antibacterial spectrum that
is limited to anaerobes?
A. Amoxicillin
B. Clarithromycin
C. Clindamycin
D. Gentamicin
E. Metronidazole
A

E. Amoxicillin, clarithromycin, and clindamycin are

effective against some anaerobes but their spec-
trum is not limited to anaerobic bacteria.

Aminoglycosides are effective only against aer-
obes. Metronidazole’s action requires a reduced

environment. Its antibacterial spectrum is limited
to anaerobes. Metronidazole is also effective
against many parasites.

How well did you know this?
1
Not at all
2
3
4
5
Perfectly
35
Q
Which drugs tend to concentrate in body
compartments of high pH?
A. Permanently charged drugs
B. Drugs that are not charged
C. Weak organic acids
D. Weak organic bases
E. Inorganic ions
A

C. Only weak acids and weak bases are greatly
affected in their distribution by changes in pH.
Weak organic acids dissociate more from

protons at higher pH, making a higher percent-
age of their molecules charged. This traps them

in that compartment.

How well did you know this?
1
Not at all
2
3
4
5
Perfectly
36
Q
Drug agonists having the same intrinsic activity
also have the same \_\_\_\_\_.
A. Maximal effect
B. Potency
C. Receptor affinity
D. Therapeutic index
E. Aqueous solubility
A

A. Drug agonists have an intrinsic activity of greater
than 0 and less than or equal to 1. This refers to
the maximal effect attainable by the drug.
Potency and receptor affinity are not directly
related to intrinsic activity. The therapeutic index
(TI) requires a quantal dose–response curve,
unlike the other characteristics listed which
require graded concentration–response curves.
Drugs with the same intrinsic activity may vary a
great deal in their aqueous solubility.

How well did you know this?
1
Not at all
2
3
4
5
Perfectly
37
Q

What receptor or signaling pathway is linked
most directly to a2-adrenoceptor stimulation?
A. Gi and a reduction in cAMP
B. Gs and an increase in cAMP
C. Gq and calcium
D. Sodium ion channel
E. Membrane receptor containing tyrosine kinase

A

A. Inhibition of adenylyl cyclase through Gi

, result-
ing from stimulation of α2-adrenergic receptor,

leads to a reduction in intracellular cAMP.

How well did you know this?
1
Not at all
2
3
4
5
Perfectly
38
Q
What tissue or organ has many muscarinic
receptors but lacks innervation to those
receptors?
A. Heart
B. Parotid gland
C. Blood vessels
D. Sweat glands
E. Urinary bladder
A

C. Circulating muscarinic cholinergic receptor
agonists stimulate these receptors on endothelial
cells, leading to release of nitric oxide and
vasodilation.

How well did you know this?
1
Not at all
2
3
4
5
Perfectly
39
Q
Which drug used in the therapy for
Parkinsonism does not cross the blood–brain
barrier?
A. Amantadine
B. Carbidopa
C. L-dopa
D. Selegiline
E. Tolcapone
A

B. Carbidopa is used to inhibit dopa decarboxylase.
Its usefulness is based on reducing conversion of
L-dopa to dopamine outside the central nervous
system. Carbidopa does not penetrate the

blood–brain barrier and therefore does not inter-
fere with the beneficial effect of L-dopa in the

brain, but prevents the adverse effects of dopa-
mine in the periphery.

How well did you know this?
1
Not at all
2
3
4
5
Perfectly
40
Q

After an injection, which drug would be expected
to have the longest duration of action? (Assume
no vasoconstrictor was injected with the local
anesthetic.)
A. Bupivacaine
B. Lidocaine
C. Mepivacaine
D. Prilocaine
E. Procaine

A

A. Bupivacaine has the highest lipid solubility of the

drugs listed. This is the major chemical charac-
teristic of the local anesthetic that determines

duration of action. Procaine is the only ester
given as a choice and is rarely used.

How well did you know this?
1
Not at all
2
3
4
5
Perfectly
41
Q

A very low blood:gas solubility coefficient
(partition coefficient = 0.47), analgesic effect,
and a drug that inhibits methionine synthase
best describes which drug?
A. Ketamine
B. Nitrous oxide
C. Halothane
D. Isoflurane
E. Propofol

A

B. Nitrous oxide oxidizes the cobalt in vitamin B12,
resulting in the inhibition of methionine synthase.
Nitrous oxide has greater analgesic potency than

other inhaled anesthetics (e.g., halothane, isoflu-
rane). Ketamine is not inhaled; rather, it is

injected. It also does not inhibit methionine syn-
thase. The same is true for propofol.

How well did you know this?
1
Not at all
2
3
4
5
Perfectly
42
Q

Levonordefrin is added to certain cartridges
containing mepivacaine. The desired effect of
levonordefrin is due to what pharmacological
effect?
A. Inhibition of nicotinic cholinergic receptors
B. Inhibition of muscarinic cholinergic receptors
C. Stimulation of α-adrenergic receptors
D. Stimulation of β-adrenergic receptors
E. Stimulation of dopamine receptors

A

C. Alpha adrenergic receptor stimulation accounts

for the vasoconstrictor effect of levonordefrin.

How well did you know this?
1
Not at all
2
3
4
5
Perfectly
43
Q

The analgesic effects of dextromethorphan are
due to what receptor effect?
A. Gamma aminobutyric acid (GABA) receptor
antagonism.
B. Dopamine receptor antagonism.
C. Nicotinic cholinergic receptor antagonism.
D. Mu (μ) opioid receptor antagonism.
E. N-methyl-D-aspartate (NMDA) receptor
antagonism.

A

E. The recently described mild analgesic effect of

dextromethorphan has been linked to N-methyl-
D-aspartate (NMDA) receptor antagonism in the

CNS.

How well did you know this?
1
Not at all
2
3
4
5
Perfectly
44
Q
Naloxone antagonizes the therapeutic and toxic
effects of which drug?
A. Acetaminophen
B. Aspirin
C. Carbamazepine
D. Fentanyl
E. Ibuprofen
A

D. Naloxone is a competitive antagonist at opioid

receptors.

How well did you know this?
1
Not at all
2
3
4
5
Perfectly
45
Q
What is the mechanism of the analgesic action
of aspirin?
A. Stimulates μ opioid receptors
B. Blocks histamine H2 receptors
C. Inhibits cyclooxygenase
D. Inhibits lipoxygenase
E. Blocks sodium channels in nerves
A

C. Cyclooxygenase (COX) is a key enzyme in the
synthesis of prostaglandins. Prostaglandins,

including PGE2 and PGF2α, are important media-
tors for such functions as pain, and are a product

of COX. Aspirin inhibits both COX-1 and COX-2.

How well did you know this?
1
Not at all
2
3
4
5
Perfectly
46
Q
What is the clinical setting for the use of
ketorolac by the oral route?
A. For severe pain
B. For initial treatment of pain
C. To continue therapy after an IV or IM dose of
ketorolac
D. Only in combination with an opioid
E. Only in combination with an NSAID
A

C. The use of oral ketorolac (an NSAID) is limited to

continue therapy after a parenteral dose.

How well did you know this?
1
Not at all
2
3
4
5
Perfectly
47
Q

The use of H2 histamine receptor blockers is
most clinically useful at what cell type?
A. Beta cells of the pancreas
B. Basophils
C. Mast cells
D. Juxtaglomerular cells
E. Parietal cells

A

E. Basophils and mast cells release histamine.
However, the cell that responds to histamine
stimulation at the H2 receptor is the parietal cell
of the stomach. Stimulation of this receptor leads
to proton release and a decrease in the pH of the
stomach lumen. H2 histamine receptor blockers
are used to reduce stomach acid.

How well did you know this?
1
Not at all
2
3
4
5
Perfectly
48
Q
14. Which class of antihypertensive drug most
effectively reduces the release of renin from the
kidney?
A. β-adrenergic receptor blockers
B. ACE inhibitors
C. α-adrenergic receptor blockers
D. Calcium channel blockers
E. Angiotensin II receptor blockers
A

A. Renin release from the kidney is enhanced by
stimulation of the β1-adrenergic receptors in
the juxtaglomerular cells. From the above list,
only β blockers reduce renin release. Although
angiotensin converting enzyme inhibitors and
angiotensin II receptor blockers act on the
renin-angiotensin system, they do not inhibit
renin release. In fact, they tend to increase
plasma renin.

How well did you know this?
1
Not at all
2
3
4
5
Perfectly
49
Q
The administration of which compound will give
“epinephrine reversal” (drop in blood pressure
from epinephrine) if given prior to
administration of epinephrine?
A. Guanethidine
B. Propranolol
C. Phenoxybenzamine
D. Tyramine
A

C. Alpha adrenoceptor blockers, like phenoxyben-
zamine, inhibit the vasoconstrictor effect of epi-
nephrine but not the vasodilator effect of

epinephrine. Therefore, the administration of alpha blockers results in epinephrine reversal.
Propranolol would only block the vasodilator

effect of epinephrine. Guanethidine and tyra-
mine act largely at prejunctional sites and don’t

block adrenergic receptors.

How well did you know this?
1
Not at all
2
3
4
5
Perfectly
50
Q

What is the mechanism of action of enoxaparin?
A. Inhibition of synthesis of clotting factors II, VII, IX,
and X
B. Activation of antithrombin III with resulting
inhibition of clotting factor Xa
C. Indirect activation of tissue plasminogen activator
D. Direct inhibition of plasminogen with resulting
degradation of fibrin
E. Dilation of coronary blood vessels

A

B. Enoxaparin is a low-molecular-weight heparin. It

activates antithrombin III and inhibits factor Xa.

How well did you know this?
1
Not at all
2
3
4
5
Perfectly
51
Q
Oropharyngeal candidiasis is an adverse effect
most likely with which drug?
A. Inhaled salmeterol
B. Inhaled ipratropium
C. Inhaled nedocromil
D. Inhaled beclomethasone
E. Inhaled methacholine
A

D. The effect of glucocorticosteroids remaining in
the mouth after inhalation is to make the oral
cavity more susceptible to fungal infection. The

mouth should be rinsed with water after inhala-
tion use. Inhaled methacholine, unlike the other

drugs listed, is not used therapeutically but,
rather, is used to diagnose hyperactive airway.

How well did you know this?
1
Not at all
2
3
4
5
Perfectly
52
Q
Oral antacids are most likely to reduce the
absorption of which drug when it is given
orally?
A. Clarithromycin
B. Clindamycin
C. Metronidazole
D. Penicillin V
E. Tetracycline
A

E. Di- and trivalent cations, such as those found in
oral antacids, chelate tetracyclines and prevent
their absorption.

How well did you know this?
1
Not at all
2
3
4
5
Perfectly
53
Q
A decrease in glycogenolysis in the liver would
be expected from which drug?
A. Albuterol
B. Epinephrine
C. Glucagon
D. Insulin
E. Parathyroid hormone
A

D. A decrease in glycogen breakdown is a classic
effect of insulin. Epinephrine (by acting as an
agonist at α1- and β2-adrenergic receptors),
albuterol (by acting as an agonist at β2
-adrenergic
receptors), and glucagon (by acting at glucagon
receptors) all tend to increase glycogen.
Parathyroid hormone has little effect on
glycogenolysis.

How well did you know this?
1
Not at all
2
3
4
5
Perfectly
54
Q

Nitrates and nitrites have what effect on blood
vessel smooth muscle?
A. Increase in the level of intracellular calcium
B. Increase in the level of cyclic guanosine
monophosphate (cGMP)
C. Antagonism at α1-adrenergic receptors
D. Antagonism at β-adrenergic receptors
E. Inhibition of L-type calcium channels

A

B. Nitroglycerin is a nitrovasodilator. It produces
nitric oxide, which activates guanylyl cyclase
which, in turn, catalyzes the production of cGMP.

How well did you know this?
1
Not at all
2
3
4
5
Perfectly
55
Q
Clavulanic acid offers an advantage
therapeutically because it has what action?
A. It inhibits Streptococci at a low minimum
inhibitory concentration (MIC).
B. It inhibits transpeptidase.
C. It inhibits penicillinase.
D. It inhibits anaerobes at a low MIC.
E. It inhibits DNA gyrase.
A

C. Clavulanic acid has very little antimicrobial activity.
Its value in combination with certain penicillins is

due to its ability to inhibit certain penicillinases.
This protects the penicillin from bacterial enzyme
attack. Transpeptidase is inhibited by β-lactams,
such as penicillin. DNA gyrase is inhibited by the
fluoroquinolones such as ciprofloxacin.

How well did you know this?
1
Not at all
2
3
4
5
Perfectly
56
Q
Identify the enzyme whose inhibition is most
responsible for the cell wall synthesis inhibitory
effect of penicillin G.
A. β-lactamase
B. DNA gyrase
C. Nitro reductase
D. Transglycosylase
E. Transpeptidase
A

E. Transpeptidase is the enzyme that catalyzes the

peptide crosslinking of peptidoglycan. Trans-
peptidase, is inhibited by penicillins and ceph-
alosporins.

How well did you know this?
1
Not at all
2
3
4
5
Perfectly
57
Q
Which drug is often combined with
sulfamethoxazole for the treatment of
respiratory tract and urinary tract infections?
A. Amoxicillin
B. Ciprofloxacin
C. Clindamycin
D. Metronidazole
E. Trimethoprim
A

E. Trimethoprim, by virtue of its inhibition of bacte-
rial dihydrofolate reductase, acts synergistically

with the sulfonamides.

How well did you know this?
1
Not at all
2
3
4
5
Perfectly
58
Q

Which of the following organisms is usually
sensitive to clindamycin?
A. Candida albicans
B. Klebsiella pneumoniae
C. Methicillin-resistant Staphylococcus aureus
D. Streptococcus viridans
E. Pseudomonas aeruginosa

A

D. Clindamycin is useful for some oral infections,
including those involving Streptococcus viridans.

Klebsiella pneumoniae and Pseudomonas aerug-
inosa are gram-negative rods and not subject to

clinical inhibition by clindamycin. Methicillin-
resistant Staphylococci are insensitive to clinda-
mycin and most traditional antistaphylococcal

drugs. Candida albicans is a yeastlike fungus and
is not inhibited by antibacterial drugs such as
clindamycin.

How well did you know this?
1
Not at all
2
3
4
5
Perfectly
59
Q
Dihydrofolate reductase is an enzyme inhibited
by which anticancer drug?
A. Bleomycin
B. Cisplatin
C. Doxorubicin
D. 5-fluorouracil
E. Methotrexate
A

E. The mammalian enzyme form of dihydrofolate

reductase is the target for methotrexate. Bleo-
mycin produces strand breaks in DNA. Cisplatin is

an alkylating agent. Doxorubicin intercalates with
DNA. 5-Fluorouracil, after undergoing activation,
inhibits thymidylate synthase.

How well did you know this?
1
Not at all
2
3
4
5
Perfectly
60
Q

Class of drugs most consistent in structure

A

local anesthetics

How well did you know this?
1
Not at all
2
3
4
5
Perfectly
61
Q

Ester anesthetics

A

procaine, tetracaine, cocaine

How well did you know this?
1
Not at all
2
3
4
5
Perfectly
62
Q

metabolism of esters vs amides

A

esters in plasma by esterases; amides (tertiary!) in liver

How well did you know this?
1
Not at all
2
3
4
5
Perfectly
63
Q

prilocaine side effect

A

methemoglobinemia

How well did you know this?
1
Not at all
2
3
4
5
Perfectly
64
Q

methemoglobinemia is side effect of what

A

prilocaine

How well did you know this?
1
Not at all
2
3
4
5
Perfectly
65
Q

local anesthetic overdose

A

myocardial depression, cardiovascular collapse, hypotensive shock

How well did you know this?
1
Not at all
2
3
4
5
Perfectly
66
Q

mechanism of local anesthetics

A

prevent generation of impulses; block Na transport

How well did you know this?
1
Not at all
2
3
4
5
Perfectly
67
Q

which form of anesthetic can penetrate

A

free base, non-ionized

How well did you know this?
1
Not at all
2
3
4
5
Perfectly
68
Q

absorption of local anesthetic in inflamed tissues

A

inflammation –> lower pH –> less non-ionized form available to penetrate

How well did you know this?
1
Not at all
2
3
4
5
Perfectly
69
Q

2% lidocaine - amount in one carpule?

A

one carpule is 1.8 mL
2% means 20 mg/mL
so 20 x 1.6 = 36 mg in one 1.8 mL carpule

How well did you know this?
1
Not at all
2
3
4
5
Perfectly
70
Q
Which of the following is a local anesthetic subject to inactivation by
plasma esterases?
a. Procaine
b. Lidocaine
c. Prilocaine
d. Mepivacaine
e. Bupivacaine
A

(a) Proccaine is the only ester listed – all the rest are amides

How well did you know this?
1
Not at all
2
3
4
5
Perfectly
71
Q

Procaine differs from lidocaine in that
a. Procaine is a p-aminobenzoic acid ester and lidocaine is not
b. Lidocaine is a meta-aminobenzoic acid ester and procaine is not
c. The duration of action of procaine is longer than that of an equal
total dose of lidocaine
d. Procaine hydrochloride is metabolized into diethylaminoethanol
and benzoic acid.
(a) this is basically a true-false type question. (a) is the only
statement that is true

A

(a) is the only

statement that is true

How well did you know this?
1
Not at all
2
3
4
5
Perfectly
72
Q
Which of the following local anesthetics would be expected to produce
a sensitization reaction in a patient allergic to lidocaine?
a. Mepivacaine
b. Tetracaine
c. Procaine
d. Prilocaine
e. Dibucaine
i. (a), (b) and (c)
ii. (a), (d) and (e)
iii. (b) and (c) only
iv. (b), (c) and (d)
v. (b), (d) and (e)
A

(ii) another ester vs. amide type identification question.

Lidoccaine is an amide, thus other amides will be cross-
allergenic - mepivacaine, prilocaine and dibucaine are the other

amides listed. Procaine and tetracaine are esters and will not be
cross-allergenic.

How well did you know this?
1
Not at all
2
3
4
5
Perfectly
73
Q

The hydrolysis of procaine occurs mainly in the

a. Liver
b. Lungs
c. Plasma
d. Muscles
e. Kidneys

A

(c) procaine is an ester; esters are metabolized predominately
by pseudocholinesterases in the plasma.

How well did you know this?
1
Not at all
2
3
4
5
Perfectly
74
Q
Which of the following is local anesthetic subject to inactivation by
plasma esterases?
a. Lidocaine
b. Prilocaine
c. Tetracaine
d. Mepivacaine
e. Bupivacaine
A

(c) esters are metabolized by plasma esterases - tetracaine is
the only ester listed, all the rest are amides

How well did you know this?
1
Not at all
2
3
4
5
Perfectly
75
Q

The activity of procaine is terminated by
a. Elimination by the kidney
b. Storage in adipose tissue
c. Metabolism in the liver only
d. Metabolism in the liver and by pseudocholinesterase in the
plasma

A

(d) remember #9 above? see the word “mainly”? same
question, but worded a little differently to throw you off. Again,
procaine is an ester; esters are metabolized predominately by
pseudocholinesterases in the plasma, but also to some extent
by esters in the liver.

How well did you know this?
1
Not at all
2
3
4
5
Perfectly
76
Q

All of the following factors are significant determinants of the duration
of conduction block with amide-type local anesthetics EXCEPT the
a. pH of tissues in the area of injection
b. Degree of vasodilatation caused by the local anesthetic
c. Blood plasma cholinesterase levels
d. Blood flow through the area of conduction block
e. Concentration of the injected anesthetic solution

A

(c) the word “EXCEPT” should alert you that this is basically a
true-false type question with 4 true statements and 1 false
statement; you just have to figure out which one! In this case,
you just have to remember that plasma cholinesterase levels
are only important for the duration of action of ester-type LAs,
not amides, which are metabolized in the liver. All the other
statements are variables which affect duration of the block, but
apply to both esters and amides.

How well did you know this?
1
Not at all
2
3
4
5
Perfectly
77
Q

Which of the following is contraindicated for a patient who had an
allergic reaction to procaine six months ago?
a. Nerve block with lidocaine
b. Topical application of lidocaine
c. Topical application of tetracaine
d. Infiltration with an antihistamine

A

(c) again, just another question that requires you to be able to
pick out an ester or an amide from a list. Since procaine is an
ester, only another ester LA would be cross-allergenic. In this
list the only ester listed is tetracaine.

How well did you know this?
1
Not at all
2
3
4
5
Perfectly
78
Q

Bupivacaine (Marcaine ) has all of the following properties relative to
lidocaine (Xylocaine ) EXCEPT bupivacaine
a. Is more toxic
b. Is an ester-type local anesthetic
c. Has a slower onset of action
d. Has a longer duration of action

A

(d) According to textbooks, local anesthetics fall into the
following classes in terms of duration of action: short: procaine;
moderate: prilocaine, mepivacaine, lidocaine; long: bupivacaine,
tetracaine, etidocaine. Statements (a), 3, and 4 would be true if
the question was comparing mepivacaine to bupivacaine, which
are structurally similar; but the comparison is to lidocaine. The
only difference that applies is duration of action ((d)),
bupivacaine is longer. (b) is wrong, both are amides.

How well did you know this?
1
Not at all
2
3
4
5
Perfectly
79
Q

Amide-type local anesthetics are metabolized in the

a. Serum
b. Liver
c. Spleen
d. Kidney
e. Axoplasm

A

(b) don’t forget: esters in plasma; amides in liver

How well did you know this?
1
Not at all
2
3
4
5
Perfectly
80
Q
15. The duration of action of lidocaine would be increased in the presence
of which of the following medications?
a. Prazosin
b. Propranolol
c. Hydrochlorothiazide
d. Lisinopril
e. Digoxin
A

(b) this is an interaction I tested you on several times – now you
know why! Propranolol interacts with lidocaine in two ways.
By slowing down the heart via beta receptor blockade, blood
delivery (and lidocaine) to the liver is reduced, thus lidocaine
remains in the systemic circulation longer, and can potentially
accumulate to toxic levels. Propranolol and lidocaine also
compete for the same enzyme in the liver, thus metabolism of
lidocaine can be reduced.

How well did you know this?
1
Not at all
2
3
4
5
Perfectly
81
Q

propranolol class

A

non-selective beta-blocker

slows down heart

How well did you know this?
1
Not at all
2
3
4
5
Perfectly
82
Q
Severe liver disease least affects the biotransformation of which of
the following?
a. Lidocaine
b. Procaine
c. Prilocaine
d. Mepivacaine
A

(b) Answer is (b)- You should be able to recognize that all of
these drugs are local anesthetics. Local anesthetics are of one
of two types, either esters or amides. Ester types are subject to
hydrolysis in the plasma and thus have short half lives. Amides
are metabolized primarily in the liver and have longer half lives.
Thus the biotransformation (e.g., metabolism; again, the rats are
using a different word to confuse you, even though they are
asking the same basic question) of an amide type local
anesthetic would be the most altered in the presence of sever
liver disease. The key word here is “least”. Of the drugs listed,
only procaine is an ester. The rest are amides.

How well did you know this?
1
Not at all
2
3
4
5
Perfectly
83
Q

A patient has been given a large volume of a certain local anesthetic
solution and subsequently develops cyanosis with
methemoglobinemia. Which of the following drugs most likely was
administered?
a. Procaine
b. Prilocaine
c. Dibucaine
d. Lidocaine
e. Mepivacaine

A

(b)

How well did you know this?
1
Not at all
2
3
4
5
Perfectly
84
Q
Use of prilocaine carries the risk of which of the following adverse
effects?
a. Porphyria
b. Renal toxicity
c. Gastric bleeding
d. Methemoglobinemia
A

(d) same as above but asked backwards. Methemoglobinemia
may result from a toluidine metabolite of prilocaine,
orthotoluidine.

How well did you know this?
1
Not at all
2
3
4
5
Perfectly
85
Q

which anesthetic has o-toluidine metabolite? why important?

A

prilocaine; metHbemia

How well did you know this?
1
Not at all
2
3
4
5
Perfectly
86
Q

The most probable cause for a serious toxic reaction to a local
anesthetic is
a. Psychogenic
b. Deterioration of the anesthetic agent
c. Hypersensitivity to the vasoconstrictor
d. Hypersensitivity to the local anesthetic
e. Excessive blood level of the local anesthetic

A

(e) Most toxic reactions of a serious nature are related to
excessive blood levels arising from inadvertent intravascular
injection. Hypersensitivity reactions (options b & c) are rare, but
excessive blood levels will induce toxic reactions like CNS
stimulation in most everyone. This is a case where option (e) is
the “best” answer, because it is more likely than the other
alternatives, which might be true, but are not as likely (e.g,
“most probable”) to happen.

How well did you know this?
1
Not at all
2
3
4
5
Perfectly
87
Q
  1. High plasma levels of local anesthetics may cause
    a. Inhibition of peristalsis
    b. stimulation of baroreceptors resulting in severe hypotension
    c. Inhibition of the vagus nerve to the heart
    d. Depression of inhibitory neurons in the CNS
A

(d) Initially LAs inhibit central inhibitory neurons, which results in
CNS stimulation, which can proceed to convulsions. At higher
doses, they inhibit both inhibitory and excitatory neurons,
leading to a generalized state of CNS depression which can
result in respiratory depression and death.

How well did you know this?
1
Not at all
2
3
4
5
Perfectly
88
Q

Unfortunately, you injected your lidocaine intra-arterially. The first
sign of lidocaine toxicity that might be seen in the patient would be
a. Elevated pulse rate
b. Sweating
c. CNS excitation
d. Cardiovascular collapse
e. CNS depression

A

(c) same question as above just worded differently. The intra-
arterial injection would result in the high plasma levels

mentioned in the previous question.

How well did you know this?
1
Not at all
2
3
4
5
Perfectly
89
Q
The first sign that your patient may be experiencing toxicity from too
much epinephrine would be
a. Cardiovascular collapse
b. Convulsions
c. Elevated pulse rate
d. Slurred speech
A

(c) it is a sympathomimetic after all. All the other reactions are
related to elevated lidocaine levels

How well did you know this?
1
Not at all
2
3
4
5
Perfectly
90
Q
Which disease condition would make the patient most sensitive to the
epinephrine in the local anesthetic?
a. Grave’s disease
b. Diabetes
c. HIV
d. Alcoholism
e. Schizophrenia
A

(a) Grave’s disease is an autoimmune disease that causes
hyperthyroidism – the resulting high levels of circulating thyroid
hormone result in a hypermetabolic state with heightened
sympathetic activity, which combined with injected epinephrine
could result in a hypertensive crisis.

How well did you know this?
1
Not at all
2
3
4
5
Perfectly
91
Q
Cardiovascular collapse elicited by a high circulating dose of a local
anesthetic may be caused by
a. Syncope
b. Vagal stimulation
c. Histamine release
d. Myocardial depression
e. Medullary stimulation
A

(d) Cardiovascular collapse is due to a direct action of the local
anesthetic on the heart muscle itself (LA’s in toxic doses
depress membrane excitability and conduction velocity), thus
(d) is the correct answer. All of the other alternatives are indirect
ways to affect the heart.

How well did you know this?
1
Not at all
2
3
4
5
Perfectly
92
Q

The most serious consequence of systemic local anesthetic toxicity is

a. Vertigo
b. Hypertension
c. Hyperventilation
d. Post depressive central nervous system convulsions
e. Postconvulsive central nervous system depression

A

(e) Of the options listed, this is the one that will kill the patient,
which I guess makes it the most serious.

How well did you know this?
1
Not at all
2
3
4
5
Perfectly
93
Q
Hypotensive shock may result from excessive blood levels of each of
the following local anesthetics EXCEPT
a. Cocaine
b. Procaine
c. Lidocaine
d. Tetracaine
e. Mepivacaine
A

(a) All the listed local anesthetics except cocaine are
vasodilators, especially ester-ctype drugs such as proccaine
and the amide lidocaine. Cocaine is the only local anesthetic
that predictably produces vasoconstriction. Cocaine is also the
only local anesthetic to block the reuptake of NE into adrenergic
neurons, and thus potentiate the NE that has been released
from nerve endings

How well did you know this?
1
Not at all
2
3
4
5
Perfectly
94
Q
Which of the following anesthetic drugs produces powerful
stimulation of the cerebral cortex?
a. Cocaine
b. Procaine
c. Lidocaine
d. Tetracaine
e. Mepivacaine
A

(a) see explanation above *NE potentiation)

How well did you know this?
1
Not at all
2
3
4
5
Perfectly
95
Q

Local anesthetics block nerve conduction by

a. Depolarizing the nerve membrane to neutrality
b. Increasing membrane permeability to K+
c. Increasing membrane permeability to Na+
d. Preventing an increase in membrane permeability to K+
e. Preventing an increase in membrane permeability to Na+

A

(e) didn’t I make you memorize this? You should at keast
remember Na+ ions are involved, which limits your choices to
(c) and (e). (c) would increase or facilitate nervous impulse
conduction, which is the opposite of what you want the local
anesthetic to do, so pick (e).

How well did you know this?
1
Not at all
2
3
4
5
Perfectly
96
Q

Which of the following is true regarding the mechanism of action of
local anesthetics?
a. Usually maintain the nerve membrane in a state of
hyperpolarization
b. Prevent the generation of a nerve action potential
c. Maintain the nerve membrane in a state of depolarization
d. Prevent increased permeability of the nerve membrane to
potassium ions
e. Interfere with intracellular nerve metabolism

A

(b)

How well did you know this?
1
Not at all
2
3
4
5
Perfectly
97
Q

Local anesthetic agents prevent the generation of nerve impulses by

a. Decreasing threshold for stimulation
b. Decreasing resting membrane potential
c. Decreasing inward movement of sodium ion
d. Increasing inward movement of potassium ion

A

(c) Answer is (c)- straight memorization- nerve impulses are
generated by the influx of sodium resulting in depolarization.
repolarization and inactivity occurs when potassium moves out.
(sodium-potassium pump). LAs act by blocking Na+ movement.

How well did you know this?
1
Not at all
2
3
4
5
Perfectly
98
Q
Local anesthetics interfere with the transport of which of the following
ions during drug-receptor interaction
a. Sodium
b. Calcium
c. Chloride
d. Potassium
e. Magnesium
A

(a)

How well did you know this?
1
Not at all
2
3
4
5
Perfectly
99
Q

If the pH of an area is lower than normal body pH, the membrane
theory of local anesthetic action predicts that the local anesthetic
activity would be
a. Greater, owing to an increase in the free-base form of the drug
b. Greater, owing to an increase in the cationic form of the drug
c. Less, owing to an increase in the free-base form of the drug
d. Less, owing to a decrease in the free-base form of the drug
e. None of the above

A

(d)

How well did you know this?
1
Not at all
2
3
4
5
Perfectly
100
Q

A local anesthetic injected into an inflamed area will NOT give
maximum effects because
a. The pH of inflamed tissue inhibits the release of the free base
b. The drug will not be absorbed as rapidly because of the
decreased blood supply
c. The chemical mediators of inflammation will present a chemical
antagonism to the anesthetic
d. Prostaglandins stabilize the nerve membrane and diminish the
effectiveness of the local anesthetic

A

(a) while some of the other alternatives sound plausible, think
about the factoids you were taught about local anesthetics and
variables that affect their action. An important one was the role
of pH and ionization factors. Remember, the free base or
nonionized form is the form that passes through membranes,
yet once inside the neuron only the ionized form is effective.
Inflamed tissue has a lower pH than normal tissue and will shift
the equilibrium of the LA solution such that most of it remains
ionized and thus unavailable to penetrate

How well did you know this?
1
Not at all
2
3
4
5
Perfectly
101
Q

The penetration of a local anesthetic into nervous tissue is a function
of the
a. Length of the central alkyl chain
b. Lipid solubility of the ionized form
c. Lipid solubility of the unionized form
d. Ester linkage between the aromatic nucleus and the alkyl chain
e. Amide linkage between the aromatic nucleus and the alkyl chain

A

(c) only options (b) and (c) are relevant here - the others have
nothing to do with LA penetration into membranes. Membrane
permeability is affected by whether or not the molecule is
“charged” or ionized or not (e.g., unionized). Only the latter form
passes readily through membranes. See, they’re asking the
same thing they asked in the previous question, just coming at it
from another angle. Remember the fact and you can cover the
angles.

How well did you know this?
1
Not at all
2
3
4
5
Perfectly
102
Q

At a pH of 7.8, lidocaine (pKa = 7.8) will exist in

a. the ionized form
b. the nonionized form
c. an equal mixture of ionized and nonionized forms
d. a mixture 10 times more ionized than nonionized forms

A

(c) the ratio of ionized to unionized forms is given by the
formula log A/AH= pH-pKa. In this instance the difference
between pH and pKa is 0. Thus lidocaine will exist as an equal
mixture ( so (c) is correct). Most local anesthetics are weak
bases with pKa ranging from 7.5 to 9.5. LA’s intended for
injection are usually prepared in salt form by addition of HCl.
They penetrate as the unionized form into the neuron where
they re-equilibrate to both charged and uncharged forms inside the neuron - the positively charged ion blocks nerve conduction.

How well did you know this?
1
Not at all
2
3
4
5
Perfectly
103
Q

The more rapid onset of action of local anesthetics in small nerves is
due to
a. The slightly lower pH of small nerves
b. The greater surface-volume ratio of small nerves
c. The increased rate of penetration resulting from depolarization
d. Smaller nerves usually having a higher threshold

A

(b) - the theory
goes that there is a size dependent critical length of
anesthetic exposure necessary to block a given nerve. Small
fibers will be blocked first because the anesthetic
concentration to h critical length in a small fiber will be reached
faster than the critical length in a larger fiber. You have to
block three nodes of ranvier, and they are farther apart in
larger fibers than they are in small diameter fibers.

How well did you know this?
1
Not at all
2
3
4
5
Perfectly
104
Q

Which of the following statements are true regarding onset, degree
and duration of action of local anesthetics?
a. The greater the drug concentration, the faster the onset and the
greater the degree of effect
b. Local anesthetics block only myelinated nerve fibers at the nodes
of Ranvier
c. The larger the diameter of the nerve fiber, the faster the onset of
effect
d. The faster the penetrance of the drug, the faster the onset of
effect
i. (a), (b), and (c)
ii. (a), (b) and (d)
iii. (a) and (c) only
iv. (b), (c) and (d)

A

(ii) if you knew the fact above about small nerves, then this
question basically becomes a true false type thing, and (c) is the
false statement. (a) and (d) make logical sense so you are
stuck picking between (b) and (c). You have your pick of
memorizing the small nerve thing or the myelinated nerve nodes
of ranvier thing.

How well did you know this?
1
Not at all
2
3
4
5
Perfectly
105
Q
A dentist administers 1.8 ml of a 2% solution of lidocaine. How many
mg of lidocaine did the patient receive?
a. 3.6
b. 9
c. 18
d. 36
e. 180
A

(d) 2% solution = 20 mg/ml X 1.8 ml = 36 mg lidocaine.

How well did you know this?
1
Not at all
2
3
4
5
Perfectly
106
Q

Three ml of a local anesthetic solution consisting of 2% lidocaine with

1: 100,000 epinephrine contains how many milligrams of each?
a. 6 mg. lidocaine, 0.3 mg. epinephrine
b. 6 mg. lidocaine, 0.03 mg. epinephrine
c. 60 mg. lidocaine 0.3 mg. epinephrine
d. 60 mg. lidocaine 0.03 mg epinephrine
e. 600 mg lidocaine, 0.3 mg. epinephrine
f. 600 mg. lidocaine, 0.03 mg. epinephrine

A

(d) 2% lidocaine = 20 mg/ml x 3 = 60 mg lidocaine

1: 100,000 epi = 0.01 mg/ml x 3 = 0.03 mg epi

How well did you know this?
1
Not at all
2
3
4
5
Perfectly
107
Q
The maximum allowable adult dose of mepivacaine is 300 mg. How
many milliliters of 2% mepivacaine should be injected to attain the
maximal dosage in an adult patient?
a. 5
b. 10
c. 15
d. 20
e. 25
A

(c) 2% mepivacaine = 20 mg/ml, so 300 mg / 20 mg/ml = 15 ml

How well did you know this?
1
Not at all
2
3
4
5
Perfectly
108
Q

A recently introduced local anesthetic agent is claimed by the
manufacturer to be several times as potent as procaine. The product
is available in 0.05% buffered aqueous solution in 1.8 ml. cartridge.
The maximum amount recommended for dental anesthesia over a 4-
hour period is 30 mg. The amount is contained in approximately how
many cartridges?
a. 1-9
b. 10-18
c. 19-27
d. 28-36
e. Greater than 36

A

(d) 0.05% = 0.5 mg/ml . To give 30 mg, you have to give
30mg/0.5 mg/ml or 60 ml. 1 cartridge = 1.8 ml, thus 60ml /1.8ml
= 33.3 cartridges. - first express the percentage of solution as a
fraction of 100, then add the units gm/ml. 0.05% equals 0.5 or
1/2 gms per 100 ml. The cartridge is 1.8 ml which you can
round off to almost 2 mls total. In this 2 ml you would have 1 gm
of the local anesthetic. You need to give 30 gms, which would
require 30 cartridges. The alternative that meets this answer is

(d).

How well did you know this?
1
Not at all
2
3
4
5
Perfectly
109
Q

According to AHA guidelines, the maximum # of carpules of
local anesthetic containing 1:200,000 epinephrine that can be used
in the patient with cardiovascular disease is
a. 1
b. 2
c. 3
d. 4
e. 11

A

(d) the AHA limit is 0.04 mg, compared to 0.2 mg in the healthy
patient. 1:200,000 equals 0.005 mg/ml or 0.009 per 1.8 ml
carpule. 4 carpules would thus contain 0.036 mg, which is just
below the 0.04 mg limit

How well did you know this?
1
Not at all
2
3
4
5
Perfectly
110
Q

Max epi that cardiovascular pts can receive (in mgs)

A

0.04 mg

How well did you know this?
1
Not at all
2
3
4
5
Perfectly
111
Q

Max epi healthy pts can receive (in mgs)

A

0.2 mg

How well did you know this?
1
Not at all
2
3
4
5
Perfectly
112
Q

how much epi in one carpule of 1:200,000

A

1.8 mL /200,000 = 0.9 ug of epi in one carpule - 0.009 mg

How well did you know this?
1
Not at all
2
3
4
5
Perfectly
113
Q

Penicillin V vs Penicillin G

A

G more susceptible to acid degradation, injected more often than peroral

How well did you know this?
1
Not at all
2
3
4
5
Perfectly
114
Q

best G- spectrum among penicillins

A

ampicillin

How well did you know this?
1
Not at all
2
3
4
5
Perfectly
115
Q

cross-allergenic with penicillin

A

ampicillin, cephalosporins

How well did you know this?
1
Not at all
2
3
4
5
Perfectly
116
Q

against penicillinase

A

dicloxacillin

How well did you know this?
1
Not at all
2
3
4
5
Perfectly
117
Q

pseudomonas-specific penicillin

A

carbenicllin (extended spectrum)

How well did you know this?
1
Not at all
2
3
4
5
Perfectly
118
Q

when prophylaxis?

A

NOT for joints anymore! unless complications, then refer to their surgeon
- yes for infective endocarditis; prosthetic valves, hx of infective endocarditis, cardiac transplant; congenital heart defects

How well did you know this?
1
Not at all
2
3
4
5
Perfectly
119
Q

1st line of abx prophylaxis

A

2g of amoxicillin (4 of 500mg) 1 hr before procedure

for kids: 50mg/hr before procedure

How well did you know this?
1
Not at all
2
3
4
5
Perfectly
120
Q

abx if penicillin allergy

A

clindamycin 600 mg 1 hour PO (4 x 150mg)

kids 20 mg/kg

How well did you know this?
1
Not at all
2
3
4
5
Perfectly
121
Q

if can’t peroral:

A

same dosages but 1/2 hr before procedure

How well did you know this?
1
Not at all
2
3
4
5
Perfectly
122
Q

procedures that don’t require prophylaxis

A

restorative, intracanal endo, impressions

How well did you know this?
1
Not at all
2
3
4
5
Perfectly
123
Q

typical penicillin VK prescription

A

250-500mg, q6h for 7 days

kids 20-50 mg/kg, qid

How well did you know this?
1
Not at all
2
3
4
5
Perfectly
124
Q

typical clinda prescription

A

150-300 mg, q8h for 7 days

kids 8-12 mg/kg tid or qid

How well did you know this?
1
Not at all
2
3
4
5
Perfectly
125
Q

typical amoxicillin prescription

A

500mg q8h for 7 days

kids 20-40

How well did you know this?
1
Not at all
2
3
4
5
Perfectly
126
Q

mechanism of penicillins

A

inhibit cell wall synthesis

bactericidal!

How well did you know this?
1
Not at all
2
3
4
5
Perfectly
127
Q

tetracycline: cidal or static?

A

static!

How well did you know this?
1
Not at all
2
3
4
5
Perfectly
128
Q

mechanism of tetracycline

A

inhibit protein synthesis on ribosomes; static

How well did you know this?
1
Not at all
2
3
4
5
Perfectly
129
Q

mechanism of antifungals

A

bind to ergosterol, weaken cell wall (like Nystatin)

How well did you know this?
1
Not at all
2
3
4
5
Perfectly
130
Q

nystatin class

A

antifungal

How well did you know this?
1
Not at all
2
3
4
5
Perfectly
131
Q

sulfonamides static or cidal?

A

static

How well did you know this?
1
Not at all
2
3
4
5
Perfectly
132
Q

sulfonamides mechanism

A

compete with PABA in folic acid synthesis

How well did you know this?
1
Not at all
2
3
4
5
Perfectly
133
Q

which drugs involved in folic acid synthesis

A

sulfonamides

How well did you know this?
1
Not at all
2
3
4
5
Perfectly
134
Q

allergy to penicillins

A

dermatitis, stomatitis, bronchoconstriction and cardiovascular
collapse

How well did you know this?
1
Not at all
2
3
4
5
Perfectly
135
Q

clindamycin side effects

A

GI upset and pseudomonas infection

How well did you know this?
1
Not at all
2
3
4
5
Perfectly
136
Q

GI upset and Pseudomonas infection whose side effects

A

clindamycin

How well did you know this?
1
Not at all
2
3
4
5
Perfectly
137
Q

most likely to give superinfection

A

broad spectrum, like tetracyclines

How well did you know this?
1
Not at all
2
3
4
5
Perfectly
138
Q

how broad in penicillin G spectrum>? superinfx likely?

A

narrow, unlikely

How well did you know this?
1
Not at all
2
3
4
5
Perfectly
139
Q

aplastic anemia whose side effects

A

chloramphenicol

How well did you know this?
1
Not at all
2
3
4
5
Perfectly
140
Q

chloramphenicol side effect

A

aplastic anemia

How well did you know this?
1
Not at all
2
3
4
5
Perfectly
141
Q

organ toxicity of tetracyclin

A

liver

How well did you know this?
1
Not at all
2
3
4
5
Perfectly
142
Q

allergic cholestatic hepatitis

A

erythromycin

How well did you know this?
1
Not at all
2
3
4
5
Perfectly
143
Q

erythromycin - cross-reactive with penicillin?

A

no

How well did you know this?
1
Not at all
2
3
4
5
Perfectly
144
Q

eryhtromycin allergy

A

cholestatic hepatitis

How well did you know this?
1
Not at all
2
3
4
5
Perfectly
145
Q

tetracyclin + penicllin interaction

A

cancel each other: cidal vs static

How well did you know this?
1
Not at all
2
3
4
5
Perfectly
146
Q

how is penicillin eliminated

A

renal

How well did you know this?
1
Not at all
2
3
4
5
Perfectly
147
Q

which drug can alter penicillin elimination

A

pribenecid

How well did you know this?
1
Not at all
2
3
4
5
Perfectly
148
Q

probenecid with penicillin elimination

A

can alter

How well did you know this?
1
Not at all
2
3
4
5
Perfectly
149
Q

what can reduce tetracylin effectiveness

A

antacids, dairy

How well did you know this?
1
Not at all
2
3
4
5
Perfectly
150
Q

antacids and dairy on tetracyclin

A

can reduce effectiveness

How well did you know this?
1
Not at all
2
3
4
5
Perfectly
151
Q

ampicillin and oral contraceptives

A

ampicillin decrease the effectiveness of
oral contraceptives due to suppression of normal Gl flora
involved in the recycling of active steroids from bile conjugates,
leading to more rapid excretion of the steroids from the body

How well did you know this?
1
Not at all
2
3
4
5
Perfectly
152
Q

erythromycin class

A

macrolide

How well did you know this?
1
Not at all
2
3
4
5
Perfectly
153
Q

macrolides can inhibit which drugs

A

seldane, digoxin

How well did you know this?
1
Not at all
2
3
4
5
Perfectly
154
Q

herpes tx

A

acyclovir

How well did you know this?
1
Not at all
2
3
4
5
Perfectly
155
Q

candidiasis tx

A

fluconazole, ketoconazole

How well did you know this?
1
Not at all
2
3
4
5
Perfectly
156
Q

fluconazole, ketoconazole class

A

systemic antifungals

How well did you know this?
1
Not at all
2
3
4
5
Perfectly
157
Q

For treating most oral infections, penicillin V is preferred to penicillin G
because penicillin V
a. Is less allergenic
b. Is less sensitive to acid degradation
c. Has a greater gram-negative spectrum
d. Has a longer duration of action
e. Is bactericidal, whereas penicillin G is not

A

b

How well did you know this?
1
Not at all
2
3
4
5
Perfectly
158
Q

The sole therapeutic advantage of penicillin V over penicillin G is

a. Greater resistance to penicillinase
b. Broader antibacterial spectrum
c. More reliable oral absorption
d. Slower renal excretion
e. None of the above

A

c

How well did you know this?
1
Not at all
2
3
4
5
Perfectly
159
Q
Which of the following penicillins is administered ONLY by deep
intramuscular injection?
a. Ampicillin
b. Dicloxacillin sodium
c. Penicillin G procaine
d. Penicillin V potassium
A

c; Penicillin G
is destroyed by acid in the stomach resulting in variable and
irregular absorption. Penicillin V is acid stable and available for
oral use. Penicillin G procaine is typically given intramuscularly
in repository form, yielding a tissue depot from which the drug is
absorbed over hours. In this form, it cannot be given IV or
subcutaneously.

How well did you know this?
1
Not at all
2
3
4
5
Perfectly
160
Q

The principal difference among potassium, procaine and benzathine
salts of penicillin G is their
a. Potency
b. Toxicity
c. Duration of action
d. Antibacterial spectrum
e. Diffusion into the cerebrospinal fluid

A

(c) again, just asking you to know something about the various
forms of penicillin. Since in most cases you are going to use Pen
VK orally, this question is an old one showing its age and probably
not likely to appear anymore on board excams

How well did you know this?
1
Not at all
2
3
4
5
Perfectly
161
Q

Which of the following antibiotics is cross-allergenic with penicillin
and should NOT be administered to the penicillin-sensitive patient?
a. Ampicillin
b. Erythromycin
c. Clindamycin
d. Lincomycin
e. Tetracycline

A

a

How well did you know this?
1
Not at all
2
3
4
5
Perfectly
162
Q
Which of the following antibiotics may be cross-allergenic with
penicillin?
a. Neomycin
b. Cephalexin
c. Clindamycin
d. Erythromycin
e. All of the above
A

b

How well did you know this?
1
Not at all
2
3
4
5
Perfectly
163
Q
  1. Which of the following antibiotics shows an incidence of
    approximately 8% cross-allergenicity with penicillins?
    a. neomycin
    b. cephalexin
    c. bacitracin
    d. vancomycin
    e. tetracycline
A

b

How well did you know this?
1
Not at all
2
3
4
5
Perfectly
164
Q

Which of the following groups of antibiotics is related both structurally
and by mode of action to the penicillins?
a. Polymyxins
b. Cycloserines
c. Cephalosporins
d. Chloramphenicols

A

c

How well did you know this?
1
Not at all
2
3
4
5
Perfectly
165
Q

For the dentist, the most reliable method of detecting a patient’s
allergy to penicillin is by
a. Injecting penicillin intradermally
b. Taking a thorough medical history
c. Placing a drop of penicillin on the eye
d. Having the patient inhale a penicillin aerosol
e. Injecting a small amount of penicillin intravenously

A

(b) all of the other methods involve unacceptable risk. Once
sensitized, even a small amount can cause an allergic
response. Remember, it is not a dose-related response that
won’t be problematic if you only inject a little bit.

How well did you know this?
1
Not at all
2
3
4
5
Perfectly
166
Q

Which of the following antibiotics is the substitute of choice for
penicillin in the penicillin-sensitive patient?
a. Bacitracin
b. Erythromycin
c. Tetracycline
d. Chloramphenicol

A

CLINDAMYCIN IS BEST BUT
b
reason (b) is the right answer is that the

spectrum of activity of erthromycin is very similar to penicillin. The
others offer a much broader spectrum of coverage than we usually
require; always use the drug with the narrowest spectrum possible
that includes the microbe in question. Standards have now
changed such that clindamycin is the drug of choice in this
situation. But if they don’t include clindamycin, look for
erythromycin, or for that matter Azithromycin

How well did you know this?
1
Not at all
2
3
4
5
Perfectly
167
Q

Most anaphylactic reactions to penicillin occur
a. When the drug is administered orally
b. In patients who have already experienced an allergic reaction to
the drug
c. In patients with a negative skin test to penicillin allergy
d. When the drug is administered parenterally
e. Within minutes after drug administration
i. (a), (b) and (d)
ii. (b), (c) and (d)
iii. (b), (d) and (e)
iv. (b) and (e) only
v. (c), (d) and (e)

A

iii

How well did you know this?
1
Not at all
2
3
4
5
Perfectly
168
Q
Which of the following penicillins has a broader gram-negative
spectrum than penicillin G?
a. Nafcillin
b. Ampicillin
c. Cephalexin
d. Methicillin
e. Penicillin V
A

b

How well did you know this?
1
Not at all
2
3
4
5
Perfectly
169
Q
Which of the following penicillins has the best gram-negative
spectrum?
a. Nafcillin
b. Ampicillin
c. Methicillin
d. Penicillin V
e. Phenethicillin
A

b

How well did you know this?
1
Not at all
2
3
4
5
Perfectly
170
Q

Which of the following antibiotics should be considered the drug of

choice in the treatment of infection caused by a penicillinase-
producing staphylococcus?

a. Neomycin
b. Ampicillin
c. Tetracycline
d. Penicillin V
e. Dicloxacillin

A

e that’s really the only use for dicloxacillin

How well did you know this?
1
Not at all
2
3
4
5
Perfectly
171
Q
Oral infections caused by organisms that produce penicillinase
should be treated with
a. Ampicillin
b. Dicloxacillin
c. Erythromycin
d. Any of the above
e. Only (a) or (c) above
A

(b) of those listed only (b) is penicillinase resistant. Ampicillin is an
extended spectrum penicillin, and is not penicillinase resistant.
Erythromycin shouldn’t be affected by penicillinases, since it isn’t a
peniciilin, but it doesn’t work against staph for other reasons.

How well did you know this?
1
Not at all
2
3
4
5
Perfectly
172
Q

does erythromycin work against staph?

A

no

How well did you know this?
1
Not at all
2
3
4
5
Perfectly
173
Q
Which of the following antibiotics is LEAST effective against
penicillinase-producing microorganisms?
a. Ampicillin
b. Cephalexin
c. Methicillin
d. Clindamycin
e. Erythromycin
A

a

How well did you know this?
1
Not at all
2
3
4
5
Perfectly
174
Q
Which of the following is a bactericidal antibiotic used specifically in
the treatment of infections caused by Pseudomonas species and
indole-positive Proteus species?
a. Ampicillin
b. Penicillin V
c. Tetracycline
d. Dicloxacillin
e. Carbenicillin
A

e

How well did you know this?
1
Not at all
2
3
4
5
Perfectly
175
Q
Penicillin's effectiveness against rapidly growing cells is primarily due
to its effect on
a. Protein synthesis
b. Cell wall synthesis
c. Nucleic acid synthesis
d. Chelation of metal ions
e. Cell membrane permeability
A

b

How well did you know this?
1
Not at all
2
3
4
5
Perfectly
176
Q

Chlortetracycline acts by interfering with

a. Cell wall synthesis
b. Nuclear acid synthesis
c. Protein synthesis on bacterial but not mammalian ribosomes
d. Protein synthesis on mammalian but not bacterial ribosomes

A

c

How well did you know this?
1
Not at all
2
3
4
5
Perfectly
177
Q

The probable mechanism of the bacteriostatic action of sulfonamides
involves
a. Disruption of the cell membrane
b. Coagulation of intracellular proteins
c. Reduction in oxygen utilization by the cells
d. Inhibition of metabolism by binding acetyl groups
e. Competition with para-aminobenzoic acid in folic acid synthesis

A

e

How well did you know this?
1
Not at all
2
3
4
5
Perfectly
178
Q

The sulfonamides act by

a. Suppressing bacterial protein synthesis
b. Inhibiting the formation of the cytoplasmic bacterial membrane
c. Inducing the formation of “lethal” bacterial proteins

d. Inducing a deficiency of folic acid by competition with para-
aminobenzoic acid

A

d

How well did you know this?
1
Not at all
2
3
4
5
Perfectly
179
Q
Which antibiotic is able to achieve a higher concentration in bone
than in serum?
a. penicillin
b. erythromycin
c. clindamycin
d. metronidazole
e. amoxicillin
A

c

How well did you know this?
1
Not at all
2
3
4
5
Perfectly
180
Q

drig that is able to achiebe high concentration in gingival crevicular fluid

A

tetracycline

How well did you know this?
1
Not at all
2
3
4
5
Perfectly
181
Q

Tetracycline reduces the effectiveness of concomitantly administered
penicillin by
a. Reducing absorption of penicillin
b. Increasing metabolism of penicillin
c. Increasing renal excretion of penicillin
d. Increasing binding of penicillin to serum proteins
e. None of the above

A

(e) tetracycline is bacteriostatic and would slow the rapid
growth of the microbial population that a bactericidal drug such
as penicillin needs to be effective, sine only when rapidly
dividing are the cells making cell walls

How well did you know this?
1
Not at all
2
3
4
5
Perfectly
182
Q

The action of which of the following drugs will most likely be impaired
by concurrent administration of tetracycline?
a. Clarithromycin
b. Erythromycin
c. Sulfonamide
d. Penicillin
e. Lincomycin

A

d

How well did you know this?
1
Not at all
2
3
4
5
Perfectly
183
Q
Which of the following antibiotics is most likely to cause liver
damage?
a. Streptomycin
b. Penicillin G
c. Tetracycline
d. Cephalosporins
e. Amphotericin B
A

(c) (a) streptomycin can damage the eighth nerve, affecting
both balance and hearing, but is not associated with liver
damage. (b) other than allergic reactions, penicillins are
extremely safe, with no effect on the liver. (d) the
cephalosporins are chemically related to the penicillins and
share their relatively nontoxic nature. (e) amphotericin B, is an
antifungal agent that produces such adverse side effects as
nephrotoxicity and hypokalemia, but not liver toxicity. Thus (c) is
the correct answer. Tetracyclines have been shown to be
hepatotoxic following high doses in pregnant patients with a
history of renal disease.

How well did you know this?
1
Not at all
2
3
4
5
Perfectly
184
Q

toxicity of streptomycin

A

balance and hearing

How well did you know this?
1
Not at all
2
3
4
5
Perfectly
185
Q

toxicity of amphetericin B

A

renal + hypoKemia

How well did you know this?
1
Not at all
2
3
4
5
Perfectly
186
Q
Which of the following erythromycins associated with an allergic
cholestatic hepatitis?
a. Erythromycin base
b. Erythromycin stearate
c. Erythromycin estolate
d. Erythromycin succinate
A

c

How well did you know this?
1
Not at all
2
3
4
5
Perfectly
187
Q
Which of the following antibiotics is LEAST likely to cause
superinfection?
a. Gentamicin
b. Tetracycline
c. Penicillin G
d. Streptomycin
e. Chloramphenicol
A

c bc narrow

How well did you know this?
1
Not at all
2
3
4
5
Perfectly
188
Q
Gastrointestinal upset and pseudomembranous colitis has been
prominently associated with
a. Nystatin
b. Cephalexin
c. Clindamycin
d. Polymyxin B
e. Erythromycin
A

(c) The only 2 possibilities that produce GI upset are (c) and
(e). As for producing colitis, (b) and (c) are associated with this
adverse side effect. (c) is the only drug which does both,
therefore it’s the right answer.

How well did you know this?
1
Not at all
2
3
4
5
Perfectly
189
Q

Symptoms that may be characterized as allergic manifestations
during penicillin therapy are
a. Deafness, dizziness and acute anemia
b. Crystalluria, nausea, vomiting and anaphylactic shock
c. Oliguria, hematuria, bronchoconstriction and cardiovascular
collapse
d. Dermatitis, stomatitis, bronchoconstriction and cardiovascular
collapse

A

d

How well did you know this?
1
Not at all
2
3
4
5
Perfectly
190
Q

Aplastic anemia is a serious toxic effect that occurs particularly after
a course of treatment with which of the following antibiotics?
a. Penicillin
b. Lincomycin
c. Tetracycline
d. Streptomycin
e. Chloramphenicol

A

e

How well did you know this?
1
Not at all
2
3
4
5
Perfectly
191
Q

Each of the following is a side effect of prolonged tetracycline
hydrochloride therapy EXCEPT:
a. Suprainfection
b. Photosensitivity
c. Vestibular disturbances
d. Discoloration of newly forming teeth
e. Gastrointestinal symptoms (when administered orally)

A

c

How well did you know this?
1
Not at all
2
3
4
5
Perfectly
192
Q

tetracycline side effects

A

Suprainfection

b. Photosensitivity
d. Discoloration of newly forming teeth
e. Gastrointestinal symptoms (when administered orally)

How well did you know this?
1
Not at all
2
3
4
5
Perfectly
193
Q
Colitis that results following clindamycin therapy is caused by an
overgrowth of
a. C. dificile
b. Staph aureus
c. Pseudomonas
d. Candida albicans
A

a

How well did you know this?
1
Not at all
2
3
4
5
Perfectly
194
Q
Which antibiotic is appropriate for premedication in the penicillin
allergic patient?
a. Cephalexin
b. Clindamycin
c. Erythromycin
d. Amoxicillin
e. Ampiciilin
A

b

How well did you know this?
1
Not at all
2
3
4
5
Perfectly
195
Q
  1. Acyclovir is useful for treating
    a. Candidiasis
    b. Colitis
    c. Herpes simplex
    d. HIV
    e. ANUG
A

c

How well did you know this?
1
Not at all
2
3
4
5
Perfectly
196
Q

A distinct advantage that tetracyclines have over penicillins is that
tetracyclines
a. Have no side effects
b. Do not cause superinfections
c. Are safer to use during pregnancy
d. Have a wider range of antibacterial activity
e. Produce higher blood levels faster after oral administration

A

(d) broad spectrum vs. narrow spectrum. Tetracyclines certainly
have more side effects than penicillin, and are certainly one of the antibiotics to avoid during pregnancy.

How well did you know this?
1
Not at all
2
3
4
5
Perfectly
197
Q

Which of the following has the broadest antimicrobial spectrum?

a. Vancomycin (Vancocin )
b. Clindamycin (Cleocin )
c. Erythromycin (Erythrocin )
d. Chlortetracycline (Aureomycin )
e. A third generation cephalosporin

A

(d) Answer is (d)- remember, tetracyclines are broad spectrum

antibiotics effective against both gram-negative and gram-
positive cocci and bacilli. Clindamycin has a spectrum of activity

similar to erthyromycin and vancomycin, which is less than that
of the tetracylines, mainly affecting gram-positive
microorganisms. Ist generation cephalosporins are effective
against both gram-negative and gram-positive organisms, but

the third generation ones have increased activity against gram-
negative but greatly decreased activity against gram-positive

microorganisms.

How well did you know this?
1
Not at all
2
3
4
5
Perfectly
198
Q

Sulfonamides and trimethoprim are synergistic bacteriostatic agents
because in bacteria they
a. Both inhibit folic acid synthesis
b. Interfere sequentially with folinic acid production
c. Are both antimetabolites of para-aminobenzoic
d. Are both inhibitors of dihydrofolic acid reductase
e. Are both transformed in vivo into a single active compound

A

b

How well did you know this?
1
Not at all
2
3
4
5
Perfectly
199
Q

Which of the following substances is the most effective agent against
fungus infections of the mucous membrane?
a. Nystatin ointment
b. Undecylenic acid
c. Polymyxin ointment
d. Saturated magnesium sulfate
e. 10 per cent aluminum chloride solution

A

a

How well did you know this?
1
Not at all
2
3
4
5
Perfectly
200
Q

The most desirable property of an antibiotic when used to treat an
odontogenic infection is
a. Rapid absorption
b. Little allergenicity
c. Ability to achieve and maintain adequate concentrations at the
site of infection
d. Lack of significant binding to plasma proteins
e. No effects on drug metabolism

A

c

How well did you know this?
1
Not at all
2
3
4
5
Perfectly
201
Q
  1. Nystatin is of greatest clinical usefulness in treating
    a. viral infections
    b. fungal infections
    c. spirochetal infections
    d. Bacterroides infections
    e. penicillin resistant gram positive infections
A

b

How well did you know this?
1
Not at all
2
3
4
5
Perfectly
202
Q

Which of the following drugs chelates with calcium?

a. Erythromycin
b. Polymyxin B
c. Tetracycline
d. Penicillin G
e. Chloramphenicol

A

c

How well did you know this?
1
Not at all
2
3
4
5
Perfectly
203
Q

Which of the following is NOT characteristic of tetracycline antibiotics?
a. Absorption is impaired when taken with antacids
b. Theypredisposetomonilialsuperinfection
c. They form a stable complex with the developing tooth matrix
d. They have a low tendency for sensitization, but a high
therapeutic index
e. They are effective substitutes for penicillin prophylaxis against
infective endocarditis

A

Answer is (e)- Again, the important phrase in the question is not (Hey, just Wayne and Garth). Obviously the fact that you will remember about tetracylines is that they can discolor teeth in the fetus when taken by the mother during pregnancy. But don’t circle that answer because (a) is also characteristic of tetracyclines (they are the most likely of all the antibiotics to cause superinfection), and is an annoying side effect in adults resulting from alteration of the oral, gastric and intestinal flora. The real answer is (e). Tetracyclines are not the drug of choice for prophylaxis against infective endocarditis. This is due to streptococcal infection. 15-20% of group A streptococci are resistant to tetracyclines, but none are resistant to penicillin or erythromycin. Recently a non-streptococcal induced subacute bacterial endocarditis has been identified, especially in juvenile periodontitis patients. The causative bacterium is not susceptible to penicillin or erythromycin. It may be necessary to treat predisposed patients with tetracycline for a few weeks, and then follow this with a course of penicillin or erythromycin. Remember that these drugs are antagonistic to each other and thus can’t be used concurrently. Penicillin is a bactericidal drug which kills or destroys microorganisms by interfering with the synthesis or function of the cell wall, cell membrane or both. Thus it is most effective against bacteria that are multiplying. Tetracycline is a bacteriostatic antibiotic that acts by inhibiting the growth and multiplication of organisms by inhibiting protein synthesis by binding reversibly to the 30 S subunit of the bacterial ribosome. When the two types are given together, their effectiveness is negated or reduced.

How well did you know this?
1
Not at all
2
3
4
5
Perfectly
204
Q

The concurrent administration of penicillin G and probenecid results in
a. Increased metabolism of penicillin G.
b. Increased renal excretion of probenecid
c. Decreased renal excretion of penicillin G
d. Decreased bactericidal effect of penicillin G
e. Increased excretion of probenecid in the feces

A

C

How well did you know this?
1
Not at all
2
3
4
5
Perfectly
205
Q

Interaction between penicillin and probenicid is best described by which of the following mechanisms?
a. competition at the receptor site
b. acceleration of drug biotransformation c. alteration in the acid-base balance
d. alteration in the rate of renal clearance

A

Answer is (d)- penicillin is metabolized in the liver, but it rapidly disappears from the blood due to rapid clearance by the kidneys. 90% is excreted by tubular secretion. Thus patients with renal disease will show high blood levels of penicillin. Similarly, probenicid, a uricosuric agent (a drug which tends to enhance the excretion of uric acid by reducing renal tubular transport mechanisms), reduces the renal clearance of penicillins. And you wondered why we had those lectures on pharmacokinetics!

How well did you know this?
1
Not at all
2
3
4
5
Perfectly
206
Q
  1. When broad-spectrum antibiotics are administered with coumarin anticoagulants, the anticoagulant action may be
    a. b. c. d.
    Reduced because of enhanced hepatic drug metabolism
    Reduced because of increased protein-binding Increased because of reduction of vitamin K sources Increasedbecauseofdecreasedrenalexcretionofthe anticoagulant
A

C

How well did you know this?
1
Not at all
2
3
4
5
Perfectly
207
Q

therapeutic effectiveness of which of the following drugs will be
most affected by concomitant ingestion of antacids?
a. Cephalexin b. Erythromycin c. Tetracycline d. Sulfisoxazole e. Penicillin V

A

C

How well did you know this?
1
Not at all
2
3
4
5
Perfectly
208
Q

Erythromycin should be avoided in the patient taking
a. Aspirin
b. Seldane
c. Benadryl
d. Ibuprofen
e. Propranolol

A

(b) remember the famous erythromycin –Seldane potentially lethal interaction, whereby erythromycin blocks the metabolism of Seldane to its antihistamine metabolite – it stays unmetabolized and causes cardiac arrthymias. Of course this question could have many other options listed, since erythromycin decreases the metabolism of so many other useful drugs, such as digoxin.

How well did you know this?
1
Not at all
2
3
4
5
Perfectly
209
Q

Atenolol class and use

A

Cardioselective beta-blocker, HTN

metoprolol too

reduces cardiac output

How well did you know this?
1
Not at all
2
3
4
5
Perfectly
210
Q

Propranolol class use

A

non-selective beta-blocker, htn, angina, also paroxysmal tachy

reduces cardiac output and renin release

How well did you know this?
1
Not at all
2
3
4
5
Perfectly
211
Q

Lisinopril class use

A

Ace inhibitor, HTN

also congestive heart failure

How well did you know this?
1
Not at all
2
3
4
5
Perfectly
212
Q

Furosemide class use

A

Loop diuretic, HTN

How well did you know this?
1
Not at all
2
3
4
5
Perfectly
213
Q

Prazosine class use

A

selective Alpha-1 - blocker, HTN

inhibits NE release

How well did you know this?
1
Not at all
2
3
4
5
Perfectly
214
Q

Guanethidine class use

A

Neuro inhibitor, severe HTN

How well did you know this?
1
Not at all
2
3
4
5
Perfectly
215
Q

Cardioselective beta-blockers

A

Metaprolol, atenolol

How well did you know this?
1
Not at all
2
3
4
5
Perfectly
216
Q

Angina meds

A

Nitroglycerin, propranolol, Ca-channel blockers (verapamil)

How well did you know this?
1
Not at all
2
3
4
5
Perfectly
217
Q

Verapamil class use

A

Ca-channel blocker, angina

also supraventricular tachyarrhythmia, paroxysmal tachycardia, afib

How well did you know this?
1
Not at all
2
3
4
5
Perfectly
218
Q

lidocaine use

A

ventricular arrhythmias

How well did you know this?
1
Not at all
2
3
4
5
Perfectly
219
Q

phenytoin use

A

reverse digitalis induced arrhythmias

How well did you know this?
1
Not at all
2
3
4
5
Perfectly
220
Q

quinidine use

A

supraventricular tachyarrhythm, afib

How well did you know this?
1
Not at all
2
3
4
5
Perfectly
221
Q

digitalis (=digoxin) use

A

afib, paroxysmal tachy

it’s a glycoside

How well did you know this?
1
Not at all
2
3
4
5
Perfectly
222
Q

tx for ventircular arrhythmias

A

lidocaine

How well did you know this?
1
Not at all
2
3
4
5
Perfectly
223
Q

tx to reverse digitalis induced arrhythmia

A

phenytoin

How well did you know this?
1
Not at all
2
3
4
5
Perfectly
224
Q

tx for supraventricular tachyarrhythmias

A

quinidine, verapamil

How well did you know this?
1
Not at all
2
3
4
5
Perfectly
225
Q

tx for afib

A

quinidine, verapamil, digoxin

How well did you know this?
1
Not at all
2
3
4
5
Perfectly
226
Q

tx for paroxysmal tachycardia

A

verapamil, digoxin, propranolol

How well did you know this?
1
Not at all
2
3
4
5
Perfectly
227
Q

tx for congestive heart failure

A

glycosides (digoxin, digitalis) and ACE inhibitors (captopril)

How well did you know this?
1
Not at all
2
3
4
5
Perfectly
228
Q

glycosides use

A

digitalis, digoxin – congestive heart failure;

afib, paroxysmal tachy

How well did you know this?
1
Not at all
2
3
4
5
Perfectly
229
Q

type 1a antiarrhythmics - example, mechanism

A

quinidine: prolongs refractory period

How well did you know this?
1
Not at all
2
3
4
5
Perfectly
230
Q

type 2a antiarrhytmics - example, mechanism

A

lidocaine: decreases excitability

How well did you know this?
1
Not at all
2
3
4
5
Perfectly
231
Q

digitalis mechanism

A

for afib: decreses rate of AV conduction

for congestive heart failure: increase force of contraction (+ inotropic) –> inhibit Na/K ATPase –> increased Ca influx. reduce compensatory changes in CHF (heart size, rate, edema)

How well did you know this?
1
Not at all
2
3
4
5
Perfectly
232
Q

what is angina:

A

insufficient oxygen for myocardium

How well did you know this?
1
Not at all
2
3
4
5
Perfectly
233
Q

nitroglycerin mechanism

A

direct vasodilatory action on smooth muscla in coronary arteries –> increases oxygen supply

How well did you know this?
1
Not at all
2
3
4
5
Perfectly
234
Q

propranolol mechanism

A

reduces oxygen demand – reduces chronotropic response to epi and stress

How well did you know this?
1
Not at all
2
3
4
5
Perfectly
235
Q

Ca channel blockers mechanism

A

decrease oxygen demand –> reduce afterload –> reduce peripheral resistance via vasodilation

How well did you know this?
1
Not at all
2
3
4
5
Perfectly
236
Q

ACE inhibitors mechanism

A

blocks conversion of angiotensin 1 to 2 (latter is potent constrictor)

How well did you know this?
1
Not at all
2
3
4
5
Perfectly
237
Q

methyldopa class mechanism

A

alpha 2 agonist, centrally reduces sympathetic outflow

How well did you know this?
1
Not at all
2
3
4
5
Perfectly
238
Q

cloidine class mechanism

A

alpha 2 agonist, centrally reduces sympathetic outflow

How well did you know this?
1
Not at all
2
3
4
5
Perfectly
239
Q

diuretics mechanism

A

decrease the renal absorption of sodium, thus resulting
in fluid loss and a reduction in blood volume. This decreases
the work the heart has to pump. Also have weak dilatory
action.

How well did you know this?
1
Not at all
2
3
4
5
Perfectly
240
Q

types of diuretics and examples

A

thiazides - chlorothiazide
loop - furosemide
K sparing - spironolactone

How well did you know this?
1
Not at all
2
3
4
5
Perfectly
241
Q

spironolactone class

A

K sparing diuretic

How well did you know this?
1
Not at all
2
3
4
5
Perfectly
242
Q

Quinidine is principally used to treat

a. Hypertension
b. Angina pectoris
c. Congestive hear failure
d. Supraventricular tachyarrhythmias

A

(d) by elimination. Hypertension ((a)) is treated primarily with
beta blockers such as propranolol. Angina is primarily treated
with nitroglycerin, while digoxin (digitalis) is the drug of choice for
congestive heart failure. Quinidine is classed as an antiarrthymic
drug (Type I-blocks sodium channels). It reduces automaticity
and responsiveness and increases refractoriness. It also has an
antimuscarinic action preventing the bradycardia that follows
vagal stimulation.

How well did you know this?
1
Not at all
2
3
4
5
Perfectly
243
Q

Quinidine is used to treat

a. Hypertension
b. Angina pectoris
c. Atrial fibrillation
d. Ventricular fibrillation
e. Congestive hear failure

A

c

How well did you know this?
1
Not at all
2
3
4
5
Perfectly
244
Q

tx for ventricular arrhythmias

A

lidocaine

How well did you know this?
1
Not at all
2
3
4
5
Perfectly
245
Q

Verapamil is most efficacious in the treatment of

a. Atrial fibrillation
b. Atrial tachycardia
c. Ventricular tachycardia
d. Catecholamine-induced arrhythmias

A

a

How well did you know this?
1
Not at all
2
3
4
5
Perfectly
246
Q
Which of the following drugs is most useful in treating or preventing
angina pectoris?
a. Digitalis
b. Quinidine
c. Propranolol
d. Procainamide
e. Pentobarbital
A

c

How well did you know this?
1
Not at all
2
3
4
5
Perfectly
247
Q
Each of the following drugs can be used in the prevention and
treatment of angina pectoris EXCEPT
a. Digitalis
b. Propranolol
c. Nitroglycerin
d. Isosorbide dinitrate
e. Pentaerythritol tetranitrate
A

a

How well did you know this?
1
Not at all
2
3
4
5
Perfectly
248
Q
All of the following drugs are useful in the treatment of hypertension
EXCEPT
a. Ephedrine
b. Reserpine
c. Methyldopa
d. Thiazide diuretics
A

a

How well did you know this?
1
Not at all
2
3
4
5
Perfectly
249
Q

Digitalis is useful in the treatment of which of the following conditions?

a. Atrial fibrillation
b. Congestive heart failure
c. Paroxysmal atrial tachycardia
d. All of the above

A

d

How well did you know this?
1
Not at all
2
3
4
5
Perfectly
250
Q

All of the following drugs are useful in the treatment of cardiac arrhythmias EXCEPT

a. Digitalis
b. Lidocaine
c. Phenytoin
d. Procainamide
e. Aminophylline

A

e

How well did you know this?
1
Not at all
2
3
4
5
Perfectly
251
Q

The drug of choice for initial therapy for mild hypertension is

a. Reserpine
b. Guanethidine
c. Phenobarbital
d. Chlorothiazide
e. lpha-methyldopa

A

d

How well did you know this?
1
Not at all
2
3
4
5
Perfectly
252
Q

Which of the following antihypertensives are usually reserved for
treatment of severe hypertension?
a. Sedatives and reserpine
b. Thiazide diuretics and reserpine
c. Sedatives and thiazide diuretics
d. Guanethidine and ganglionic blocking agents

A

d

How well did you know this?
1
Not at all
2
3
4
5
Perfectly
253
Q
Which of the following beta-adrenergic receptor blocking agents is
thought to be cardioselective?
a. Nadolol
b. Timolol
c. Metoprolol
d. Propranolol
A

c

How well did you know this?
1
Not at all
2
3
4
5
Perfectly
254
Q

Antiarrhythmic drugs, such as quinidine, suppress certain cardiac
arrhythmias by
a. Stimulating the beta-adrenergic receptor
b. Suppressing cardiac ATP-ase activity
c. Increasing ectopic pacemaker activity
d. Increasing the refractory period of cardiac muscle

A

d

How well did you know this?
1
Not at all
2
3
4
5
Perfectly
255
Q

Most drugs useful in the treatment of cardiac arrhythmias act
primarily by
a. Blocking Purkinje fibers
b. Blocking the alpha-adrenergic receptor
c. Suppressing SA node impulse formation
d. Causing a positive inotropic effect
e. Increasing the refractory period of cardiac muscle

A

e

How well did you know this?
1
Not at all
2
3
4
5
Perfectly
256
Q

The most important pharmacologic action of drugs that suppress
cardiac arrhythmias is
a. Blockade of the vagus nerve
b. Stimulation of cardiac ATP-ase activity
c. Blockade of the Beta-adrenergic receptor
d. Stimulation of the beta-adrenergic receptor
e. Increased refractory period of cardiac muscle

A

e

How well did you know this?
1
Not at all
2
3
4
5
Perfectly
257
Q

Lidocaine produces its antiarrhythmic effects by

a. Increasing AV conduction
b. Decreasing cardiac excitability
c. Increasing cardiac conduction velocity
d. Increasing spontaneous pacemaker activity

A

(b) arrhythmias are defined as any abnormality of the normal sinus rhythm of the heart due to disease or injury induced
damage to the impulse conducting systems. They also result
from the development of ectopic pacemakers or abnormal
pacemaker rhythms. Drugs such as lidocaine are used to
normalize these rhythms. Lidocaine, a local anesthetic,
depresses cardiac excitability, answer (b). The refractory period
of cardiac muscle is increased, thus slowing the heart down. All
of the other alternatives given would exacerbate the arrhythmia.

How well did you know this?
1
Not at all
2
3
4
5
Perfectly
258
Q

When digitalis is used in atrial fibrillation, the therapeutic objective is
to
a. Abolish cardiac decompensation
b. Inhibit vagal impulses to the heart
c. Decrease the rate of A-V conduction
d. Increase the rate of cardiac repolarization
e. Produce a decrease in the rate of atrial contraction

A

c

How well did you know this?
1
Not at all
2
3
4
5
Perfectly
259
Q

Nitroglycerin dilates the coronary arteries in angina pectoris by

a. Decreasing the heart rate reflexly
b. Increasing the metabolic work of the myocardium
c. Direct action on smooth muscle in the vessel walls
d. Increasing the effective refractory period in the atrium
e. Blocking beta-adrenergic receptors

A

c

How well did you know this?
1
Not at all
2
3
4
5
Perfectly
260
Q

Propranolol is of value in treating angina pectoris because it
a. Has a direct action on vascular smooth muscle
b. Blocks autoregulatory mechanisms in the heart
c. Inhibits oxygen metabolism in cardiac cells
d. Provides relief within seconds of an acute anginal attack
e. Prevents chronotropic responses to endogenous epinephrine
emotions and exercise

A

e

How well did you know this?
1
Not at all
2
3
4
5
Perfectly
261
Q

Administration of angiotensin results in

a. Anti-inflammatory effects
b. Antihistaminic effects
c. Increased blood pressure
d. Increased heart rate
e. A sedative effect

A

c

How well did you know this?
1
Not at all
2
3
4
5
Perfectly
262
Q
The primary antihypertensive effect of captopril (Capoten) is due to
accumulation of
a. Serotonin
b. Angiotensin I
c. Angiotensin III
d. Bradykinin metabolites
A

b

How well did you know this?
1
Not at all
2
3
4
5
Perfectly
263
Q

Administration of angiotensin results in

a. A sedative effect
b. Increased heart rate
c. Increased blood pressure
d. Antihistaminic effects
e. Anti-inflammatory effects

A

c

How well did you know this?
1
Not at all
2
3
4
5
Perfectly
264
Q

Which of the following is NOT characteristic of the thiazide diuretics?

a. Increase renal excretion of sodium and chloride
b. Increase renal excretion of potassium
c. Increase the toxicity of digitalis
d. Exacerbate existing diabetes
e. Cause hypokalemia
f. Cause hypoglycemia

A

(f) first off, how can you have an option (f)?! (a) is how diuretics
lower BP, (b) is why they can cause hypokalemia, which is
conveniently option (e), and hypokalemia can potentiate digitalis
induced arrythmias option(c). Theyy apparently can also cause
hyperglycemia, which would relate to option (d). How the heck
are you supposed to remember all of this?

How well did you know this?
1
Not at all
2
3
4
5
Perfectly
265
Q

thiazide on blood sugar

A

hyperglycemia

How well did you know this?
1
Not at all
2
3
4
5
Perfectly
266
Q

thiazide and digitalis

A

thiazide increases K excretio –> can exacerbte digitalis toxicity

How well did you know this?
1
Not at all
2
3
4
5
Perfectly
267
Q

The most useful diuretic drugs act by

a. Increasing the glomerular filtration rate
b. Decreasing the renal reabsorption of sodium
c. Decreasing the renal excretion of chloride
d. Increasing the renal reabsorption of potassium
e. Increasing the secretion of antidiuretic hormone

A

(b) people with high BP are always told to reduce salt intake,
since high sodium levels cause fluid retention which can
increase BP, so ipso facto, reducing renal reabsorption of
sodium makes BP go down

How well did you know this?
1
Not at all
2
3
4
5
Perfectly
268
Q
Which of the following drugs act by inhibiting renal reabsorption of
sodium?
a. Urea
b. Chlorothiazide
c. Theophylline
d. digitalis glycosides
e. Procainamide
A

b

How well did you know this?
1
Not at all
2
3
4
5
Perfectly
269
Q

Digoxin exerts its positive inotropic effect by
a. Activation of adenylcyclase
b. Inhibition of phosphodiesterase
c. An agonist effect of beta-receptors
d. Inhibition of Na+, K+ ATPASE leading to increased calcium
influx
e. Decreasing the amount of calcium available for excitation-
contraction coupling

A

Answer is (d)- Remember, cardiac glycosides such as digoxin
are used in the treatment of congestive heart failure, which is
the failure of the heart to function adequately as a pump and
thus maintain an adequate circulation. Cardiac glycosides are
thought to act by altering calcium ion movement, with a desired
effect of increasing the force of contraction of the myocardium
(e.g. the inotropic effect). While several of the alternatives
involve calcium, the way digoxin does it is via (d), inhibition of
Na+, K+ ATPase, resulting in an increase of calcium ion influx
into the cardiac cells, and a subsequent enhancement of the
contractile mechanism. (a) is the way epinephrine works.

How well did you know this?
1
Not at all
2
3
4
5
Perfectly
270
Q

Digitoxin is effective in the treatment of cardiac failure because it

a. Is primarily a diuretic
b. Reduces the ventricular rate
c. Decreases abnormal cardiac rhythms
d. Produces peripheral vasoconstriction
e. Has a positive cardiac inotropic action

A

e

How well did you know this?
1
Not at all
2
3
4
5
Perfectly
271
Q

The primary action of therapeutic doses of digitalis on cardiac muscle
is an increase in
a. Force of contraction
b. Ventricular excitability
c. Refractory period of the atrial muscle
d. Refractory period of the ventricular muscle
e. Rate of conduction of impulse to the muscle

A

a

How well did you know this?
1
Not at all
2
3
4
5
Perfectly
272
Q

The beneficial effects of digitalis in congestive heart failure result in
part from the fact that digitalis causes
a. A decrease in end-diastolic volume
b. A decrease in end-diastolic pressure
c. An increase in stroke volume and cardiac output
d. A decrease in central venous pressure
e. A decrease in rate of the hear where tachycardia exists

i. (a), (b) and (c)
ii. (a) and (c) only
iii. (c) and (d)
iv. (e) only
v. All of the above

A

v

How well did you know this?
1
Not at all
2
3
4
5
Perfectly
273
Q
The cardiac glycosides will increase the concentration of which ion in
an active heart muscle?
a. Sodium
b. Bromide
c. Calcium
d. Chloride
e. Potassium
A

c

How well did you know this?
1
Not at all
2
3
4
5
Perfectly
274
Q

Which of the following ions augments the inotropic effect of digitalis?

a. Sodium
b. Lithium
c. Calcium
d. Chloride
e. Magnesium

A

c

How well did you know this?
1
Not at all
2
3
4
5
Perfectly
275
Q

In the treatment of congestive heart failure, digitalis glycosides
generally decrease all of the following EXCEPT
a. Edema
b. Urine flow
c. Heart size
d. Heart rate
e. Residual diastolic volume

A

b

How well did you know this?
1
Not at all
2
3
4
5
Perfectly
276
Q

The mechanism of action of prazosin, an antihypertensive agent is to
a. Block beta-adrenergic receptors
b. Inhibit formation of angiotensin II
c. Inhibit nerve-induced release of norepinephrine
d. Stimulate central inhibitory alpha-adrenergic receptors
e. Inhibit the postsynaptic action of norepinephrine on vascular
smooth muscle

A

e

How well did you know this?
1
Not at all
2
3
4
5
Perfectly
277
Q

Which of the following owes a significant amount of its
antihypertensive effect to a central action?
a. Methyldopa
b. Metoprolol
c. Hydralazine
d. Propranolol
e. Guanethidine

A

(a) All of these drugs are used to treat hypertension, but act by
different mechanisms. (a), methyldopa, is the drug with central
action- it alters CNS control of blood pressure by acting on
cardioregulatory and vasomotor systems of the brain by
stimulating alpha2 receptors in the brain stem. Clonidine is the
usual drug that is involved in this particular question. (b)
metropolol is a selectively blocks beta-1 receptors in the heart to
reduce cardiac output. (c) hydralazine has a direct action on
vascular smooth muscle to reduce hypertension via
vasodilation. (d) propranolol blocks beta receptors in the heart,
while (e) guanethidine prevents the release and causes
depletion of catecholamines taken up into storage vesicles and

is released like a false transmitter. It does not cross the blood-
brain barrier.

How well did you know this?
1
Not at all
2
3
4
5
Perfectly
278
Q
Which of the following drugs is thought to reduce arterial blood
pressure by activating alpha receptors in the vasomotor center of the
medulla?
a. Prazosin
b. Clonidine
c. Propranolol
d. Guanethidine
e. Chlorothiazide
A

b

How well did you know this?
1
Not at all
2
3
4
5
Perfectly
279
Q

Propranolol (Inderal) can be useful in the treatment of hypertension
because it blocks
a. Alpha-1 adrenergic receptors
b. Sodium reabsorption in the kidney
c. The release of renin from juxtaglomerular cells
d. The release of norepinephrine from nerve terminals
e. The reflex tachycardia seen with the use of other
antihypertensives
i. (a) and (b)
ii. (a) and (d)
iii. (b), (c) and (d)
iv. (c), (d) and (e)
v. (c) and (e) only

A

(v) Answer is (v)- You should immediately recognize that
propranolol is the prototypic beta-adrenergic receptor blocker,
thus any answer with alternative a (i and ii) is wrong. Similarly, d
is wrong as well-propranolol is a competitive beta- receptor
blocker- it has no effect on NE release. Another drug used for
hypertension, Clonidine, acts via this mechanism by stimulating
alpha-2 autoreceptors. Thus ii, iii, and iv are wrong. This leaves
(v) as the only possible right answer. Indeed, aside from
blocking beta-1 receptors, blocking of renin release is thought to
be the other mechanism whereby beta-blockers alter
hypertension.

How well did you know this?
1
Not at all
2
3
4
5
Perfectly
280
Q

One of the proposed mechanisms of the antihypertensive effect of
beta-adrenergic receptor blocking agents is
a. Sedation
b. A diuretic effect
c. An antirenin effect
d. A vagal blocking effect
e. An increase in cardiac output

A

c

How well did you know this?
1
Not at all
2
3
4
5
Perfectly
281
Q
Selective beta-1 adrenergic agonists will produce which of the
following effects?
a. Glycogenolysis
b. Increased cardiac output
c. Decreased diastolic pressure
d. Decreased peripheral resistance
e. Relaxation of bronchial smooth muscle
A

b

How well did you know this?
1
Not at all
2
3
4
5
Perfectly
282
Q

Ototoxicity with deafness may encountered occasionally in patients
taking which of the following diuretic agents?
a. Osmotic
b. Thiazide
c. Mercurial
d. High-ceiling

A

d

How well did you know this?
1
Not at all
2
3
4
5
Perfectly
283
Q

Symptoms of digitalis toxicity include all of the following EXCEPT

a. Extrasystoles
b. Nausea and vomiting
c. Yellow-green vision
d. A-V conduction block
e. Decreased P-R interval

A

e

How well did you know this?
1
Not at all
2
3
4
5
Perfectly
284
Q
Administration of which of the following drugs increases the likelihood
of a toxic response to digitalis?
a. Diazepam
b. Lidocaine
c. Spironolactone
d. Chlorothiazide
e. Acetylsalicylic acid
A

(d) Chlorthiazide is a diuretic which causes potassium loss or
hypokalemia. This results in greater penetration of digitalis into
the myocardium, and thus potential toxicity.

How well did you know this?
1
Not at all
2
3
4
5
Perfectly
285
Q

aspirin vs acetaminophen

A

acetaminphen - no anti-inflammatory activity
no GI upset
is hepatotoxic

How well did you know this?
1
Not at all
2
3
4
5
Perfectly
286
Q

The therapeutic effect of the salicylates is explained on the basis of the
ability of the drug to
a. Activate autonomic reflexes
b. Uncouple oxidative phosphorylation
c. Inhibit the synthesis of prostaglandins
d. Competitively antagonize prostaglandins at the receptor site

A

c

How well did you know this?
1
Not at all
2
3
4
5
Perfectly
287
Q

The mechanism of the antipyretic action of salicylates probably results
from
a. Inhibition of prostaglandin synthesis in the CNS affecting
hypothalamic temperature regulation
b. Inhibition of bradykinin in the periphery leading to sweating
c. Depression of oxidative enzymes leading to decreased heat
production
d. Suppression of cholinergic mediators in the hypothalamus
e. Stimulation of norepinephrine in the hypothalamus

A

a

How well did you know this?
1
Not at all
2
3
4
5
Perfectly
288
Q

The antipyretic action of salicylates is explained in part by
a. Analgesia leading to sedation
b. Increased blood flow through the hypothalamus
c. Cutaneous vasodilation leading to increased heat loss
d. Depression of oxidative processes leading to decreased heat
production

A

c

How well did you know this?
1
Not at all
2
3
4
5
Perfectly
289
Q

The locus of action of aspirin’s central antipyretic effect is the

a. Brain stem
b. Hypothalamus
c. Basal ganglia
d. Limbic system
e. Cerebral cortex

A

b

How well did you know this?
1
Not at all
2
3
4
5
Perfectly
290
Q

t regulation center in CNS

A

hypothalamus

How well did you know this?
1
Not at all
2
3
4
5
Perfectly
291
Q

A patient who has been taking large quantities of aspirin might show
increased postoperative bleeding because aspirin inhibits
a. Synthesis of thromboxane A2 and prevents platelet
aggregation
b. Synthesis of prostacyclin and prevents platelet aggregation
c. Synthesis of prostaglandin and prevents production of blood
platelets
d. Thrombin and prevents formation of the fibrin network
e. G.I. absorption of vitamin K and prevents synthesis of blood
clotting factors

A

(a) The first fact you must remember is that aspirin prevents
platelet aggregation- this limits your choices to (a) and (b). They
hope to confuse you by using prostacylin, but of course you
know that this is wrong immediately, the right word is
prostaglandin, as in (c), but you have already eliminated that
choice because it doesn’t mention prevention of platelet
aggregation. Thus, even if you didn’t remember that
thromboxane A2 induces platelet aggregation, and aspirin
blocks this action, you could get the answer by elimination. (d) is
how heparin works, while (e) is how coumarin works.

How well did you know this?
1
Not at all
2
3
4
5
Perfectly
292
Q

Anti-inflammatory agents, such as aspirin, interfere with hemostasis by

a. Activating antithrombin
b. Preventing vasoconstriction
c. Inhibiting thrombin generation
d. Inhibiting platelet aggregation
e. Inhibiting polymerization of fibrin

A

d

How well did you know this?
1
Not at all
2
3
4
5
Perfectly
293
Q

Which of the following anti-inflammatory agents does NOT act
primarily by inhibiting activity of prostaglandin synthetase?
a. Diflunisal
b. Ibuprofen
c. Triamcinolone
d. Oxyphenbutazone
e. Acetylsalicylic acid

A

(c) triamcinolone is a corticosteroid. Corticosteroids inhibit
phospholipase A2, the enzymatic step that precedes
prostaglandin synthetase. Diflunisal is a salicylate analgesic, like
aspirin.

How well did you know this?
1
Not at all
2
3
4
5
Perfectly
294
Q

A nonsteroidal, anti-inflammatory agent that appears to produce fewer
gastrointestinal disturbances than high does of aspirin is
a. Ibuprofen
b. Probenecid
c. Pentazocine
d. Acetaminophen
e. Phenylbutazone

A

(a) you might be tempted to answer acetaminophen, because it

doesn’t cause GI upset, but remember it is also not anti-
inflammatory. The answer is ibuprofen.

How well did you know this?
1
Not at all
2
3
4
5
Perfectly
295
Q

Prolonged use of which of the following drugs does NOT cause a
predisposition to gastric irritation and bleeding?
a. Phenytoin
b. Ibuprofen
c. Indomethacin
d. Phenylbutazone
e. Acetylsalicylic acid

A

(a) This is a straight drug identification question. Answers 2-5
are all non-steroidal antiinflammatory drugs which cause gastric
irritation and bleeding due to their effects on prostaglandin
synthesis in the mucosal wall of the gut. # , phenytoin, is an
anti-convulsant-its major side effect that often appears as a
question on boards is the production of gingival hyperplasia.

How well did you know this?
1
Not at all
2
3
4
5
Perfectly
296
Q
Each of the following agents has been associated with gastric
irritation EXCEPT
a. Aspirin
b. Alcohol
c. Ibuprofen
d. Indomethacin
e. Acetaminophen
A

(e) note the difference in this question and #11 and 12.
Ibuprofen was previously the answer to “shows reduced GI
irritation”, but it does cause some, which you have to remember
to answer #12 and this question. So aspirin and ibuprofen are
out. Indomethacin is a very strong NSAID that causes lots of GI
irritation, so much that use is limited in humans, so it is out.
What about alocohol vs. acetaminophen. Well, you should really
know that acetaminophen is usually the answer to these types
of analgesics questions, but if you didn’t know that, perhaps may know that alcohol also causes GI irritation, so it is out.

How well did you know this?
1
Not at all
2
3
4
5
Perfectly
297
Q

Which of the following is NOT produced by excessive doses of acetylsalicylic acid?

a. Delirium
b. Tinnitus
c. Hypothermia
d. Hyperventilation
e. Metabolic acidosis

A

(c) it only lowers your temperature if you have a fever,, taking
aspirin does not have any effect on body temperature in the
non-feverish patient, but high doses can cause all the other
effects listed.

How well did you know this?
1
Not at all
2
3
4
5
Perfectly
298
Q
All of the following are pharmacologic and toxicologic properties of
aspirin EXCEPT
a. Tinnitus
b. Analgesia
c. Salicylism
d. Antipyresis
e. Suppression of the immune response
A

e

How well did you know this?
1
Not at all
2
3
4
5
Perfectly
299
Q

Therapeutic effects of aspirin include

a. Analgesia
b. Tranquilization
c. Pyretic action
d. Anti-inflammatory action
e. Antirheumatic action
i. (a), (b) and (c)
ii. (a), (c) and (d)
iii. (a), (d) and (e)
iv. (b), (c) and (d)
v. (b), (d) and (e)

A

iii

How well did you know this?
1
Not at all
2
3
4
5
Perfectly
300
Q
All of the following are pharmacologic or toxicologic properties of
acetylsalicylic acid EXCEPT
a. Tinnitus
b. Analgesia
c. Antipyresis
d. Methemoglobinemia
e. Inhibition of prostaglandin synthesis
A

d

How well did you know this?
1
Not at all
2
3
4
5
Perfectly
301
Q

All of the following are possible effects of aspirin EXCEPT

a. Reduction of fever
b. Shortening of bleeding time
c. Suppression of inflammatory response
d. Bleeding from the gastronintestinal tract
e. Increase in the renal excretion of uric acid at high doses

A

b

How well did you know this?
1
Not at all
2
3
4
5
Perfectly
302
Q

Of the following, aspirin does NOT cause

a. Occult bleeding
b. Nausea and vomiting
c. Acid-base disturbance
d. Suppression of the cough reflex
e. Decreased tubular reabsorption of uric acid

A

d
Answer is (d)- (a) & (b) are the major side effects of aspirin (resulting from the inhibition of prostaglandin synthesis) for the
majority of people, and one reason for the popularity of aspirin
alternatives such as acetaminophen and ibuprofen, which
produce these effects to a lesser extent. # 3 & 5 may also be
seen following larger doses of aspirin. (d) is not seen with
aspirin, but is a major therapeutic use of narcotic opiates such
as codeine. I guess they are hoping that you will get the effects
of codeine and aspirin mixed up, since the two are often
compared and contrasted as moderate pain relievers.

How well did you know this?
1
Not at all
2
3
4
5
Perfectly
303
Q

Which of the following is NOT true about acetaminophen?
a. Is a non-prescription drug
b. Is cross-allergenic with aspirin
c. Possesses both analgesic and antipyretic effects
d. May induce methemoglobinemia at high doses
e. May be the pharmacologically active form of acetophenetidin
(phenacetin)

A

(b) because it ain’t a salicylate, but the other statements are true

How well did you know this?
1
Not at all
2
3
4
5
Perfectly
304
Q

Which of the following is NOT true regarding acetaminophen?

a. It has antipyretic properties
b. It may induce methemoglobinemia
c. It can be combined with codeine
d. It has anti-inflammatory properties
e. It is not cross-allergenic with aspirin

A

d

How well did you know this?
1
Not at all
2
3
4
5
Perfectly
305
Q
The most prominent acute toxic effect associated with
acetaminophen use is
a. Hemorrhage
b. Renal necrosis
c. Hepatic necrosis
d. Gastric ulceration
e. Respiratory alkalosis
A

(c) Remember, acetaminophen (tylenol) is an aspirin
alternative. Alternatives 1, 4, 5 are side effects of aspirin-type
drugs. The popularity of acetaminophen as an aspirin alternative
is because the incident of such effects with this drug is very low.
However, because acetaminophen can undergo
biotransformation to a toxic intermediate, hepatic and renal
necrosis have been reported, especially after very high doses.
(c), hepatic necrosis is the most prominent, especially when
combined with alcohol consumption, since the alcohol induces
the liver enzymes which make the hepatotoxic metabolites of
acetaminophen

How well did you know this?
1
Not at all
2
3
4
5
Perfectly
306
Q

Which of the following anti-inflammatory agents does NOT act
primarily by inhibiting the activity of cyclooxygenase?
a. Ibuprofen
b. Diflunisal
c. Prednisone
d. Indomethacin
e. Phenylbutazone

A

Answer is (c)- (a), 2, 4, and 5 are NSAIDS that reduce
inflammation by reducing prostaglandin synthesis by blocking
the activity of cyclooxygenase. Prednisone is a corticosteroid. Corticosteroids are potent nonspecific inhibitors of the
inflammatory process, acting at a variety of point throughout the
inflammatory process. Although they do reduce prostaglandin
production as well, they do this by a mechanism other then
blocking cyclooxygenase, probably by inhibiting the release of
the fatty acid substrate for prostaglandin synthesis.

How well did you know this?
1
Not at all
2
3
4
5
Perfectly
307
Q
24. Which of the following is the most appropriate drug to use to lower
fever in a child under 12?
a. Aspirin
b. Ibuprofen
c. Acetaminophen
d. Salicylate
e. Diflunisal
A

(c) acetaminophen is the best choice. Aspirin is contraindicated due to
the potential for causing Reye’s syndrome. Ibuprofen is approved,
but usually not the #1 choice. The others would be inappropriate as
well.

How well did you know this?
1
Not at all
2
3
4
5
Perfectly
308
Q
  1. Which analgesic from the following list has the longest half-life?
    a. Acetaminophen
    b. Aspirin
    c. Diflunisal (Dolobid)
    d. Ibuprofen
A

(c) Diflunisal can be taken twice a day, the others three-four times a
day is required.

How well did you know this?
1
Not at all
2
3
4
5
Perfectly
309
Q
Occurrence of which of the following is LEAST characteristic of
narcotic ingestion?
a. Vomiting
b. Diarrhea
c. Urinary retention
d. Bronchiolar constriction
e. Increase in intracranial pressure
A

(b) Again, the key word is least. Narcotics, in the form of
paregoric (tincture of opium), and Lomotil (loperamide) are over
the counter oral preparations for the treatment of diarrhea.
Opiates act on receptors in the gut to produce constipation.
Thus (b) is obviously wrong. All of the other answers are side
effects of opiate administration.

How well did you know this?
1
Not at all
2
3
4
5
Perfectly
310
Q
Therapeutic doses of morphine administered intramuscularly may
produce
a. Constipation
b. Euphoria
c. Dysphoria
d. Mental clouding
e. Decreased response to pain
i. (a) and (b) only
ii. (a), (b) and (d)
iii. (a), (d) and (e)
iv. (c), (d) and (e)
v. All of the above
A

v

How well did you know this?
1
Not at all
2
3
4
5
Perfectly
311
Q

Which of the following are pharmacologic effects of morphine?

a. Respiratory depression
b. Euphoria
c. Sedation
d. Constipation
e. Dysphoria
i. (a), (b) and (c)
ii. (a), (b) and (d)
iii. (a) and (e)
iv. (c), (d) and (e)
v. All of the above

A

v

How well did you know this?
1
Not at all
2
3
4
5
Perfectly
312
Q

Which of the following drugs acts to suppress the cough reflex?

a. ASA
b. Codeine
c. Meperidine
d. Acetaminophen
e. Phenyibutazone

A

(b) the only drugs that do this are opioids, and codeine and
merperidine are the two opioids on the list. Of the two, codeine
is much better at this than meperidine.

How well did you know this?
1
Not at all
2
3
4
5
Perfectly
313
Q

Morphine binds to which site to produce analgesia?

a. By binding to specific receptors in the CNS
b. By decreasing the influx of sodium
c. By decreasing the synthesis of prostaglandins
d. By decreasing nerve activatin at the site of injury

A

(a) They might reword the question in a way that asks you to

remember that the specific receptors are the mu receptors.

How well did you know this?
1
Not at all
2
3
4
5
Perfectly
314
Q

Morphine causes vomiting by

a. A direct irritant action on the gastric mucosa
b. Stimulation of the nodose ganglion of the vagus nerve
c. Stimulation of the medullary chemoreceptor trigger zone
d. Direct stimulation of the gastrointestinal musculature

A

c

How well did you know this?
1
Not at all
2
3
4
5
Perfectly
315
Q

The decrease in ventilation caused by morphine, meperidine and
some of the related opioids depends chiefly upon
a. Depression of cortical activity
b. Peripheral blockade of chemoreceptor impulses
c. An increase in carbon dioxide concentration in the blood
d. Blockade of afferent autonomic impulses from the lungs
e. Loss of sensitivity of the medullary respiratory center to carbon
dioxide

A

e

How well did you know this?
1
Not at all
2
3
4
5
Perfectly
316
Q

Small doses of barbiturates and morphine depress respiration
primarily by
a. A parasympathominetic action
b. Inhibiting the Herine-Bueuer reflex
c. Rendering the aortic chemoreceptor system insensitive to O2
d. Rendering the respiratory centerin the brain stem less sensitive
to changes in CO2
e. A specific effect at myoneural junctions of phrenic and
intercostal nerves

A

d

How well did you know this?
1
Not at all
2
3
4
5
Perfectly
317
Q

Which of the following are pathognomonic symptoms of narcotic
overdose?
a. Miosis, coma and depressed respiration
b. Mydriasis, coma and smooth muscle spasms
c. Mydriasis, coma and depressed respiration
d. Miosis, convulsions and depressed respiration
e. Mydriasis, convulsions and depressed respiration

A

a

How well did you know this?
1
Not at all
2
3
4
5
Perfectly
318
Q

The cause of death with opioid intoxication is

a. Oxygen apnea
b. Cardiac arrest
c. Terminal convulsions
d. Circulatory collapse
e. Respiratory depression

A

e
opioids decrease the response of respiratory centers in the
brainstem to the carbon dioxide tension of the blood, and also
depresses pontine and medullary centers regulating respiratory
frequency. Opioids do not cause oxygen apnea, ((a)), they can
be convulsive, but not terminally so ((c)), they are stabilizing on
the heart and some are actually used in open-heart surgery
((b)), and they do not cause circulatory collapse ((d)).

How well did you know this?
1
Not at all
2
3
4
5
Perfectly
319
Q
Which of the following is an opioid that has both agonistic and
antagonistic activities?
a. Codeine
b. Methadone
c. Naloxone
d. Meperidine
e. Pentazocine
A

(e) This is an example of the type of question where the drug
class is given. You are asked to not only identify a drug from the
list as being from this class, but additionally that it has the
properties that are given in the question that distinguish it from
the other drugs of the class that are listed as alternatives. In this
example, there is only one drug which meets this criterion. All
are drugs which act via opiate receptors, but 3 are agonists ((a),
(b), (d)), 1 is an antagonist only ((c)). (e) pentazocine is the only
drug which has both types of action, and is the one drug left by
the process of elimination.

How well did you know this?
1
Not at all
2
3
4
5
Perfectly
320
Q

A heroin-dependent patient should NOT be given nalbuphine (Nubain
) for pain because
a. It has no analgesic properties
b. It may produce respiratory depression
c. As a mixed agonist-antagonist, it can elicit withdrawal
symptoms
d. The high abuse potential of nalbuphine may add to the patient’s
problems

A

c

How well did you know this?
1
Not at all
2
3
4
5
Perfectly
321
Q
A patient while not currently taking drugs has a history (6 months
ago) of narcotic dependency. Which of the following analgesics
should be avoided in this patient?
a. Aspirin
b. Pentazcine
c. propoxyphene
d. Indomethacin
e. Acetaminophen
f. None of the above
A

b

How well did you know this?
1
Not at all
2
3
4
5
Perfectly
322
Q

Which of the following statements does NOT characterize
pentazocine?
a. It is equianalgesic with codeine
b. It is a partial opioid antagonist
c. Its abuse potential is less than that of heroin
d. It may induce dysphoria and mental aberrations
e. It is effective only on parenteral administration

A

(e) lot of memorization required here for a drug that isn’t
used that much. I guess it was big news when these
questions were written many years ago and they seemed
hopeful, since statement (c) was true and was
therapeutically an advantage, but it soon became apparent
that (d) was also true

How well did you know this?
1
Not at all
2
3
4
5
Perfectly
323
Q

The antagonist of choice in the treatment of opioid overdosage is

a. Naloxone
b. Nalorphine
c. Pentazocine
d. Levallorphan
e. Propoxyphene

A

(a) nalorphine and pentazocine are mixed agonist-antagonists,
levallorphan is an opioid agonist, as is propoxyphene

How well did you know this?
1
Not at all
2
3
4
5
Perfectly
324
Q

Which of the following is a complete antagonist of the opioid receptor
and the agent of choice in the treatment of narcotic overdose?
a. Naloxone
b. Nalorphine
c. Cyclazocine
d. Levallorphan
e. None of the above

A

a

How well did you know this?
1
Not at all
2
3
4
5
Perfectly
325
Q

Methadone is used in detoxification (drug withdrawal) of patients
physically dependent on morphine because methadone
a. Precipitates withdrawal reactions
b. Antagonizes the depressant actions of morphine
c. Will not in itself produce physical dependence
d. Withdrawal reactions are less intense and stressful than those of
morphine

A

(d) This is an example of the kind of question that requires that
you have memorized a fact about a particular drug, in this case
the fact is (d). Methadone you will remember is not an
antagonist like naloxone- it is a full agonist with analgesic
properties, just like morphine. When taken orally it is not
euphoric in addicts, but acts just like morphine to produce
tolerance and physical dependence. Withdrawal is less severe
than with morphine because methadone has a much longer half
life. Facts 1, 2, and 3 would be met by an antagonist such as
naloxone, or perhaps even a mixed agonist-antagonist such as
pentazocine.

How well did you know this?
1
Not at all
2
3
4
5
Perfectly
326
Q
Which of the following drugs is currently widely used in treating
opioid-dependent individuals?
a. Codeine
b. Methadone
c. Alphaprodine
d. Pentazocine
e. Meperidine
A

b

How well did you know this?
1
Not at all
2
3
4
5
Perfectly
327
Q
  1. Meperidine (Demerol) is
    a. An antidepressant
    b. An opioid analgesic
    c. A sedative
    d. A long-acting local anesthetic
    e. An antipsychotic
A

b

How well did you know this?
1
Not at all
2
3
4
5
Perfectly
328
Q

sympathetic receptors

A

ganglionic ACh nicotine

end-organ ACh muscarinic

How well did you know this?
1
Not at all
2
3
4
5
Perfectly
329
Q

parasympathetic receptors

A

ganglionic ACh nicotine

end-organ usually adrenergic: except sweat glands ACh muscarinic

How well did you know this?
1
Not at all
2
3
4
5
Perfectly
330
Q

motor neuron receptors

A

muscular ACh nicotine

How well did you know this?
1
Not at all
2
3
4
5
Perfectly
331
Q

Atropine and propantheline exert their effects on peripheral structures
by
a. preventing release of acetylcholine
b. preventing synthesis of acetylcholine
c. enhancing destruction of acetylcholine
d. competeing with acetylcholine for receptor sites
e. producing physiologic effects opposite to those of acetylcholine

A

(d) - (a) is wrong- botulinum toxin does this. (b) is wrong-
hemicholinium works this way. (c) is wrong- ACh is broken

down almost instantaneously, so it is almost impossible to
enhance its destruction. (e) is wrong-these drugs don’t have
any actions of their own, they just prevent ACh effects by
blocking receptors: atropine and propantheline are
postganglionic muscarinic receptor blockers-thus the answer is
(d).

How well did you know this?
1
Not at all
2
3
4
5
Perfectly
332
Q

Neostigmine produces its effect by
a. depressing acetylcholinesterase release
b. inhibiting acetylcholinesterase activity
c. increasing the rate of acetylcholine synthesis
d. acting like acetylcholine at ganglionic sites
e. increasing the amount of acetylcholine released from nerve
terminals

A

(b) – neostigmine is a cholinesterase inhibitor like physostigmine. These differ from the insecticides and nerve
gases listed below in that they are reversible and can be used
clinically; the latter are irreversible.

How well did you know this?
1
Not at all
2
3
4
5
Perfectly
333
Q
Organophosphate insecticides and nerve gases inhibit the action of
which of the following enzymes?
a. adenylate cyclase
b. monoamine oxidase
c. phosphodiesterase
d. acetylcholinesterase
e. carbonic anhydrase
A

d

How well did you know this?
1
Not at all
2
3
4
5
Perfectly
334
Q
Drugs which are additive with or potentiate the effects of acetylcholine
include
(a) methacholine
(b) scopolamine
(c) pralidoxime
(d) neostigmine
(e) pilocarpine
1. (a), (b), and (c)
2. (a), (c), and (d)
3. (a), (d) and (e)
4. (b), (d), and (e)
5. (c), (d) and (e)
A

(3) drugs which potentiate cholinergic stimulation can do so by
being either direct acting cholinergic agonists, acting on the
cholinergic receptor, or by indirectly increasing the duration of
action Ach by preventing its enzymatic degradation.
Methacholine and pilocarpine are direct-acting cholinergic
agonists, whereas neostigmine acts indirectly. Scopolamine is a
muscarinic antagonist like atropine, and will reduce or block
cholinergic action via direct receptor antagonism. Pralidoxime is
a chemical antidote used to regenerate AchE after nerve gas or
insecticide exposure.

How well did you know this?
1
Not at all
2
3
4
5
Perfectly
335
Q

Which of the following drugs is best to administer after poisoning by an
organophosphate cholinesterase inhibitor?
a. atropine
b. phenytoin
c. pralidoxime
d. propantheline
e. phenobarbital

A

c

How well did you know this?
1
Not at all
2
3
4
5
Perfectly
336
Q

Which of the following acts by antagonizing cholinesterase?

a. atropine
b. muscarine
c. neostigmine
d. pilocarpine
e. acetylcholine

A

c

How well did you know this?
1
Not at all
2
3
4
5
Perfectly
337
Q

When neostigmine is administered before acetylcholine, the action of
acetylcholine will be
a. blocked
b. enhanced and prolonged
c. less intense and of shorter duration
d. none of the above. The action of acetylcholine is not affected by
neostigmine

A

(b) the action will be prolonged because neostigmine prevents
its breakdown by AchE! Its action would be blocked by atropine
or scopolamine

How well did you know this?
1
Not at all
2
3
4
5
Perfectly
338
Q

Neostigmine can stimulate denervated skeletal muscle because it

a. is a congener of acetylcholine
b. is a competitive blocking agent
c. has no effect on acetylcholinesterase
d. is more potent than diisopropylfluorophosphate
e. is capable of acting directly on the end-plate

A

(e)a strange and amazing truth! Denerevated means the
skeletal muscle is not receiving neural input, and thus
stimulating a cholinergic neuron to release Ach which would
then stimulate the NMJ can’t happen. But if you inject a drug
which can stimulate the nicotinic receptors directly then you can
see an effect on the muscle. Neostigmine is one of those
anticholinesterases that can act like Ach at nicotinic receptors,
in addition to prolonging the action of Ach itself by blocking the
acetylcholinesterase that is trying to break the Ach down.

How well did you know this?
1
Not at all
2
3
4
5
Perfectly
339
Q

Which of the following is used to prevent laryngospasm?

a. atropine
b. epinephrine
c. diazepam (Valium)
d. neostigmine (Prostigmine)
e. succinylcholine (Anectine)

A

(e) What is needed is a skeletal muscle relaxant. This requires a
drug that acts at the neuromuscular junction. Of those listed,
only succinylcholine (e) is in this category. (a) atropine is a
cholinergic (Muscarinic) receptor blocker, (b) epinephrine is an
adrenergic agonist, and (c) diazepam is a benzodiazepine, and
(d) is an anticholinesterase.

How well did you know this?
1
Not at all
2
3
4
5
Perfectly
340
Q

In treating xerostomia, which of the following might be prescribed?

a. atropine
b. ephedrine
c. neostigmine
d. scopolamine
e. mecamylamine

A

(c) salivation is typically considered to be a cholinergic
response. Xerostomia is too little saliva and thus one could use
a cholinergic agonist to stimulate more saliva secretion
(assuming there is functional salivary gland tissue, which may
not be the case in patients that have been subject to radiation
therapy!) From the list, only neostigmine would produce a
cholinergic effect, since it is an indirect acting cholinergic
agonist. Of the others, atropine and scopolamine are cholinergic
antagonists, and are actually used to reduce salivation,
mecamylamine is a ganglionic blocker (not the action we desire,
too non-specific, and a blocker at that!). Ephedrine is a mixed
acting adrenergic agonist.

How well did you know this?
1
Not at all
2
3
4
5
Perfectly
341
Q
Which of the following drugs is most likely to dry secretions in the oral
cavity?
a. diazepam
b. promethazine
c. physostigmine
d. propantheline
e. diphenhydramine
A

d

How well did you know this?
1
Not at all
2
3
4
5
Perfectly
342
Q
The most useful drug to induce salivation is one which has properties
that are
a. adrenergic
b. cholinergic
c. ganglionic blocking
d. adrenergic blocking
e. cholinergic blocking
A

b

How well did you know this?
1
Not at all
2
3
4
5
Perfectly
343
Q
Drugs that are commonly used in the control of excessive salivation
include
(a) meprobamate
(b) atropine
(c) methantheline
(d) codeine
(e) chlorpromazine
1. (a) and (b) only
2. (a), (b), and (c)
3. (b) and (c) only
4. (b), (c), and (d)
5. (d) and (e)
A

(3) you are probably thinking, who the hell ever heard of
meprobamate and methantheline, or for that matter,
chlorpromazine! Well, these are drugs that may have been
useful in the olden days, but would be probably replace in this
type of questions by more modern equivalents. But of course
you should figure out that atropine (b), being the prototype
anticholinergic drug has to be one of the answers. So option 5
has to bee incorrect, since it does not include atropine. Now
only the most corrupt dentist would prescribe codeine to reduce
salivation, so #4 should also be incorrect – that leaves 1, 2, or 3.
So see, you didn’t even have to recognize that chlorpromazine
is an antipsychotic drug. So what is meprobamate – if we can
eliminate that one then we are down to only option 3 as a
possible answer. Meprobamate happens to be an antianxiety,
skeletal muscle relaxant drug sometimes used by dentists to
treat muscle spasms associated with TMD – also has use for
external sphincter spasticity – imagine! But it doesn’t seem to
have anticholinergic activity that is significant enough to cause
significant reduction of saliva. Methantheline, in contrast, is
Banthine, a synthetic version of atropine! So option 3, atropine
and methantheline are the drugs for this purpose.

How well did you know this?
1
Not at all
2
3
4
5
Perfectly
344
Q

Administration of ganglionic blocking agents will result in

a. miosis
b. diarrhea
c. copious salivation
d. orthostatic hypotension
e. enhanced activity of the parasympathetic nervous system

A

(d) a ganglionic blocker, since it acts by preventing cACh from
stimulating nicotinic receptors at the ganglia level will have both
anticholinergic and antiadrenergic effects. Options a, b, c, and e
are symptoms of cholinergic stimulation, and thus can’t be right.
Option (d), the remaining answer, is an antiadrenergic effect,
arising from decreases in sympathetic tone to the vasculature

How well did you know this?
1
Not at all
2
3
4
5
Perfectly
345
Q

Tachycardia in a patient administered with atropine or scopolamine
results from
a. release of adrenal catecholamines
b. blockade of vagus nerve activity
c. blockade of the nicotinic cholinergic receptor
d. stimulation of the alpha adrenergic receptor
e. stimulation of the beta adrenergic receptor

A

(b) - Atropine and scopolamine are muscarinic cholinergic
receptor blockers. Just knowing that eliminates all the
alternatives except (b). But you should also remember that
heart rate is kept under tight reflexive control: any sudden
increase in HR usually stimulates baroreceptors to send a
signal to the vagus nerve to stimulate the heart to slow it back
down. This reflex is cholinergically mediated, and will be blocked
by cholinergic blockers such as atropine. Even when given in
the absence of higher than normal heart rate, atropine will block
the normal cholinergic control over the heart, leaving the
sympathetic system in charge with a resulting tachycardia.

How well did you know this?
1
Not at all
2
3
4
5
Perfectly
346
Q

All of the following are possible effects of cholinomimetic drugs
except
a. mydriasis
b. bradycardia
c. increased peristalsis
d. stimulation of sweat glands
e. increased secretion by bronchial glands

A

(a) - The first thing you have to know is that a cholinomimetic drug is one that mimics the action of acetylcholine, the
endogenous neurotransmitter in the parasympathetic or
cholinergic nervous system. The acronym for remembering the
effects of cholinergic stimulation is SLUD, or increased
salivation, lacrimation, defecation, and urination. The heart is the
exception in that activity or heart rate is decreased
(bradycardia)- thus since the question asks for an effect which
does not occur with cholinergic stimulation, that leaves (a) as
the only possibility. Miosis, not mydriasis, occurs with
cholinergic stimulation.

How well did you know this?
1
Not at all
2
3
4
5
Perfectly
347
Q

A paralyzing dose of succinylcholine initially elicits

a. CNS stimulation
b. CNS depression
c. decreased salivation
d. muscle fasiculation
e. extrapyramidal reactions

A

(d) because succinylcholine (SUX) is an agonist at nicotinic
receptors, so the initial response is muscle stimulation. But the
NMJ rapidly depolarizes due to the inability of the plasma
cholinesterase to break down the SUX, which isjust two
molecules of acetylcholine fused together – the other
information has no relevance unless you’re just stuck thinking
“gee, I know that SUX has something to do with autonomics but
not sure exactly what!”

How well did you know this?
1
Not at all
2
3
4
5
Perfectly
348
Q
Based on its known mechanism and sites of action, scopolamine
should theoretically be useful in
(a) treatment of peptic ulcer
(b) providing euphoria and amnesia prior to surgery
(c.) relieving bronchoconstriction
(d) relieving some of the symptoms of Parkinson disease
(e) visualization of the retina
1.. (a), (b), (d) and (e)
2.. (a), (b), and (e) only
3. (a),and (c)
4. (b) and (e) only
5. All of the above
A

5

How well did you know this?
1
Not at all
2
3
4
5
Perfectly
349
Q
Symptoms of poisoning by an organophosphate insecticide include all
of the following except:
a. skeletal muscle fasiculation
b. excessive salivation
c. bronchoconstriction
d. hot, dry skin
e. diarrhea
A

(d) organophosphates kill you from too much cholinergic
stimulation (SLUD). Option (a) is from nicotinic receptor
stimulation, (b) and (c) and (e) are also cholinergic stimulation.
Option (d) is an atropine, anticholinergic type reaction and thus
doesn’t fit the pattern of responses given.

How well did you know this?
1
Not at all
2
3
4
5
Perfectly
350
Q
All of the following symptoms are associated with neostigmine
poisoning except
a. diarrhea
b. salivation
c. convulsions
d. bonchiolar constriction
e. skeletal muscle paralysis
A

(e) basically the same question as the preceding question, they
just changed organophosphate insecticides to neostigmine. The
difference is that neostigmine is a reversible anticholinesterase,
whereas insecticides are irreversible. But again, the question
just basically wants you to recognize two things – that
neostigmine is an indirect acting cholinergic drug and then know
what the symptoms of cholinergic stimulation are. But even if
you didn’t know those facts you might be able to get if you
remember that neostigmine is a drug that is used to reverse the
skeletal muscle paralysis produced by drugs of the curare class
– the non-depolarizing neuromuscular junction blockers

How well did you know this?
1
Not at all
2
3
4
5
Perfectly
351
Q

Symptoms of atropine poisoning in man include

(a) decreased intraocular pressure
(b) burning dry mouth
(c) nausea, vomiting and diarrhea
(d) hyperthermia
(e) orthostatic hypotension
1. (a) and (c)
2. (b) and (d)
3. (b), (d), and (e)
4. (d) and (e)
5. All of the above

A

(2) glandular secretions are generally under cholinergic control,
so sweating and salivation are greatly reduced by the
anticholinergic drug atropine. So if you can’t salivate or sweat,
you very possible will show what kind of symptoms? How about
a burning dry mouth and hyperthemia? Nausea and vomiting
are cholinergic overdose, as is orthostatic hypotension.

How well did you know this?
1
Not at all
2
3
4
5
Perfectly
352
Q

The most likely signs or symptoms of overdosage with atropine are

a. CNS excitation and tachycardia
b. intestinal cramps and diarrhea
c. skin rash and cutaneous itching
d. ptyalism and increased sweating
e. constriction of the pupils and blurring of vision

A

(a) just when you thought you could get by with the anti-SLUD
strategy, they then expect you to remember that atropine
overdose causes CNS excitation and tachycardia? Actually,
that is one of the interesting diffs between atropine and
scopolamine, and the reason that scopolamine is used for
sedation, while atropine isn’t (b) and (e) are cholinergic
stimulation I think, (c) is histamine produced, while I haven’t got
a clue what “ptyalism” is – do you? But I do know that if atropine
causes dry hot skin because it prevents sweating then (d) can’t
be right!

How well did you know this?
1
Not at all
2
3
4
5
Perfectly
353
Q

Disorientation, confusion and hallucinations resulting from an
overdose of scopolamine are most efficaciuosly treated by
administering
a. atropine
b. levodopa
c. acetylcholine
d. physostigmine

A

(d) so ya gotta know scopolamine is anticholinergic, so ya need
a cholinergic agonist, either direct or indirect to overcome its
effects. The options in the list are c and (d). Acetylcholine won’t
work because it gets broken down way to rapidly by
acetylcholinesterase, and thus is useless to inject.
Physostigmine will work since it is an indirect
acetylcholinesterase.

How well did you know this?
1
Not at all
2
3
4
5
Perfectly
354
Q

The immediate cause of death from irreversible cholinesterase
inhibitors is
a. shock
b. convulsion
c. cardiac arrhythmia
d. respiratory paralysis
e. dehydration from vomiting and diarrhea

A

(d) - while some of these are indeed associated with
organophosphate toxicity, the immediate cause of death is due
to (d), which results from the stimulation of nicotinic receptors at
the neuromuscular junction resulting in paralysis of skeletal
muscles.

How well did you know this?
1
Not at all
2
3
4
5
Perfectly
355
Q

Each of the following is a symptom of cholinergic crisis except

a. bradycardia
b. lacrimation
c. vasoconstriction
d. extreme salivation
e. weakness of voluntary muscles

A

(c) this is an except question, don’t miss that word! So looking
at the list, we got the S (option d) and the L (option b) from
SLUD, so we are left with (a), c, and (e) as possibles. (e) results
from cholinergic stimulation of the NMJ, so that can’t be it. (a) or
bradycardia, can occur from too much cholinergic stimulation of
the heart (that’s why atropine is useful in surgery, to reverse the
bradycardia that sometimes arises. So, by default,
vasoconstriction is the exception we are looking for.

How well did you know this?
1
Not at all
2
3
4
5
Perfectly
356
Q

Succinylcholine is a short-acting neuromuscular junction blocking
agent useful for providing a brief paralysis to aid in intubating
patients. It is short-acting because
a. it is subject to rapid metabolism in the liver
b. it rapidly redistributes away from the NMJ
c. it is subject to inactivation by plasma esterases
d. it undergoes rapid inactivation in the GI tract

A

(c) that’s why it is long-acting in patient’s that have a deficiency in
this enzyme

How well did you know this?
1
Not at all
2
3
4
5
Perfectly
357
Q

Atropine-like drugs are classed as

a. anti-adrenergic
b. cholinomimetic
c. sympatholytic
d. anti-cholinergic
e. sympathomimetic

A

d

How well did you know this?
1
Not at all
2
3
4
5
Perfectly
358
Q

reserpine class action

A

adrenergic blocker

inhibits NE reuptake, depletes NE

How well did you know this?
1
Not at all
2
3
4
5
Perfectly
359
Q

guanethidine clss action

A

adrenoblocker, inhibits catecholamine release

How well did you know this?
1
Not at all
2
3
4
5
Perfectly
360
Q

alpha methyldopa effect

A

adrenoblocker

How well did you know this?
1
Not at all
2
3
4
5
Perfectly
361
Q

which adrenomimetics block NE reuptake

A

cocaine, TCAs

How well did you know this?
1
Not at all
2
3
4
5
Perfectly
362
Q

which adrenomimetics stimulate release of stored NE

A

amphetamine, ephedrine, tyramine

How well did you know this?
1
Not at all
2
3
4
5
Perfectly
363
Q

which adrenomimetics block destruction of NE

A

MAOIs

How well did you know this?
1
Not at all
2
3
4
5
Perfectly
364
Q

in presence of alpha blocker, what happens to epi

A

decrease in blood pressure bc beta-dilatio predominates

How well did you know this?
1
Not at all
2
3
4
5
Perfectly
365
Q

a1 stimulation

A

vasoconstriction, urinary retention, mydriasis

How well did you know this?
1
Not at all
2
3
4
5
Perfectly
366
Q

b1 stimulation

A

increased heart rat

How well did you know this?
1
Not at all
2
3
4
5
Perfectly
367
Q

b2 stimulation

A

broncho and vasodilation

How well did you know this?
1
Not at all
2
3
4
5
Perfectly
368
Q

a1 block

A

vasodilation

How well did you know this?
1
Not at all
2
3
4
5
Perfectly
369
Q

b1 block

A

decreased heart rate

How well did you know this?
1
Not at all
2
3
4
5
Perfectly
370
Q

b2 block

A

bronchoconstriction

How well did you know this?
1
Not at all
2
3
4
5
Perfectly
371
Q

levodopa

A

Parkinson’s tx
can enter brain
carbidopa given to divert peripheral decarboxylase activity
sympathomimetic
toxicity: face twitches, mental disturbances, nausea vomiting, cardiac arrhythmias

How well did you know this?
1
Not at all
2
3
4
5
Perfectly
372
Q

guanethidine

A

inhibits catecholamine release

373
Q

Alpha or beta-adrenergic blocking drugs act by

a. Inhibiting synthesis of norepinephrine.
b. Increasing the metabolism of norepinephrine
c. Competitive inhibition of postjunctional adrenergic receptors
d. A local anesthetic effect on the adrenergic nerve terminal
e. Depleting norepinephrine from adrenergic nerve terminals

A

c

374
Q

A mechanism for the antiadrenergic action of guanethidine is
a. Inhibition of dopa decarboxylase
b. Increased rate of metabolism of norepinephrine
c. Depletion of norepinephrine from the nerve terminals
d. Substitution for norepinephrine and subsequent action as a
false transmitter
e. Uncoupling of the action potential from the norepinephrine
release mechanism

A

e

375
Q

Which of the following statements most accurately describes the
effectiveness of action of methyldopa?
a. It causes marked cardiac slowing
b. It directly relaxes vascular smooth muscle
c. It causes rapid depletion of norepinephrine from adrenergic
nerve terminals
d. It causes formation of a false transmitter that is released at
vascular smooth muscle
e. It produces a false transmitter, the effect of which is primarily
at central nuclei

A

(e) see, didn’t I just tell you in the previous question that they
could just change the drug and use some of the same
definitions? But they are tricky, since they think you will speed
reading the options, see “false” transmitter, and jump at option
(d), and not read further to see that option (e) is the correct
answer. Drugs like methyldopa and clonidine are the only
centrally acting antihypertensives, reducing sympathetic outflow
via alpha-1 agonist action.

376
Q

The mechanism of action of reserpine is to
a. Inhibit monoamine oxidase
b. Inhibit catechol-O-methyltransferase
c. Block the passage of the nerve action potential in the
postganglionic nerve fibers
d. Stabilize the axon terminal membrane thus preventing release of norepinephrine

A

d

377
Q

Amphetamine acts by

a. Promoting storage of the mediator
b. Causing a rapid release of the mediator
c. Causing a slow depletion of the mediator
d. Combining with a receptor substance on the effector cell
e. Interfering with the response of the receptor to the mediator

A

(b) amphetamine is one of the many indirect-acting
sympathomimetics. It acts by causing the release of
neurotransmitter.Just as a review: cocaine: reuptake inhibition
and release, TCA antidepressant:, reuptake inhibition,
ephedrine:causes release but also acts at receptor itself,
MAOIs: block NT degradation.

378
Q

Which of the following characterizes the mechanism of action on
levodopa?
a. It acts through a direct anticholinergic action
b. It stimulates specific L-dopa receptors in basal ganglia
c. It replenishes the otherwise deficient dopamine in patients with
parkinsonism
d. It increases concentrations of norepinephrine in the brain to
counterbalance an otherwise overactive cholinergic system

A

(c) this you just gotta memorize. Option (d) is true except for the
fact that the imbalance is between dopamine and too much
cholinergic activity, not norepinephrine. Since levodopa has
dopa in its name, you could just narrow the question down to
options (b) and (c). But you still have to know, Parkinson’s is
due to a deficiency in dopamine. So the treatment is to restore
DA levels.

379
Q
Which of the following combinations of agents would be necessary to
block the cardiovascular effects produced by the injection of a
sympathomimetic drug?
a. Atropine and prazosin
b. Atropine and propranolol
c. Prazosin and propranolol
d. Phenoxybenzamine and curare
e. Amphetamine and propranolol
A

(c) - The injection of a sympathomimetic (e.g. a drug that acts

like NE) stimulates both alpha and beta-receptors. Alpha-
receptor stimulation produces vasoconstriction, increased

systolic and diastolic pressure and reflex tachycardia. Alpha and
beta-receptor blockers can block these effects. the only
alternative that lists both an alpha (prazosin) and beta
(propranolol) blocker is (c). Atropine is a muscarinic (cholinergic
) receptor blocker that would accelerate the heart, the opposite
effect that you want, thus (a) and 2 are wrong.
Phenoxybenzamine is an alpha-blocker, but curare is a nicotinic
receptor blocker, not beta receptor- thus (d) is wrong. As for #
5, amphetamine is an indirect acting sympathomimetic, not a
blocker, thus (e) is wrong.

380
Q

Which of the following drugs competitively blocks the action of
norepinephrine at beta-adrenergic receptors?
a. Atropine
b. Naloxone
c. Propranolol
d. Phentolamine
e. Hexamethonium

A

c

381
Q
Pretreatment with reserpine prevents a response to which of the
following agents?
a. Amphetamine
b. Epinephrine
c. Phenylephrine
d. Isoproterenol
d. Norepinephrine
A

(a) reserpine causes depletion of NE from storage sites, thus it
cannot be released by amphetamine. All of the others listed act
postsynaptically. Since this drug is no longer used clinically, I
would be very surprised if they asked such a question! Wait, I
think I said that a few questions ago!

382
Q
Each of the following drugs is considered to be a direct-acting
catecholamine EXCEPT
a. Epinephrine
b. Amphetamine
c. Isoproterenol
d. Norepinephrine
A

b

383
Q
After pretreatment with phentolamine, intravenous administration of
epinephrine should result in
a. Relaxation of bronchial smooth muscle
b. Positive chronotropic and inotropic effects
c. Splanchnic vasoconstriction
d. Dilation of skeletal muscle vascular beds
e. Secretion of a mucoid viscous saliva
i. a, and b only
ii. a, b and d
iii. a, d and e
iv c and d only
v. c, d and e
A

(ii) phentolamine is an alpha blocker, thus the epinephrine will
stimulate beta receptors primarily, with the effects listed in
option (ii): relaxation of bronchiolar smooth muscle (Beta-1),
skeletal muscle vessel dilation (beta-2), and positiver
chronotropic and inotropic action on the heart (b)

384
Q

Which of the following changes produced by intravenous

administration of epinephrine result from stimulation of beta-
adrenergic receptors?

a. Respiratory inhibition
b. Cardiac acceleration
c. Dilation of the pupil
d. Increased systolic pressure
e. Decreased diastolic pressure

i. a, b and c
ii. a, b and d
iii. b and e
iv. c, d and e
v. c and e only

A

(iv)well, beta receptor stimulation means stimulation of beta-1
receptors in the cardiac muscle,which will increase systolic bP,
and beta-2 stimulation which will dilate vessels going to the liver
and skeletal muscle, producing a decrease in diastolic BP.
Mydriasis or papillary dilation is also a beta receptor response,
so c is also right. While you might be tempted by “cardiac
acceleration”, none of the options including (b) work.

385
Q

Which of the following is NOT an action of epinephrine when administered intravenously in a high dose?

a. Increases liver glycogenolysis
b. Causes bronchiolar constriction
c. Produces a rise in blood pressure
d. Evokes extrasystoles in the heart
e. Produces restlessness and anxiety

A

(b)all of the above are actions of epinephrine except bronchiolar
constriction. Epinephrine would cause bronchodilation – that is
why it is used to treat acute brochospasm. So option (b) has to
be the exception, because it is just obviously wrong!

386
Q

“Epinephrine reversal” of blood pressure can best be demonstrated
by injecting epinephrine intravenously after pretreatment with
a. Prazosin
b. Atropine
c. Propranolol
d. Neostigmine
e. Isoproterenol

A

(a) epinephrine is a potent stimulator of both alpha and beta
receptors. Injection of epi usually causes a rise in blood
pressure due to 1) myocardial stimulation that increases
ventricular contraction, 2) an increase in heart rate, and most
important, 3) vasoconstriction due to alpha receptor stimulation.
However, blood flow to skeletal muscles is increased due to
powerful beta-2 receptor vasodilator action that is only partially
counterbalanced by a vasoconstrictor action on the alpha
receptors that are also present in the vascular bed. When given
in the presence of an alpha blocker, beta-receptor mediated
vasodilation is more pronounced, the total peripheral resistance
is decreased and the mean blood pressure falls. This decrease in blood pressure is called “epinephrine reversal”. The only
alpha-blocker listed is prazosin, answer (a). Atropine is a

cholinergic muscarinic receptor blocker, propranolol is a beta-
blocker, neostigmine is a cholinesterase inhibitor, and

isoproterenol is a predominately beta receptor agonist.

387
Q

Each of the following is a predictable adverse effect of drugs that
block the sympathetic nervous system EXCEPT
a. Gastrointestinal disturbance
b. Postural hypotension
c. Nasal congestion
d. Urinary retention
e. Miosis

A

d

388
Q

Injection of a pressor dose of norepinephrine may result in a
decreased heart rate because of
a. Activation of baroreceptor reflexes
b. Direct stimulation of alpha receptors
c. Direct stimulation of beta-1 receptors
d. Direct stimulation of beta-2receptors
e. Direct stimulation of muscarinic receptors

A

(a)- alternatives 2-4 all increase heart rate, while NE has no
effect at muscarinic receptors (e), which are specific for
cholinergic drugs.

389
Q

Alpha-adrenergic agonists are used in combination with local
anesthetics to
a. Stimulate myocardial contraction
b. Reduce vascular absorption of the local anesthetic
c. Increase the rate of liver metabolism of the local anesthetic
d. Increase the concentration of the local anesthetic at its
receptor site
e. Antagonize the vasodilating effects of the local anesthetic

A

(iii) Answer is (c)- Alpha-adrenergic agonists such as
epinephrine produce vasoconstriction, which would accomplish
both “b” and “e”. # 3 is the only alternative that includes both b
and e thus you don’t have to know anything else. “a” and “c”
are false. Vasoconstrictors are include in local anesthetic
preparations to (1) prolong and increase the depth of
anesthesia by retaining the anesthetic in the area injected, (2)
reduce the toxic effect of the drug by delaying its absorption into
the general circulation and (3) to render the area of injection
less hemorrhagic.

390
Q
Administration of an otherwise effective pressor dose of epinephrine
could cause an "epinephrine reversal" in a patient taking which of the
following drugs?
a. Reserpine
b. Propranolol
c. Amphetamine
d. Chlorpromazine
e. Lithium carbonate
A

(d) CPZ is a potent alpha blocker like prazosin.

391
Q

Of the following sympathomimetic agents, the most potent bronchodilator is

a. Amphetamine
b. Norepinephrine
c. Phenylephrine
d. Isoproterenol
e. Methoxamine

A

(d) What is needed for bronchodilation is relaxation of bronchial
smooth muscles. This is accomplished with beta2 receptor
stimulation. Isoproterenol is the only drug listed with potent
beta2 action. (a) stimulates alpha receptors in the CNS, (b) NE
stimulates alpha and beta1 receptors more than beta2, (c)
phenylephrine is an alpha receptor agonist, while (e)
methoxamine is a vasoconstrictor that stimulates alpha
receptors preferentially.

392
Q
Administration of which of the following drugs would produce
vasoconstriction of the gingival vessels?
a. Levonordefrin
b. Phentolamine
c. Epinephrine
d. Propranolol
e. Phenylephrine
i. (a) and (b)
ii. (a) and (c) only
iii. (a), (c) and (e)
iv. (b), (d) and (e)
v. (b) and (d) only
A

(iii) vasoconstriction is a result of stimulation of the smooth
muscle of the peripheral vasculatire, which is an alpha-1
stimulatory action. Of the options, (a), (c) and (e) are alpha-1
agonists. Phentoalmine is a nonselective alpha blocker and thus

would cause vasodilation. Propranolol is a non-specific beta-
blocker.

393
Q

Carbidopa, a dopa-decarboxylase inhibitor, is often used in the
treatment of parkinsonism because it
a. Potentiates the central action of dopamine
b. Potentiates the central action of norepinephrine
c. Decreases the peripheral metabolism of levodopa
d. Inhibits the peripheral stimulatory fibers from the central
nervous system
e. Increases the permeability of the blood-brain barrier to
levodopa

A

c

394
Q
Of the following, one of the most effective treatments currently
available in the U.S. for most patients suffering from parkinsonism
involves oral administration of
a. Dopamine
b. Amantadine
c. Benztropine
d. Levodopa alone
e. Levodopa plus carbidopa
A

e

395
Q

Levodopa therapy for Parkinson disease may result in each of the
following effects EXCEPT:
a. Development of abnormal, involuntary movements,
especially in the face
b. Extreme sensitivity to sympathomimetic drugs
c. Exacerbation of an acute psychosis
d. Nausea and vomiting
e. Extreme sedation

A

(e) levodopa apparently overstimulates DA receptors in the
basal ganglia and can cause visual and auditory hallucinations
(option c), dyskinesia (option (a)), mood changes, depression
and anxiety. It also can stimulate the emetic center, producing
the effects given in (d). That leaves you with a choice between
(b) and (e). Well, apparently, one of the first things you should
know about levodopa therapy is that some people think that
levodopa can sensitize the beta-1 receptors in the heart and this
is a contraindication to using a local anesthetic containing
epinephrine. So option (e) is all that is left to be the exception.
How you are supposed to remember all this stuff about
levodopa is way beyond me, but good luck!

396
Q

Adverse effects of levodopa include:

a. Arrhythmias
b. Psychotic disturbances
c. Nausea and vomiting
d. Abnormal involuntary movements
i. (a), (b), and (c)
ii. (a), (b) and (d)
iii. (a), (c) and (d)
iv. (b), (c) and (d)
v. All of the above

A

v

397
Q
Which of the following drugs would be most likely taken by an
asthmatic patient?
a. Phenylephrine
b. Albuterol
c. Pseudoephedrine
d. Prazosin
e. Propranolol
A

(b) an asthmatic patient typically takes a drug that has
bronchodilatory effects. For this action, you need a drug that is a
beta-2 agonist (remember those charts?). Of the list, albuterol is
the only beta-2 agonist listed. Phenylephrine is an alpha-1 agonist used for stuffy noses, pseudoephedrine might have distracted you,
since it is a decongestant, prazosin and propranolol are
antihypertensives.

398
Q

A pharmacologic agonist is a chemical substance that

a. Binds to a specific receptor and produces a response
b. Elicits a pharmacologic response without binding to a receptor
c. Possesses the property of affinity but not of intrinsic activity
d. Exhibits no activity except to oppose the effect of an antagonist

A

a

399
Q

When comparing drugs with respect to intensity of response, the drug
that produces the greatest maximum effect is the one with the highest
a. Affinity
b. Potency
c. Efficacy
d. Therapeutic index

A

c

400
Q

If drug has a greater efficacy than drug B, then drug A

a. Is more toxic than drug B
b. Has a greater affinity for the receptor than drug B
c. Has a greater margin of safety than drug B
d. Is capable of producing a greater maximum effect than drug B

A

d

401
Q

A drug with a high LD50 and a low ED50 has a

a. High therapeutic index and is, therefore, very dangerous
b. High therapeutic index and is, therefore, relatively safe
c. Low therapeutic index and is, therefore, very dangerous
d. Low therapeutic index and is, therefore, relatively safe

A

(b) TI= LD50/ED50, never forget

402
Q
The ratio of the median lethal dose (LD50) to the median effective
dose (ED50) is the
a. Morbidity index
b. Mortality index
c. Anesthetic ratio
d. Therapeutic index
A

d

403
Q

The therapeutic index of a drug is the ratio of

a. The effective dose to the toxic dose
b. Half the toxic dose to half the effective dose
c. The maximum tolerated dose to the minimum effective dose
d. The lethal dose for 50% of animals to the effective dose for 50% of animals

A

d

404
Q

The phenomenon in which two drugs produce opposite effects on a
physiologic system but do not act at the same receptor site is
a. Potentiation
b. Chemical Antagonism
c. Competitive antagonism
d. Physiologic antagonism
e. Noncompetitive antagonism

A

d

405
Q

Epinephrine antagonizes the effects of histamine by

a. Preventing the release of histamine
b. Acting on the central nervous system
c. Producing physiologic actions opposite to that of histamine
d. Competitively blocking histamine at the cellular receptor site

A

(c) epinephrine acts as a physiologic antagonist. It
nonspecifically antagonizes histamine by exerting its own
distinct effects, for example, vasoconstriction, bronchodilation,
and decreased GI motility. It does not reverse the effect of
histamine by blocking at a specific receptor ((d)), as do
antihistamines. It does not prevent the release of histamine as
does a drug such as cromolyn ((a), by preventing mast cell
degranulation). Answer (b) is not relevant, while (a) is the
mechanism of action for cromolyn, which inhibits mast cell
degranulation..

406
Q
Interaction between nitroglycerin and epinephrine is what type of
antagonism?
a. Allosteric
b. Physiologic
c. Biochemical
d. Competitive
e. Pharmacologic
A

(b) epinephrine would stimulate alpha adrenergic receptors to
produce vasoconstriction, whereas nitroglycerin relaxes
vascular smooth muscle. Thus the two drugs would have
opposing actions. However, the actions are produced by the
drugs acting on different mechanisms; nitroglycerin does not act
at alpha receptors as does epinephrine. If it did, the interaction
would be competitive. In this case the interaction is via
competing physiological effects.

407
Q
When the combined action of two drugs is greater than the sum of
their individual actions, this is
a. Induction
b. Synergism
c. Idiosyncrasy
d. Hypersensitivity
e. Cumulative action
A

b

408
Q

Which of the following responses is least predictable in occurrence?

a. Toxicity
b. Side-effects
c. Idiosyncrasy
d. Tachyphylaxis
e. Therapeutic effects

A

(c) Idiosyncratic reactions are genetically determined abnormal
responses to a drug. They are the most unpredictable in
occurrence because the genetically-based difference
responsible for such a reaction to a drug may not become
evident until the drug is taken for the first time by the patient.
Typically, the effect is one of abnormal sensitivity to a drug,
such that a therapeutic effect is present at doses much lower
than normally used, while the normal dose may result in a toxic
reaction. An example is the response to succinylcholine in
patients with atypical plasma cholinesterase. These patients
don’t metabolize succinylcholine at the same rate, and thus
show a prolonged drug action and increased sensitivity to the
drug. The other alternatives are typically related to the dose of
the drug and because the majority of the population do not
possess an atypical genetic basis for the response to the drug,
the effects are predictable given the dose and knowledge of
what the drug does.

409
Q

Idiosyncrasies to drugs are related to

a. Species
b. Genetic factors
c. Age of the patient
d. All of the above

A

b

410
Q

Two drugs, A and B, have the same mechanism of action. Drug A in
a dose of 5 mg. produces the same magnitude of response as drug B
in a dose of 500 mg. Which of the following statements is correct?
a. Drug A is less toxic
b. Drug A is more efficacious
c. Drug A is 100 times as potent
d. Drug A has a shorter duration of action
e. Drug A is a better drug to use when a maximal response is
desired

A

c

411
Q

According to the theory that agonists and antagonists occupy the
same receptor site, an effective antagonist should exhibit
a. High intrinsic activity and high affinity
b. Low intrinsic activity and low affinity
c. High intrinsic activity and low affinity
d. No intrinsic activity and high affinity

A

d

412
Q

All of the following statements are true regarding the occupation
theory of drug-receptor interaction EXCEPT:
a. The affinity of a drug is dependent on its intrinsic activity
b. The maximum effect of a drug occurs when all receptors are
occupied
c. An antagonist has affinity for the receptor but not intrinsic activity
d. The magnitude of the effect of a drug is proportional to the
number of receptors occupied
e. It follows the law of mass action

A

a

413
Q

The occupational theory of drug-receptor interaction states that
a. The magnitude of the drug response is proportional to the
number of receptors occupied
b. A partial agonist has intrinsic activity but no affinity for the
receptor site
c. An antagonist drug has affinity but no intrinsic activity
d. The rate at which the drug-receptor complex associates and
dissociates determines drug efficacy
e. The degree of drug action is dependent on the law of mass
action

A

(ii)

414
Q

contraindication for barbiturates

A

intermitten porphyria

415
Q

barbiturate overdose

A

respiratory depression

416
Q

Diazepam is preferred to a barbiturate as an antianxiety agent
because diazepam
a. Produces no sedation
b. Has less addiction potential
c. Is a very short-acting drug
d. Is substantially less expensive
e. Does not potentiate the action of CNS depressants

A

b

417
Q
Benzodiazepines produce their antianxiety effects by modulating
which of the following neurohumors?
a. GABA
b. Glycine
c. Dopamine
d. Acetylcholine
e. Norepinephrine
A

a

418
Q

Benzodiazepines exert their main effect on

a. Neuromuscular junctions
b. Peripheral reflex synapses
c. Central GABAergic neurons
d. Central serotonergic neurons
e. Central adrenergic nerve endings

A

c

419
Q
All of the following pertain to general anesthesia induced by thiopental
EXCEPT:
a. Fast induction
b. Decreased secretions
c. Low therapeutic index
d. Short duration of anesthesia
e. Predisposition to laryngospasm
A

(b) Barbiturates are problematic as anesthetics because they
often induce excessive salivation and bronchial secretion,
usually requiring the use of an anticholinergic drug to be
administered to reduce these secretions. Thus (b) has to be the
false statement.

420
Q

Which of the following factors contributes to the short duration of
action of a single dose of thiopental?
a. rapid biotransformation
b. rapid accumulation in body fat
c. high lipid solubility of the undissociated form
d. ability to enter and leave the brain tissue rapidly

A

(d) is correct- thiopental is the classic example always given of
a drug whose duration of action is determined by redistribution
away from its site of action in the brain to less well perfused
tissues.

421
Q

Speed of recovery from short-acting anesthesia with thiopental
depends chiefly on the rapidity of
a. Renal tubular secretion
b. Hepatic degradation of the thiopental group
c. Redistribution from the brain to skeletal muscle
d. Reverse diffusion across the blood-brain barrier
e. None of the above

A

c

422
Q
The action of the ultrashort-acting barbiturates is terminated primarily
by the process of
a. Oxidation
b. Redistribution
c. Renal excretion
d. Plasma protein binding
e. Conjugation with sulfate
A

b

423
Q

Which of the following is NOT characteristic of barbiturates?

a. Possess anticonvulsant properties
b. Possess significant analgesic properties
c. Possess serious drug dependence potential
d. Vary in degree of lipid solubility and hypnotic potency

A

b

424
Q

The cause of death from acute barbiturate poisoning is

a. Convulsions
b. Liver damage
c. Renal failure
d. Respiratory failure
e. Cardiovascular depression

A

d

425
Q

Which of the following adverse effects is most commonly associated
with administration of an intravenous barbiturate?
a. Hypotension
b. Renal failure
c. Hepatic necrosis
d. Nausea and vomiting
e. Respiratory depression

A

e

426
Q

Important steps in the treatment of barbiturate poisoning include:

a. Maintaining an open airway
b. Increasing the input of afferent stimuli
c. Maintaining respiration
d. Administering a narcotic antagonist
e. Administering a central nervous system stimulant

i. (a) and (b) only
ii. (a), (b), (c) and (e)
iii. (a) and (c) only
iv. (b) and (e) only
v. (d) only

A

ii

427
Q

Which of the following are true regarding barbiturates?

a. Significantly elevates pain thresholds
b. Are metabolized by the liver
c. Are classified according to duration of action
d. Depress all levels of the CNS
e. Cause death by cardiovascular depression

i. (a), (b) and (c)
ii. (a) and (d)
iii. (b), (c) and (d)
iv. (b) and (e)
v. (c), (d) and (e)

A

(iii) Answer is (c)- Barbiturates are not analgesics, thus any
answer with “a” such as (a) can be eliminated.. “b” is true, thus
the answer must contain “b” as one of the alternatives, thus (d)
and (e) are eliminated.. “c” is also true, barbiturates are always
classified according to duration of action (thiopental- ultra-short
acting; phenobarbital-long-acting, etc.). This eliminates # 3 so
the answer must be (b). Of course, you should have been able
to rapidly eliminate answers (c), 4, and 5 because barbiturates
cause death by respiratory depression, not cardiovascular
depression.

428
Q

Barbiturates are contraindicated in a dental patient with:

a. Emphysema
b. Hypertension
c. Undiagnosed severe pain
d. Acute intermittent porphyria
i. (a), (c) and (d)
ii. (a) and (d) only
iii. (b) and (c)
iv. (b) only

A

(i) Answer is (a)- “d” is the absolute contraindication for
barbiturate use, since these drugs stimulate the synthesis of
enzymes involved in the synthesis of porphyrins and thus will
aggravate this disease. Thus the answer must contain “d”,
eliminating (c) and (d). Since both (a) and (b) differ only by
alternative c, that is the second fact you must know.
Barbiturates are not analgesics, but sedatives. When pain is
present, they may even make the pain worse, resulting in
arousal, rage and perhaps delirium in the patient. Thus, “c”
would seem to be a pretty strong contraindication, making (a)
the right answer.

429
Q

If diazepam (Valium) is to be given intravenously, it is recommended
that a large vein be used in order to
a. Hasten the onset of action
b. Decrease the risk of thrombophlebitis
c. Offset the vasoconstrictor qualities of diazepam
d. None of the above

A

(b) is correct. This is one of the adverse side effects of IV
diazepam. None of the other alternatives apply.

430
Q

another name for diazepam

A

valium

431
Q

another name for valium

A

diazepam

432
Q
The most important therapeutic measure to be taken in a case of
barbiturate poisoning is to
a. Alkalinize the urine
b. Aspirate stomach contents
c. Administer a CNS stimulant
d. Assure adequate respiration
e. Administer osmotic diuretics
A

(d) barbs can be lethal due to respiratory depression, so keep
them breathing!

433
Q

benzos given IV side effect

A

thrombophlebitis

434
Q
For oral sedation in the dental setting, the most ideal group of agents
is:
a. Narcotics
b. Barbiturates
c. Antihistamines
d. Benzodiazepines
e. Anticholinergics
A

(d) while all have sedative actions, and have their uses given the
appropriate setting, benzodiazepines are by far the most popular
now given their safety compared to the other agents. The most
popular oral BDZs has been diazepam (Valium), but there is a
trend recently to the ultrashort acting versions such as triazolam
(Halcion).

435
Q
  1. Which of the following is an advantage of midazolam over diazepam
    EXCEPT one. Which one is this EXCEPTION?
    a. Lesser incidence of thrombophlebitis
    b. Shorter elimination half-life
    c. No significant active metabolites
    d. Less potential for respiratory depression
    e. More rapid and predictable onset of action when given
    intramuscularly
A

BDZs in general are great for dental sedation
precisely because they present less risk of
respiratory depression in comparison to barbiturates
and opioids. This is an advantage shared across all
the BDZs, so if you know this you don’t really have
to know anything else. The others are true
advantages of midazolam vs. diazepam. Midazolam
is water soluble, and thus does not need to be
dissolved in propylene glycol like valium – it is the
solvent that cause the thrombophlebitis seen with
valium injections. Midazolam also is short acting
compared to valium because it does not have active
metabolites like valium. Because of the above
factors, statement (d) is also true.

436
Q
Which of the following drugs best reverses the effects of
benzodiazepines?
a. Naloxone
b. Flumazenil
c. Midazolam
d. Aminophylline
e. Physostigmine
A

b
naloxone, since this is used to reverse the effects of Opioids.
Midazolam is a BDZ, aminophylline is a theophylline derivative, and
physostigmine is a cholinesterase inhibitor.

437
Q

1st gen antipsychotics mechanism

A

blocking dopaminergic receptors

438
Q

1st gen antipsychotics side effects

A

tardive dyskinesia from extrapyramidal stimulation

anticholinergic effects

439
Q

haloperidol class

A

antipsychotic 1st generation

440
Q

clozapine class

A

2nd gen antipsychotic

441
Q

2nd gen vs 1st gen antipsychotics

A

less extrapyramidal side effects (like tardive dyskinesia)
treat negative and positive symptoms
dopaminergic + serotonin (%-HT) receptors

halo = 1st; clozapine - 2nd

442
Q

fluoxetine class

A

SSRI

443
Q

TCA example

A

imipramine

amitriptyline

444
Q

MAO example

A

phenylene

445
Q

manic phase tx

A

Lithium

446
Q
Which of the following are pharmacologic properties of antipsychotic
drugs?
a. They block the dopamine receptor
b. They affect the hypothalamic temperature regulation system
c. They cause emesis
d. They are synergistic with LSD
e. They cause hypertension
i. (a) and (b) only
ii. (a), (b) and (c)
iii. (b), (c) and (d)
iv. (c), (d) and (e)
v. (d) and (e) only
A

(i) You should immediately know that (a) has to be included as
part of the answer, thus i or ii has to be right. If you also can
dredge up from the recesses of your memory banks that
antipsychotics are also effective antiemetics, and are often used
clinically for this purpose, then you realize that (c) is wrong, thus
limiting your answer to i.

447
Q

The antipsychotic effects of the haloperidol are probably the result
of
a. Release of serotonin in the brain
b. Release of norepinephrine in the brain
c. Blockade of dopaminergic sites in the brain
d. Prevention of the release of norepinephrine from brain neuron
terminals
e. Increase in the dopamine content of the cerebral cortex

A

c

448
Q

The antipsychotic effects of haloperidol result from
a. Release of serotonin in the brain
b. Release of norepinephrine in the brain
c. Blockade of dopaminergic sites in the brain
d. An increase in the dopamine content of the cerebral cortex
e. Prevention of the release of norepinephrine from brain neuron
terminals

A

c

449
Q
Chlorpromazine and related drugs are thought at act by blocking
which of the following receptors?
a. Adrenergic
b. Muscarinic
c. Dopaminergic
d. Central serotonin
A

(c) Answer is (c)- Chlorpromazine is the prototypic
phenothiazine, an antipsychotic drug used in the treatment of
schizophrenia. Other antipsychotic drugs used for this purpose
are haloperidol and thioridazine. These drugs act via
dopaminergic receptors.

450
Q

Which of the following is an irreversible side-effect resulting from long-
term administration of phenothiazines?

a. Sedation
b. Xerostomia
c. Infertility
d. Parkinsonism
e. Tardive dyskinesia

A

e

all are seen – key word here is irreversible

451
Q
Tardive dyskinesia is a neurological side-effect of which of the
following classes of drugs?
a. Alcohols
b. Tricyclic antidepressants
c. Barbiturate antiepileptics
d. Phenothiazine antipsychotics
e. Monoamine oxidase inhibitors
A

(d) Tardive dyskinesia is an irreversible condition that consists
of involuntary movement of skeletal muscles, a condition which
may be seen following prolonged use of drugs. This is typically a
dopaminergic mediated effect. The phenothiazine antipsychotics
are the only drugs listed which act via dopamine. The others,
such as tricyclics and MAO inhibitors affect adrenergic
transmission, barbiturates act via GABA, as does alcohol.

452
Q
Which of the following drugs are most likely to cause extrapyramidal
stimulation?
a. Antibiotics
b. Salicylates
c. Barbiturates
d. Phenothiazines
e. Benzodiazepines
A

(d) - Extrapyramidal side effects are the major side effects of
antipsychotic medication and include Parkinson-like effects as
well as tardive dyskinesia (from development of supersensitivity
resulting from chronic blockade of dopamine receptors in basal
ganglia)- abnormal rapid alternating movements of tongue and
perioral areas, facial grimacing, etc. Phenothiazines are the only
antipsychotic drugs listed.

453
Q

Phenothiazine derivatives do NOT produce

a. Jaundice
b. Xerostomia
c. Gingival hyperplasia
d. Postural hypotension
e. Symptoms of parkinsonism

A

(c) Answer is (c)- (c) should stand out immediately because
this is almost always mentioned as a side effect of dilantin,
which is not a phenothiazine, but an anti-convulsant.
Phenothiazine derivatives are antipsychotic drugs such as
haloperidol (Halcyon) or chlorpromazine used in the treatment of
disorders such as schizophrenia. You should remember that the
most troubling side effect of these drugs is the production of
tardive dyskinesia and the parkinsonian-like extrapyramidal
disorders. Thus # 5 is eliminated. As a rule these drugs have
anticholinergic and anti-alpha-adrenergic side effects, which
would eliminate xerostomia, and postural hypotension (due to
an anti-adrenergic depressant effect on both vasomotor centers
and the autonomic nervous system) as possible answers.
Jaundice is a less frequent side effect than the extrapyramidal
symptoms, and often results from an allergic reaction to these
drugs. Thus the answer is (c).

454
Q
All of the following actions are associated with the use of
chlorpromazine EXCEPT:
a. Jaundice
b. Photosensitivity
c. Excessive salivation
d. Anticholinergic effects
e. Antiadrenergic effects
A

c – bc anticholinergic effects, dry

455
Q

Which of the following is most likely to be the major mechanism of
action of the tricyclic antidepressants?
a. Enhanced release of acetylcholine
b. Inhibition of neuronal synthesis of norepinephrine
c. Potentiation of serotonin synthesis in nervous tissue
d. Stimulation of reuptake of norepinephrine from the synaptic cleft
e. Blockade of the reuptake of amine neurotransmitters released
into the synaptic cleft

A

(e) a long winded way of saying reuptake blockers

456
Q

Tricyclic antidepressants have a prominent side effect that most
nearly resembles the usual pharmacological action of
a. Codeine
b. Atropine
c. Ephedrine
d. Propranolol
e. Methacholine

A

(b) TCAs are strong anticholinergics, and atropine is the only
anticholinergic drug listed

457
Q
Which of the following drugs has its primary use in the treatment of
the manic phase of depressive psychosis?
a. Lithium
b. Reserpine
c. Imipramine
d. Amphetamine
e. Chlorpromazine
A

a

458
Q

Lithium carbonate is particularly effective in treating

a. Parkinsonism
b. Hypertension
c. Schizophrenia
d. Acute anxiety
e. Manic-depressive psychosis

A

e

459
Q
The current drug of choice for treatment of the manic phase of
manic-depressive psychosis is
a. Lithium
b. Caffeine
c. Reserpine
d. Imipramine
e. Amphetamine
A

a

460
Q

Which of the following statements is NOT true regarding the adrenal
corticosteroids?
a. Cause retention of sodium and fluid
b. Decrease activity in lymphoid tissue
c. Heighten the immune response to antigens
d. Can produce a diabetes-like syndrome with high blood levels
e. Are therapeutically beneficial when administered orally,
parenterally or topically

A

c

461
Q
Which of the following conditions contraindicate use of
corticosteroids in a dental patient?
a. AIDS
b. Candidiasis
c. Tuberculosis
d. Peptic ulcers
i. (a), (b) and (c)
ii. (a), (b) and (d)
iii. (b), (c) and (d)
iv. (c) and (d) only
v. All of the above
A

(v) Answer is (e)- Corticosteroids are antiinflammatory drugs
used topically, orally and parenterally. However, they suppress
the immune system of the body. They have been known to
cause peptic ulcers, as well as mask the symptoms of an ulcer,
and perforation and hemorrhage may result. Thus “d” has to be
in the answer, eliminating (a). Because they are
immunosuppressive, they would obvious make an AIDS patient,
who already has a compromised immune system, worse.
Similarly, latent tuberculosis could also be reactivated. Thus # 3
and (d) can be eliminated. Use of corticosteroids in inhalers for
asthma, while advantageous in reducing the side effects
resulting from systemic administration, has led to an increase in
problems with Candidiasis, so “b” has to be in the answer. (e) is
the only answer that meets all these requirements.

462
Q
Which of the following does NOT result from prolonged treatment with
steroids?
a. Gastric ulcer
b. Osteoporosis
c. Hyperglycemia
d. Myocardial atrophy
e. Redistribution of body fat
A

d

463
Q

Glucocorticosteroids are useful as secondary treatment of anaphylaxis
because they
a. Inhibit the production of antibodies
b. Prevent the union of antigen with antibody
c. Prevent the release of histamine from sensitized cells
d. Suppress the inflammatory response to cell injury
e. Inhibit the release of serotonin from vascular storage sites

A

(d) Answer is (d)- Glucocorticoids such as hydrocortisone are
classed as antiinflammatories, inhibiting every step of the
inflammatory process.- thus (d) is the correct answer. The other
alternatives are single steps along the pathway, that are
handled by other drugs that are more selective than
glucocorticoids.

464
Q
Adrenal steroids are used successfully to treat all of the following
conditions EXCEPT
a. Gastric ulcers
b. Addison disease
c. Lupus erythematosus
d. Rheumatoid arthritis
e. Aphthous stomatitis
A

(a) adrenal steroids, otherwise known as corticosteroids,

actually cause gastric ulcers! All the rest are therapeutic uses.

465
Q

general anesthetics: solubility vs dose needed

A

more soluble - need to give more

466
Q

halothane side effect

A

hepatotoxicity

467
Q

stages of anesthesia

A

1 - analgesia
2 - delirium
3 - surgical anesthesia
4 - medullary paralysis

468
Q

Signs and stages of anesthesia are most likely to be seen with a
general anesthetic that has a
a. Low potency
b. Slow rate of induction
c. Low Ostwald coefficient
d. High oil-water solubility coefficient
e. High tissue-blood partition coefficient

A

b

469
Q

All of the following influence the rate of induction during anesthesia
EXCEPT:
a. Pulmonary ventilation
b. Blood supply to the lungs
c. Hemoglobin content of the blood
d. Concentration of the anesthetic in the inspired mixture
e. Solubility of the anesthetic in blood (blood-gas partition coefficient, Ostwald coefficient).

A

c

470
Q
The rapidity of onset of anesthesia with an inhalation anesthetic agent
is primarily related to its
a. Molecular weight
b. Degree of blood solubility
c. Temperature in the gas phrase
d. Interaction with preoperative drugs
A

(b) the rule is, the more insoluble the agent is, the faster the
onset and offset of effect. That’s why nitrous oxide, which is
very insoluble in blood works so fast and leaves the body so
quickly once you stop administration.

471
Q

Which of the following forms of drug toxicity is associated with the
halogenated hydrocarbon general anesthetics?
a. Liver damage
b. Myocardial atrophy
c. Peripheral neuritis
d. Severe hypertension

A

a

472
Q

The behavior of patients under general anesthesia suggests that the
most resistant part of the central nervous system is the
a. Spinal cord
b. Medulla oblongata
c. Cerebral cortex (motor area)
d. Cerebral cortex (sensor area)

A

b

473
Q

General anesthetics can do all of the following EXCEPT:

a. Produce delirium
b. Stimulate medullary centers
c. Produce a state of unconsciousness
d. Reduce perception of painful stimuli
e. Decrease excitability of the motor cortex

A

(b) maybe cuz it’s the base of the brain? They will eventually
depress medullary centers (Stage IV), patient will stop breathing
and die. (c), (d), and (e) are desirable actions, (a) is stage II of
anesthesia

474
Q

In general anesthesia, the last part of the CNS to be depressed is the

a. Medulla
b. Cerebrum
c. Midbrain
d. Cerebellum
e. Spinal cord

A

a

475
Q

General anesthesia with halothane is commonly preceded by
administration of atropine to
a. inhibit vagal overactivity commonly caused by halothane
b. induce muscular relaxation by blocking cholinergic receptors
c. reduce salivation and bronchial secretions caused by
halothane
d. all of the above.

A

Answer is (c)- this is the classic clinical use of atropine and one
you should have committed to memory. Atropine does not
induce muscular relaxation- that would be a neuromuscular
junction blocker such as curare, thus (b) and (d) are wrong. #!
might confuse you. Atropine is used to override vagal activity,
but that is not the reason it is given before halothane.

476
Q

antihistamines uses

A

H1:
derm allergies (chlorpheniramine)
preop antiemesis, sedation, anticholinergic (promethazone)
parkinson sx control (diphenhydramine)

H2: gastric secretion reduction, heartburn - cimetidine

477
Q

cimetidine class use

A

H2 antihistamine, gastric secretion control, heartburn

478
Q

promethazine class use

A

H1: preop, sedation, antiemesis, anticholinergic

479
Q

chlorpheniramine class use

A

H1 antihistamine, derm allergies

480
Q

diphenhydramine class use

A

H1 parkinson sx control

481
Q
Which of the following drugs is useful in treating dermatologic
manifestations of an allergic response?
a. Diazepam
b. Atropine
c. Hexylresorcinol
d. Chlorpheniramine
e. Phenoxybenzamine
A

d

482
Q

Cimetidine is used therapeutically to

a. Stimulate respiration
b. Protect against anaphylaxis
c. Decrease gastric acid secretion
d. Hasten excretion of barbiturates
e. Dilate smooth muscles of the bronchioles

A

c

483
Q
Gastric acid secretion has been shown to be most effectively reduced
with the use of
a. Adrenal steroids
b. Anticholinergic drugs
c. Serotonin antagonists
d. H1-histamine receptor antagonists
e. H2-histamine receptor antagonists
A

e

484
Q

Drug-mediated inhibition of H2-histamine receptors is most useful in treating

a. Asthma
b. Anaphylaxis
c. Contact dermatitis
d. Gastric hyperacidity
e. Localized allergic reactions

A

d

485
Q
Which of the following antihistamines is most commonly used as
preoperative medication?
a. Meclizine
b. Cyclizine
c. Promethazine
d. Dimenhydrinate
e. Chlorpheniramine
A

(c) meclizine is used for vertigo, the others for allergies

486
Q

Use of diphenhydramine (Benadryl ) in controlling the symptoms of
parkinsonism is based upon which of the following effects?
a. Anticholinergic
b. Local anesthetic
c. Adrenergic-blocking
d. CNS depressant on the midbrain
e. Stimulant to dopaminergic nerves in the basal ganglia

A

(a) Remember that Parkinsonism is a due to a deficiency of DA
in the brain, and is currently treated with levodopa and
carbidopa. However, prior to these, anticholinergic drugs were
the first drugs found to be somewhat effective for treatment of
this disease, in that cholinergic and dopaminergic tracts interact
in the brain, and thus reducing cholinergic activity via
anticholinergic drugs improves or enhances dopaminergic
function, suggesting one is inhibitory to the other. Drugs with
anticholinergic activity are often still the first drug tried.
Antihistamine drugs such as diphenhydramine often have
strong anticholinergic activity, which accounts for their
effectiveness in drying nasal secretions associated with a cold.
Therefore, the answer is (a). (e) is there to confuse you, but
don’t be. Diphenhydramine does not stimulate dopaminergic
nerves in the basal ganglia. Adrenergic blockers ((c)) do see
some use in the treatment of Parkinson’s, but diphenhydramine
has no adrenergic blocking activity. Diphenhydramine does
have the actions given in (b) and (d), but these are not
responsible for its efficacy in Parkinson’s..

487
Q

The mechanism of action of H1 antihistamines is

a. MAO enzyme inhibition
b. competitive antagonism
c. physiologic antagonism
d. noncompetitive antagonism
e. inhibition of release of bound histamine

A

b
H1 antihistamines are
competitive histamine receptor blockers. Many students answer
(e), but this is the mechanism of action of cromolyn. (c) also
draws some answers, but is wrong-epinephrine is the
physiological antagonist of histamine.

488
Q
24. Which of the following drugs is often used to treat trigeminal
neuralgia?
a. clonazepam
b. carbamazepine
c. acetazolamine
d. succinylcholine
A

Answer is (b)- This is a memorization question. Carbamazepine,
as well as phenytoin, are the main drugs used to treat trigeminal
neuralgia. Clonazepam is a benzodiazepine, succinylcholine is a
depolarizing neuromuscular blocking agent, while acetazolamine
is an anticonvulsant like carbamazepine.

489
Q
The use of epinephrine for local hemostasis during surgery might result
in
a. hypoglycemia
b. cardiac arrthymia
c. an acute asthma attack
d. a drastic drop in blood pressure
e. any of the above
A

The answer is (b), Cardiac arrthymias are the main bad side
effect of epinephrine as a vasoconstrictor. Epi stimulates both
alpha and beta receptors. Beta receptors are found in the heart
and stimulation of beta receptors increases heart rate, force of
contraction, cardiac output and oxygen utilization. (a) is wrong,
epi elevates blood glucose. (c) is wrong- beta stimulation in
smooth muscle and bronchi causes bronchodilation, and is thus
a drug of choice for acute asthmatic attacks. (d) is obviously
wrong- a decrease in blood pressure would not result from
adrenergic stimulation, increased blood pressure is the rule.

490
Q

The highest risk associated with use of oral contraceptives is

a. hepatic necrosis
b. permanent sterility
c. cancer of the breast
d. cancer of the uterus
e. thromboembolic disorders

A

e

491
Q

Factors common to all forms of drug abuse include

a. miosis
b. tolerance
c. physical dependence
d. psychological dependence
e. any of the above

A

The answer is (d). Miosis ((a)) is a cholinergic effect typically
observed with opiates. Cocaine doesn’t cause this kind of effect
for example. (b), tolerance, is common to many drugs such as
opiates, barbiturates and sedatives, but has not been clearly
demonstrated for all abused drugs. (c), physical dependence, is
usually thought to be true only for morphine, alcohol, caffeine,
and perhaps cocaine.

492
Q

Alcoholic euphoria results from

a. increased activity of the cerebellum
b. increased activity of the cerebral areas
c. increased activity of the spinal synapses
d. decreased activity of the medullary centers
e. removal of inhibitory activity of the cortex

A

Answer is (e)- Alcohol inhibits the CNS, thus eliminating (a), (b),
and (c), leaving the choice between 4 and 5. Alcohol has been
postulated to inhibit GABA effects, the major inhibitory
transmitter in the CNS, especially in the cortex. Thus (e) is the
answer.

493
Q

Sulfonyl ureas cause insulin secretion by

a. adrenergic simulation
b. cholinergic stimulation
c. direct stimulation of pancreatic beta cells

A

Answer is (c)- memorization- alpha agonists decrease insulin
release, beta agonists increase it. Thus because both of these
are adrenergic stimulation, (a) can’t be the answer. (b)-
muscarinic agonists increase insulin release, but this is not the
mechanism of sulfonylureas. They cause insulin release by (c),
and are the primary oral antidiabetic agents used
therapeutically.

494
Q
37. If a patient requiring an extraction reports that he is on dicoumarol
therapy, the laboratory test most valuable in evaluating the surgical
risk is
a. clotting time
b. bleeding time
c. sedimentation rate
d. complete blood cell count
e. plasma prothrombin time
A

Answer is (e)- Dicoumarol is an oral anticoagulant. Thus the risk
in an extraction is that the patient may bleed excessively, or that
a serious interaction with a drug that you might require in your
management of the patient, such as barbiturates or salicylates
may occur. You may need to adjust the anticoagulant activity to
within a safe range for surgical procedures. Since dicoumarol
prevents blood clotting by preventing the conversion of Vitamin
K to prothrombin, (e) is the only appropriate test.

495
Q
A drug has a half-life of 4 hours. If 2 gms are given every 4 hrs what
will be the amount in gms in the body immediately after the third
dose?
a. 1.5
b. 2.0
c. 3.5
d. 4.0
e. 6.0
A

Answer is (c)- 2 gm at 0 hr, at 4 hrs 1/2 from first injection is
gone, leaving 1 gm in the body and you inject 2 more for a total
after the second injection of 3. At the third dose, 1/2 of 3 is
gone, leaving 1.5 gms, you inject 2 more for a total of 3.5
immediately after the third dose.

496
Q

Which of the following combinations represents acceptable agonist-
antagonist pairs in antidotal therapy?

a. morphine-naloxone
b. dicoumarol-protamine
c. warfarin-phenylbutazone
d. carbon monoxide-carbon dioxide

A

(a)
(b) is wrong-Protamine is an antagonist of heparin,
not dicoumarol. (c): phenylbutazone enhances the toxicity of
warfarin by displacing it from plasma protein binding sites. (d):
hyperbaric oxygen would be a useful treatment for carbon
monoxide poisoning, not CO2

497
Q
The various insulin preparations useful in the treatment of diabetes
mellitus differ primarily in
a. locus of action
b. mechanism of action
c. mode of transformation
d. onset and duration of action
e. none of the above
A
Answer is (d)- remember, diabetes medications can be
organized into 3 groups based on their onset and duration of
action: 1)fast-acting: insulin injection, 2) intermediate acting:
Isophane insulin suspension, and 3) long acting: protamine zinc
insulin suspension
498
Q
Displacement of a drug from protein binding sites is expected to
increase the
a. drug effect observed
b. duration of drug effect
c. dose required for a given effect
d. none of the above
A

answer is (a)- most people answer 3, confusing protein binding
sites from active receptor sites where drugs exert their effects.
Protein binding sites are just uptake sites that take up a drug
and keep it from getting to its real receptor site of action. Some
drugs are extensively protein bound, for example coumarin is
97% protein bound and only 3% is left to reach effective sites.
Administration with another drug that is also extensively plasma
protein bound will displace the drug that is already on these
sites via competition, and thus effectively increase the level of
drug that can now get to an active receptor site.

499
Q

In an addisonian crisis (hypoaldosteronism) resulting from stress from
a minor dental procedure, the patient should be treated immediately with
a. 0.5 ml norepinephrine
b. 5 mg. prednisolone acetate
c. 1% triamcinolone acetonide
d. 0.5 ml, 1:1000 epinephrine
e. 2 ml (100 mg) hydrocortisone hemisuccinate

A

Answer is (e)- Addison’s disease results from failure of the
adrenal cortices to produce adrenocortical hormones such as
aldosterone. Aldosterone is a mineralocorticoid that controls
sodium retention and potassium excretion. Lack of aldosterone
results in electrolyte imbalances, with the major problem being
hyponatremia (sodium loss). Similar symptoms may also be
seen when a patient is withdrawn from chronic adrenal steroid
therapy. Due to depressed adrenal function, patients can’t
respond to stressful situations (such as dental procedures)
adequately, and an adrenal crisis may occur. The
recommended treatment is (e), 100 mg of hydrocortisone
hemisuccinate. Of the other corticosteroids given, he dose
given for prednisolone ((b)) is too low, while triamcinolone lacks
any effects on sodium retention. NE and Epi would not be used
in this situation- they might be used in a case of adrenal medulla
insufficiency.

500
Q

Drug A inhibits the biotransformation of Drug B. The duration of
action of drug B in the presence of drug A will usually be
a. shortened
b. prolonged
c. unchanged

A

Answer is (b)- usually biotransformation results in a more water
soluble, more readily excreted form of the parent drug. In most
cases this inactivates the drug, but there are some exceptions
which involve the formation of metabolites with activity
(diazepam) or when an inactive prodrug is given (levodopa)
which becomes active after the first step in the
biotransformation pathway (DOPA) sometimes are asked in
questions of this form.

501
Q
In which of the following groups of drugs is there the most
consistency in chemical structure?
a. diuretics
b. antiepileptics
c. local anesthetics
d. general anesthetics
e. nonbarbiturate sedatives
A
Answer is (c)- Remember that local anesthetics are either
esters or amides. All of the other alternatives diverge widely in
their structures.
502
Q

Cimetidine is administered to

a. aid in sleeping
b. relieve asthma
c. inhibit gastric secretion
d. relieve cold and flu symptoms

A

Answer is (c)- Remember, cimetidine or “Tagamet” is an H2
antihistamine are used therapeutically to inhibit gastric secretion
((c)) in cases of peptic ulcer. This is its only clinical use. You
might see it in a question regarding drug metabolism- it is also a
potent inhibitor of the mixed function oxidase drug metabolizing
enzyme system in the liver. The other alternatives are to
confuse you because you probably at least remember that it is
an antihistamine, but you don’t know the difference between H1
and H2 antihistamines.

503
Q
Which of the following anticancer drugs can be classified as an
antimetabolite?
a. cisplatin
b. lomustine
c. vincristine
d. methotrexate
A

d
(a) & (b) are alkylating agents, (c) is an alkaloid
derived from plants. (d) is a folate antagonist, which acts as an
antimetabolite. Just for your future edification, most cancer
chemotherapy drugs cause cell death by affecting the ability of
cells to divide. The drugs thus inhibit one or more phases of the
cell cycle or prevent a cell in Go (the nondividing phase) from
entering into the cycle of cell division. Antimetabolites may act
in 2 ways (1) by incorporation into a metabolic pathway and
formation of a false metabolite which is nonfunctional or (2) by
inhibition of the catalytic function of an enzyme or enzyme
system. Methotrexate is an example of a cell cycle specific
antimetabolite that inhibits DNA synthesis during the S phase.
Vincristine acts during the mitotic phase

504
Q

cisplatin mechanism use

A

alkylating agentm anti-cancer

505
Q
Which of the following hormones acts to elevate blood concentration
of ionic calcium?
a. glucagon
b. parathyroid
c. aldosterone
d. thyrotropin
e. thyrocalcitonin
A

Answer is (b)- maintaining the concentration of Ca++ in extracellular
fluid by regulating the deposition and mobilization of calcium
from bone, absorption from the GI tract, excretion etc. is the
main function of parathyroid hormone. Thyrocalcitonin is another
name for calcitonin. They hope to confuse you because there is
a correlation between calcitonin and calcium, except that calcium
concentrations regulate the synthesis and release of calcitonin.
Glucagon is a pancreatic hormone that stimulates glucose
production, thyrotropin is there to confuse you with parathyroid
hormone, and aldosterone regulates Na+ levels not Ca++.

506
Q

Disorientation, confusion, and hallucinations resulting from an
overdose of scopolamine are most efficaciously treated by
administering
a. atropine
b. levodopa
c. acetylcholine
d. physostigmine

A

Answer is (d)- scopolamine is a muscarinic receptor blocker
similar to atropine, thus (a) is wrong. Levodopa has nothing to
do with this question. To counteract a competitive muscarinic
receptor block, you need to increase the levels of agonist, in this
case acetylcholine. However, you can’t give ACh because it is
broken down almost instantaneously by acetylcholinesterase.
The answer is to give a drug which knocks out the
acetylcholinesterase, allowing endogenously released ACh to
accumulate to overcome the action of scopolamine. The drug
which will do this is physostigmine.

507
Q

scopolamine is like what

A

like atropine, anticholinergic

508
Q

Developed hyporeactivity to a drug is

a. tolerance
b. antagonism
c. detoxification
d. desensitization

A

a

509
Q

Absorption of a drug from the intramuscular site of administration may
be slowed by
a. excercise
b. vasoconstriction
c. the presence of congestive heart failure
d. administering the drug as an insoluble complex
1. a and b
2. b only
3. b, c, and d
4. b and d
5. all of the above

A

Answer is (c)- “a” can’t be right because exercise increases blood flow
through muscles and thus improves absorption. Thus # 1 is wrong.
“b” is obviously right for the same reason. “c” would also result in
reduced blood flow so slower absorption would also be a problem.
“d” is often used for prolonged and steady drug release so it also is
true. The answer must thus be (c).

510
Q
  1. Salicylism includes which of the following?
    a. nausea
    b. tinnitus
    c. vomiting
    d. gastrointestinal bleeding
  2. a, b, and c
  3. a, b, and d
  4. a and c only
  5. b, c, and d
  6. all of the above
A

Answer is (e)- “Salicylism” is a mild toxic reaction to aspirin
(acetylsalicylic acid), usually occurring after prolonged treatment
with large doses. Nausea, tinnitus, vomiting and GI bleeding are
all symptoms of salicylism. Other notable side effects of aspirin
which result from ingestion of a single large dose are
disturbances of acid-base imbalance (acidosis or alkalosis),
fever, hypoglycemia. Remember, aspirin, is contraindicated in
children suffering from influenza or chicken pox: aspirin has
been implicated in the development of Renee’s syndrome.

511
Q
Which of the following are important criteria for the adequate clinical
evaluation of a new drug?
a. comparison with a placebo
b. evaluation of side effects
c. utilization of control groups
d. comparison with a standard drug
e. double blind experimental design
1. a, b, c, and d
2. a, b, d, and e
3. a, c, d, and e
4. b, c, and e
5. b, d and e only
6. all of the above
A

6

512
Q

Each of the following agents has a long duration of action due to the
presence of liver generated active metabolites EXCEPT
a. diazepam
b. oxazepam
c. flurazepam
d. chlordiazepoxide

A

Answer is (b)- This is a pure memorization question. All of these
drugs are benzodiazepines, which typically have a long duration
of action because they are converted to pharmacologically
active metabolites with long half-lives. Oxazepam, midazolam,
and lorazepam are exceptions that are not converted to active
metabolites. The answer is thus (b).

513
Q

Each of the following statements relates to the general aspects of
toxicology EXCEPT
a. most drugs exert a single action
b. toxicity is both time and dose dependent
c. toxicity can be due to overdosage of a drug
d. symptoms of toxicity can be anything ranging from nausea to
death
e. for some drugs, even a minimal concentration can be harmful

A

a

514
Q
The central actions of ethyl alcohol are not synergistic with which of
the following?
a. diazepam
b. meperidine
c. pentobarbital
d. chlorpromazine
e. methylphenidate
A

Answer is (e)- The central actions of alcohol are depressant.
Thus the choice of correct answer comes down to knowing
which of the drugs listed is not a CNS depressant. Diazepam is
a benzodiazepine, pentobarbital is a barbiturate. Both are
sedating. Meperidine is an opiate, while chlorpromazine is a
phenothiazine antipsychotic. Both of these classes of drugs are
also typically sedating. The correct answer is (e),
methylphenidate (Ritalin). Ritalin is an indirect acting

sympathomimetic, and acts similar to amphetamine in the CNS-
it is a stimulant.

515
Q

A heroin dependent patient should not be given nalbuphine (Nubain)
for pain because
a. it has no analgesic properties
b. it may produce respiratory depression
c. as a mixed agonist-antagonist, it can elicit withdrawal symptoms
d. the high abuse potential of nalbuphine may add to the patient’s
problems

A

Answer is (d)- Nalbuphine is a mixed agonist-antagonist that is
analgesically equipotent with morphine (thus # 1 is ruled out).
However, analgesia is produced by its agonistic effects at
kappa opioid receptors. It has pronounced antagonistic effects
at the mu receptor, and can be used clinically to reverse
respiratory depression (by blocking mu receptors) without a loss
of analgesic effects (by stimulating kappa receptors). Thus, # 2
is ruled out. # 4 is not likely. The mixed agonist-antagonists
were designed to combine analgesia with enough antagonistic
properties to prevent their abuse. Thus, # 3 is the right answer.
Typically, these drugs can mimic the effect of morphine in a
drug free patient, but antagonize opiate action in a dependent
patient, thus precipitating withdrawal. Of this class of drugs,
butorphanol is the exception- it will not precipitate withdrawal
because it is only a weak mu antagonist.

516
Q
All of the following methods of drug biotransformation are classified as
synthetic except:
a. N-alkylation
b. O-dealkylation
c. sulfate conjugation
d. glucuronide conjugation
A

Answer is (b)- Nonsynthetic reactions (phase I reactions)
include the various transformations of molecular structure:
oxidation, reduction, and hydrolysis; they represent the first
stage of biotransformation. Synthetic (phase II) reactions
consist of the conjugation of drugs or their metabolites with
functional groups. Of the alternatives, (c) and (d) are
conjugation reactions, and thus are wrong because they are
synthetic reactions. Alternative (a) is tricky. It is made to sound
like N-dealkylation, which is a nonsynthetic or phase I oxidative
reaction. There is no reaction called N-alkylation. Thus (b), an
oxidative phase I reaction is left as the right answer

517
Q

Which of the following drugs is often administered to treat life-
threatening arrhythmias?

a. quinidine
b. lidocaine
c. verapamil
d. propranolol

A

(b) ventricular arrythmias are the life threatening ones, and lidocaine
is particularly effective for this kind of arrhythmia when given
parenterally in an emergency situation. It has a very rapid onset
of action when given IV, which is obviously important in an
emergency. The others all also have antiarrythmic action, mostly
used for supraventricular tachyarrythmias

518
Q

A male patient who is receiving Coumadin therapy presents for an
elective extraction. His protrhombin time (PT) is prolonged. Which of
the following methods is preferred for reducing the PT to an acceptable
level?
a. administering vitamin K (Aqua Mephyton)
b. Withdrawing Coumadin for two days
c. reducing Coumadin to one half the usual dose for two days
d. administering a Coumadin antagonist, such as heparin
e. administering a platelet transfusion to enhance coagulability

A

(b) the issue is obviously that the patient may bleed excessively, a
situation that would cause problems during dental surgical
procedures. Dentist should consult with the patient’s physician
about stopping the Coumadin for a few days. Options (a) and (e)
might be appropriate in an emergency situation, but that is not what
we are dealing with here. Option (c) would take longer to reduce
(PT) than just stopping the Coumadin. Finally, heparin is not a
Coumadin antagonist, but an anticoagulant in its own right.

519
Q

Each of the following drugs has a significant anti-inflammatory
property except one. Which one is the exception?
a. aspirin
b. cortisol
c. acetaminophen
d. ibuprofen
e. indomethacin

A

c

520
Q

A patient has a history of significant cardiovascular impairment. The
maximum safe dose of epinephrine that can be administered to this
patient is:
a. 1 cc, 1: 50,000
b. 2 cc, 1: 50,000
c. 1 cc, 1: 100,000
d. 2 cc, 1: 100,000

A

(b) the rule is ..04 mg of epi max in CV patients. The easiest way to
figure this one out is to remember you shouldn’t give more than
2.2 carpules of xylocaine with 1:100,000 epi - your usual choice
as a local anesthetic. Two carpules is 3.6 cc. This eliminates
options (c) and (d), since they would be safe but not maximal.
Since 1:50,000 is twice as concentrated as 1:100,000, 1 cc of
1:50,000 (option (a)) is the same as 2 cc of 1:100,000 so still not
close to max, so (b) has to be the right answer. Of course, you
could have just remembered, 1:100,000 equals .01 mg per cc, so
1:50,000 equals .02 mg per cc, so (b) would equal the max of .04
mg.

521
Q

Which of the following is the current drug of choice for status-
epilepticus?

a. diazepam (Valium)
b. phenytoin (Dilantin)
c. chlorpromazine (Thorazine)
d. carbamazepine (Tegretol)
e. chlordiazepoxide (Librium)

A

(a) memorize, memorize. Most are anticonvulsants, but diazepam is
drug of choice. Chlorpromazine is an antipsychotic, not an
anticonvulsant. Phenytoin, is an anticonvulsant, and is one of the
answers to the question “which of the following causes gingival
hyperplasia?” Don’t forget diazepam is also given for antianxiety
and sedation.

522
Q
83. Which of the following is an example of an enteral route of
administration?
a. oral
b. submucosal
c. inhalation
d. subcutaneous
e. intramuscular
A

a

523
Q

Propranolol (Inderal) exerts its major antianginal effect by

a. dilating coronary arteries
b. dilating systemic blood vessels
c. increasing cardiac contractility
d. stimulating vagal slowing of the heart
e. blocking beta-adrenergic receptors of the heart

A

e

524
Q
Bradycardia is MOST commonly treated with which of the following
drugs?
a. atropine
b. epinephrine
c. a diuretic
d. a potent vasodilator
A

(a) bradycardia is a reflexive slowed heart rate, controlled by vagal input to the heart, and is cholinergically mediated, which means
you need a cholinergic receptor blocker to reduce the vagal
effect. Atropine is the only drug listed which is a cholinergic
agent. They threw in epinephrine as a tease, because if you
knew bradycardia was cardiac slowing, you might be tempted to
think epi, which usually speeds up the heart, would be the right
answer- but you’ve got to block the vagal input, epi won’t work.

525
Q

Which of the following best explains why drugs that are highly
ionized tend to be more rapidly excreted than those that are less
ionized? The highly ionized are
a. less lipid soluble
b. less water soluble
c. more rapidly metabolized
d. more extensively bound to tissue

A

(a) nonionized forms of drugs cross membranes more readily and
are highly lipid soluble, and tend to get stored in fat tissue from
where they are only slowly released. Thus highly ionized drugs,
which are less lipid soluble don’t get stored in fatty tissue and are
subject to more rapid excretion.

526
Q
Which of the following groups of drugs is CONTRAINDICATED for
patients who have glaucoma?
a. adrenergic
b. cholinergic
c. anticholinergic
d. adrenergic blocking
A

c

527
Q

Low dose aspirin therapy prevents the formation of thromboemboli by
preferentially inhibiting which of the following?
a. phospholipase A2 in the blood vessel walls
b. prostacyclin synthetase in the blood vessel walls
c. thromboxane synthetase in the platelets
d. vitamin K in the liver

A

(c) platelet aggregation is controlled by two factors, prostacyclin,
which decreases it, and thromboxane, which enhances it. Low
dose aspirin blocks the latter, so (c) is right, (a) is the mechanism
of action of corticosteroid drugs, (d) is the mechanism of action of
anticoagulant drugs like coumadin.

528
Q

A patient who is receiving an IV diazepam sedation has upper eye-lid
ptosis (Verill’s sign). The dentist should
a. assist respiration immediately
b. consider the patient to be adequately sedated
c. place the patient in the Trendelenberg position
d. administer one more increment of diazepam and proceed with the
treatment

A

b

529
Q

A dentist is considering the use of nitrous-oxide conscious sedation
for a patient. However, this type of sedation will be
CONTRAINDICATED, should the patient have a history of which of the
following?
a. dental anxiety
b. psychotic care
c. controlled hypertension

A

b

530
Q

The correct total liter flow of nitrous oxide- oxygen is determined by
a. a standard 6 liter per minute flow
b. the patient’s metabolic oxygen requirements
c. the amount necessary to keep the reservoir bag 1/3 to 2/3
full
d. the largest volume that the patient can exchange within one
minunte

A

c

531
Q
Which of the following sympathomimetic agents is the most potent
bronchodilator?
a. amphetamine
b. norepinephrine
c. phenylephrine
d. isoproterenol
e. methoxamine
A

(d) for bronchodilation you want a potent beta-2 adrenergic receptor

agonist- of the drugs listed (d) is the best. Isoproterenol is a non-
specific beta agonist. Methoxamine and phenylephrine are alpha-
1 agonists, amphetamine is an indirect acting agonist that causes

the release of norepinephrine, which is less potent at beta
receptors than isoproterenol.

532
Q

Succinylcholine blocks neuromuscular transmission by

a. inhibiting cholinesterase
b. inhibiting the central nervous system
c. depolarizing the motor endplate of skeletal muscle
d. inducing the formation of cholinesterase at the endplate
e. blocking release of acetylcholine at the endplate

A

c
Neuromuscular transmission requires the
action of acetylcholine at the nicotinic receptors on the
neuromuscular junction endplate. The two drugs used clinically to
do this are curare and succinylcholine. Curare is a nicotinic
receptor blocker, succinylcholine acts to overstimulate the
receptor, thereby causing its subsequent depolarization of the
neuron and a block of nuscle activity. (e) is how spider venoms
and snake toxins work, not succinylcholine. (a) would enhance
neuromuscular action, and is actually made use of clinically with
myasthenia gravis patients. (d) isn’t possible.

533
Q

Which of the following effects are common to pentobarbital,
diazepam, and meperidine?
a. anticonvulsant and hypnotic
b. analgesia and relief of anxiety
c. sedation and ability to produce dependence
d. amnesia and skeletal muscle relaxation

A

(c) the only definition which covers all three drugs. (a) applies only
to pentobarbital, (b) applies only to meperidine, and (d) applies
only to diazepam.

534
Q

(c) the only definition which covers all three drugs. (a) applies only
to pentobarbital, (b) applies only to meperidine, and (d) applies
only to diazepam.

A

b

535
Q

Injection of a local anesthetic into an inflamed area usually has a less
than optimal result. Which of the following best explains why?
a. the prostaglandins stabilize the nerve membrane
b. inflammation reduces the availability of the free base
c. the drug will be absorbed more rapidly because of the increased
blood supply
d. the chemical mediators of inflammation will present a chemical
antagonism to the anesthetic

A

b

536
Q
The major effect of a drug is produced by the amount of the drug
that is
a. free in plasma
b. excreted by the kidney
c. detoxified in the liver
d. bound to plasma protein
A

a

537
Q

A 4 yr old child is shy, timid, and fearful. Which of the following will
be MOST appropriate for the restorative appointments for this child?
a. Naloxone
b. Nitrous oxide/oxygen
c. Promethazine
d. Hydroxyzine hydrochloride (atarax) or hydroxyzine pamoate
(Vistaril) in divided doses
e. Meperidine (Demerol), promethazine (Phenergan) and
chlorpromazine (Thorazine) combined

A

b

538
Q

A primary advantage of intravenous sedation is which of the
following?
a. fewer side effects from the sedation
b. slower biotransformation for prolonged action
c. the ability to titrate individualized dosage
d. a smooth and more gradual onset of sedation

A

c

539
Q

Each of the following is true regarding drug biotransformation
EXCEPT one. Which one is the EXCEPTION?
a. the rate may differ significantly in various animal species
b. it primarily occurs in the liver microsomal enzyme system
c. it usually converts a drug to its lipid soluble, nonionized form
d. it generally involves alterations of the chemical structure of the
drug

A

c

540
Q
Which of the following drugs causes the LEAST CNS depression
and impairment of psychomotor skills?
a. diazepam
b. buspirone
c. alprazolam
d. chloral hydrate
A

b
Buspirone is used to relieve anxiety – it is
not a benzodiazepine like diazepam and alprazolam (BDZs may
not be that sedating, but can cause impairment of psychomotor
skills), and not a strong sedative like chloral hydrate

541
Q

A patient who has which of the following conditions is most likely to
have postoperative bleeding after multiple extractions?
a. angina
b. diabetes
c. cirrhosis
d. rheumatic fever
e. chronic bronchitis

A

(c) vitamin K is stored in the liver – this is decreased by cirrhosis – the
result is deficiencies in prothrombin dependent coagulation
factors

542
Q

Local anesthetics aid in reducing the flow of saliva during operative
procedures by
a. blocking the cholinergic nerve endings
b. blocking innervation to major salivary glands
c. blocking efferent parasympathetic nerve pathways
d. reducing sensitivity and anxiety during tooth preparation

A

(d) locals don’t have actions (a), (b), or (c)! Nervous patients, however,
do salivate more.

543
Q
49. Which of the following combinations of agents would be necessary to
block the cardiovascular effects produced by the injection of a
sympathomimetic drug?
a. atropine and prazosin
b. atropine and propranolol
c. prazosin and propranolol
d. phenoxybenzamine and curare
e. amphetamine and propranolol
A

(c) a sympathomimetic drug is going to potentially activate both
alpha and beta receptors, so you would need a pairing of
drugs which blocks those receptors. Prazosin, an alpha-1
blocker,, and propranolol, a non-specific beta-blocker, are
the only pair that block both types sympathetic/adrenergic
receptors. Atropine is a muscarinic blocker (anticholinergic),
phenoxybenzamine is an alpha-blocker, but curare is neuromuscular junction blocker. Amphetamine is a
sympathomimetic drug, not a blocker or sympatholytic drug. a

544
Q

When compared therapeutically to penicillin G, penicillin V has a

a. slower renal excretion
b. more reliable oral absorption
c. broader antibacterial spectrum
d. greater resistance to penicillinase
e. lower potential for allergic reaction

A

b

545
Q

Each of the following is a common side effect of prolonged
tetracycline therapy EXCEPT one. Which one is the EXCEPTION?
a. diarrhea
b. superinfection
c. photosensitivity
d. visual disturbance
e. discoloration of newly forming teeth

A

d

546
Q
To reduce a patient’s salivary flow, a dentist has prescribed
atropine. As a result of this medication, the patient might experience
which of the following side effects?
a. sedation
b. diarrhea
c. bradycardia
d. blurred vision
e. stomach cramping
A

(d) because it will also cause the anticholinergic effect of

mydriasis, or papillary dilation

547
Q
Of the following local anesthetics, which has intrinsic
vasoconstrictive actions?
a. cocaine
b. procaine
c. xylocaine
d. bupivacaine
A

a

548
Q

Which of the following antibiotics is found at much higher
concentrations in crevicular fluid than in serum?
a. clindamycin
b. penicillin
c. metronidazole
d. tetracycline

A

d

549
Q

When administered as oral centrally acting analgesics, which of the
following is considered to have the highest dependence liability?
a. codeine
b. oxycodone (in Percodan)
c. propoxyphene (Darvon)
d. pentazocine (Talwin)

A

(b) memorize that list of opioid analgesic potencies. It would go
propoxyphene< codeine< pentazocine< oxycodone

550
Q

opioid potencies

A

propoxyphene < codeine < pentazocine < oxycodone

551
Q

Antibiotics are useful in the treatment of which of the following?

a. herpangina
b. angina pectoris
c. recurrent apthous stomatitis
d. necrotizing ulcerative gingivitis

A

d

552
Q

Auditory nerve deafness is associated with the use of

a. polymixin B
b. chloramphenicol
c. amphotericin B
d. gentamycin

A

(d) each one of these drugs has a side effect that they may use
in the stem of this question, and give you the same list of
drugs: the effects are polymixin- renal necrosis,
chloramphenicol- bone marrow depression, amphotericin B
–nephrotoxicity

553
Q

amphotericin toxicity

A

renal

554
Q

chloramphenicol toxicity

A

bone marrow

555
Q

polymixin toxicity

A

rental necrosis

556
Q

gentamycin toxicity

A

auditory nerve deafness

streptomycin too

557
Q
The maximal or ceiling effect of a drug is also correctly referred to as
the drug’s
a. agonism
b. potency
c. efficacy
d. specificity
A

c

558
Q
Which of the following agents found in tobacco products cause
addiction?
a. tar
b. formaldehyde
c. nicotine
d. carbon monoxide
A

c

559
Q

Allergic reactions to local anesthetics are caused by

a. rapid absorption
b. slow detoxification
c. an antigen-antibody reaction
d. improper administration technique

A

c

560
Q

Which drug group is the LEAST likely to cause xerostomia?

a. opioids
b. antidepressants
c. antihistamines
d. benzodiazepines
e. anticholinergics

A

(a) xerostomia is an anticholinergic effect. All the drugs listed
except opioids are unfortunately strongly anticholinergic, in
addition to their desired mechanism of action.

561
Q
Currently, the BEST oral sedative drugs for dentistry fall into the
class of
a. narcotics
b. barbiturates
c. phenothiazines
d. benzodiazepines
A

(d) BDZs are better than narcotics or barbiturates because they
don’t cause anywhere near the problem with respiratory
depression that those two do.. Phenothiazines have more
side effects.

562
Q
Which of the following best describes the drug-receptor activity of
naloxone?
Affinity Intrinsic Activity
a. high high
b. low high
c. high none
d. none low
A

c
“pure”
competitive receptor antagonist – such a drug binds to a
receptor with high affinity, but has no intrinsic activity. If it did
it would be an agonist.

563
Q

Which of the following types of chemical bonding is the least likely
to be involved in a drug-receptor interaction?
a. covalent bonding
b. hydrogen bonding
c. dipole-dipole bonding
d. electrostatic bonding
e. van der Waal’s forces

A

(a) why would you care which one is the least likely? Actually,
because if it binds covalently, that is a non-reversible
situation that can’t be overcome – you get a long-lasting
drug effect that can only be overcome by making new
receptors. Some antagonists work this way.

564
Q

After a threshold stimulus, the cell membrane becomes permanently
altered. The liberation of which of the following transmitter
substances causes this alteration?
a. acetylcholine
b. cholinesterase
c. hydroxycholine
d. acetylsalicylic acid

A

(a) easy – the only “transmitter” listed is acetylcholine. You don’t
have to have any idea what the question is about (I don’t!)
as long as you recognize that there is only one transmitter
listed – the others kinda sound the same – cholinesterase is
the enzyme that breaks acetylcholine down, acetylsalicylic
acid is aspirin, have no idea what hydroxycholine is

565
Q

Which of the following compounds is used as an antiviral agent?

a. amantadine (Symmetrel)
b. novobiocin
c. miconazole (monistat)
d. amphotericin B

A

(a) miconazole and amphotericin B you should recognize, I hope,
as antifungals, so your choice is between (a) and (b).
Novobiocin sounds like an antibiotic, which it actually is, so
guess (a)

566
Q
With an overdose of a cholinergic drug, one would expect to see each
of the following signs EXCEPT one. Which one is the
EXCEPTION?
a. sweating
b. urination
c. mydriasis
d. bradycardia
e. copious serous saliva
A

(c) mydriasis is papillary dilation, which is a hallmark effect of
anticholinergic drugs, and why they are used in eye exams

567
Q

Cephalosporins are definitely CONTRAINDICATED for penicillin-
allergic patients who exhibit

a. immediate-type reactions
b. nausea and vomiting with erythromycin
c. any type of reaction to the penicillins

A

a

568
Q
Which of the following is a beta-adrenergic receptor blocking agent
used for the treatment of hypertension?
a. prazosin (Minipress)
b. clonidine (Catapress)
c. atenolol (Tenormin)
d. hydralazine (Aprezoline)
e. verapamil (Calan)
A

c
(a) is an alpha-1 blocker, (b) alpha-2 agonist that
decreases sympathetic outfloe from CNS, (d) is a direct
acting vasodilator, and (e) is a type I calcium channel
blocker

569
Q
In which of the following categories are ephedrine, tyramine, and
amphetamine classified?
a. anticholinesterases
b. alpha-adrenergic blocking agents
c. indirect-acting sympathomimetics
d. direct-acting parasympathomimetics
A

c

570
Q
57. Thiazides, which are used in the treatment of hypertension, may
require supplemental administration of
a. sodium
b. chloride
c. calcium
d. potassium
A

d

571
Q
Which of the following adverse reactions of oral contraceptives is the
most common and the most serious?
a. hypotension
b. hepatotoxicity
c. uterine neoplasia
d. thromboembolic disorder
e. decreased resistance to infection
A

d

572
Q

The supraspinal analgesic activity of morphine is mediated primarily
through its influence upon which opioid receptor subtype?
a. mu
b. kappa
c. delta
d. sigma
e. epsilon

A

(a) these are all opioid receptors – only mu and kappa are
involved in analgesia, mu at the supraspinal level and kappa
at the spinal level (the receptor that pentazocine (Talwin),
the mixed acting agonist/antagonist acts on

573
Q

Which pair of anesthetics is most likely to show cross-allergenicity?

a. lidocaine-mepivacaine
b. prilocaine-tetracaine
c. procaine-mepivacaine
d. procaine-lidocaine
e. lidocaine-benzocaine

A

a
procaine, benzocaine and tetracaine are
esters, lidocaine, prilocaine and mepivacaine are amides.
Amides, unlike esters are not by rule cross-allergenic

574
Q

Thrombophlebitis, which occurs after intravenous administration of
diazepam, is usually attributed to which of the following substances
in the mixture?
a. benzoic acid
b. ethyl alcohol
c. propylene glycol
d. sodium metabisulfite

A

c

575
Q

Each of the following is a pharmacologic effect of phenothiazines
EXCEPT one. Which one is the EXCEPTION?
a. sedation
b. an antiemetic effect
c. alpha-adrenergic effect
d. potentiation of the action of narcotics
e. an anticonvulsant

A

e
Actions a, c, and d are all clinically
useful actions of the phenothiazines, which you might
remember were discussed under the category of
antipsychotic drugs. But wait- Promethazine (Phenergan) is
used in dentistry as a sedative, often in combination with
Demerol because it reduces the nausea associated with the
use of the opioid and also potentiates its analgesic effect,
allowing lower dose to be used, thus again reducing the
potential for adverse side effects. Alpha adrenergic effects
are an adverse side effect.

576
Q

Epinephrine antagonizes the effects of histamine by

a. preventing the release of histamine
b. acting on the central nervous system
c. producing physiologic actions opposite to that of histamine
d. competitively blocking histamine at the cellular receptor site

A

c

577
Q

Which of the following represents the drug-of-choice in the
treatment of candidiasis for an HIV-infected patient?
a. acyclovir
b. nystatin
c. AZT
d. chlorhexidine

A

b

578
Q

A patient presents for treatment of a large fluctuant mass in the
submandibular space as a result of extension of odontogenic
infection. He has a temperature of 38.5 degrees C (101 degrees F).
Initially, the dentist should treat this patient with which of the
following?
a. salicylate therapy to reduce the temperature
b. alternate application of heat and cold to the area to improve
circulation
c. incision and drainage and a culture for antibiotic sensitivity
d. antibiotic therapy to reduce the swelling and infection

A

c

579
Q
The only local anesthetic that increases the pressor activity of both
epinephrine and norepinephrine is
a. cocaine
b. procaine
c. dibucaine
d. lidocaine
e. mepivacaine
A

a
cocaine is an
indirect acting adrenergic agonist, acting by causing the
release of adrenergic neurotransmitters as well as blocking
their reuptake, thereby prolonging their activity. The other
drugs listed are just local anesthetics that don’t have this
action - they just block sodium influx into the neuron.

580
Q

Corticosteroid therapy for arthritis is contraindicated for a patient who
also has which of the following conditions?
a. anemia
b. nephritis
c. alcoholism
d. peptic ulcer
e. rheumatic heart disease

A

(d) Patients using corticosteroids for arthritis often develop ulcers
because these drug block prostaglandin action in the
stomach, thereby increasing acid secretion while
decreasing the protective mucosal barrier of the stomach.

581
Q

Which of the following is the first symptom that is usually perceived
by the patient being administered nitrous oxide?
a. nausea
b. euphoria
c. giddiness
d. tingling of the hands

A

d
the first three are all signs that the patient is getting too much
nitrous.

582
Q

Each of the following, EXCEPT one, is a good reason for using
sedation. Which one is this EXCEPTION?
a. to allay apprehension, anxiety or fear
b. to decrease the amount of local anesthesia that is required for a
given procedure
c. to alleviate stress in a severely medically compromised patient
d. to accomplish certain procedures that a practitioner would not
normally be able to do on an anxious patient

A

(b) sedation does not decrease the local anesthetic requirement,
the patient will still feel pain. Sedatives, with the exception of
nitrous oxide, have no analgesic effect -barbiturate
sedatives may make the patient more sensitive to pain.

583
Q

Which of the following is classified as an antianxiety drug?

a. methohexital
b. lorazepam
c. haloperidol
d. pentazocine
e. phenylpropanolamine

A

(b) only benzodiazepines are actually classified as antianxiety
drugs, although other drugs, such as opioids have
antianxiety actions in addition to their other clinically useful
actions. (b) lorazepam is the only BDZ listed. Methohexital is a barbiturate - these are classified as seatives.
Haloperidol is an antipsychotic, pentazocine an opioid, and
phenylpropanolamine is a decongestant used in cold
medications.

584
Q

Alchol abuse occurs when someone
A. drinks more than the legal limit of alcohol
B/ experiences withdrawal symptoms when not drinking
C. gets seriously intoxicated on a regular basis
D. drinks enough that it puts their relationship, job, or health in jeopardy

A

d

585
Q
For oral sedation in the dental settingm the most ideal group of agents is
A. narcotics
B. barbiturates
C. antihistamines
D. benzodiazepines
E. anticholinergics
A

d

586
Q

In most cases, which of the following is the accpeted dose schedule of penicillin in the treatment of actinomycosis of the jaw
A. 250 mg oral qid 7 days
500 mg oeal qid 7 days
600,000 units IM bid for two weeks followed by oral medication
19 million units IV per day for 10-14 days followed by long-term oral medication

A

d

587
Q

which indicator is watched in coumadin patients

A

INR

588
Q

vit K dependent factors

A

2, 7, 9, 10, antociagulant proteins C and S

589
Q

what does warfarin inhibit

A

vit K reductase

590
Q

heparin mechanism

A

anticoagulant

binds reversibly to anti-thrombin II, prevents conversion of fibrinogen to fibrin

591
Q

dicoumarol mechanism

A

inhibits vit K reductase

592
Q

hemophilia factors

A

A - factor 8

B - factor 9

593
Q

Pt is taking warfarin (Coumadin), what test do you run prior to extraction or surgery:

A

INR (= 2.0-3.0)

594
Q

What is the best way to test clotting function on a patient taking Warfarin?

A

INR

595
Q

Patient is taking warfarin, what could you do, INR < 2.5?

A

Proceed with treatment because his INR is < 2.5

1 is normal (12 seconds)

higher INR – more bleeding, higher PT

596
Q

What INR is OKAY to place implant? 2.5, 3.5, etc

A
  1. 5
    - Bleeding measurements: PTT 25-36 sec PT 5-7 sec platelets 150K-450K minimum platelets 50k bleeding time: less than 9 min INR: 1

do
not treat with more than 3.5

597
Q

Patient is on Coumadin, what do you do prior to extractions?

a. Stop for 1 day
b. Stop medication of 5 days
c. Do not need to stop medication

A

b. Stop medication of 5 days (stop drug 5 days before, and resume the day after surgery)

598
Q

How does warfarin work on anti-coagulation (MOA)?

A

Decrease K+ needed to synthesize factors II, VII, IX, X

599
Q
The most important anti-coagulant effect of heparin is to interfere with the conversion of
A. PTA to PTC.
B. PTC to Factor VIII.
C. fibrinogen to fibrin.
D. prothrombin to thrombin.
E. proaccelerin to accelerin.
A

C. fibrinogen to fibrin.

600
Q

Pt taking dicoumarol (Vit K antagonist) is probably treated for?

A

Coronary infarct

601
Q

QUESTION: Pt is taking dicumarol, what are they being treated for?

A

Myocardial infarction

602
Q

Coumadin (warfarin):

A

give vitamin KKKKKKKKKK

603
Q

Alcoholic patient comes in for extraction? order:

A

PT/INR

604
Q

Severe alcoholic now recovering needs 24 tooth extraction, which tests are needed? INR, CBC

A

INR

605
Q

Alcoholic patient is about to undergo surgery. Which blood work test is most important?

  • creatinine
  • PT
  • PTT
  • Bleeding time
A
  • PT –> extrinsic system (Vit. K coagulation factors-2,7,9,10); used to test warfarin/Coumadin effectiveness, for liver damage, and Vit. K
    status

PTT –> intrinsic system, iused to test Heparin

606
Q

pt on heparin, which test to order

A

PTT

607
Q
Accurate way to detect blood alcohol in the body except
liver glucuronidation
weight
amount of food in stomach 
percentage of alcohol in drink
how fast you drank it
A

liver glucouronidation

608
Q

Best way to determine platelet function:

a. platelet count
b. bleeding time
c. PTT
d. INR

A

b. bleeding time

609
Q

Aspirin burn is due to:

A

coagulation necrosis.

610
Q

What does aspirin affects? Extrinsic, intrinsic, bleeding time, common pathway

A

bleeding time

611
Q

Aspirin ____ platelet function.

A

decrease

612
Q

What determines the bleeding time? Intrinsic, extrinsic, platelet adherence, common pathway

A

platelet adherence

Bleeding time = time required for blood to stop (2-6min normal)
- Bleeding time is increased in disorders of platelet count, uremia, and ingestion of aspirin and other anti-inflammatory medication

613
Q

aspirin effect on PT, PTT, INR

A

none

it affects plts and bleeding time

614
Q

normal bleeding time

A

2-6 minutes

615
Q
Aspirin is CONTRAINDICATED with which of the following drugs?
A. Coumarin (Coumadin®)
B. Triazolam (Halcion®)
C. Barbiturates (Phenobarbital®)
D. Pentobarbital (Nembutal®)
E. Methylprednisolone (Medrol®)
A

A. Coumarin (Coumadin®)

616
Q

Clopidogrel (Plavix) and aspirin:

A

alter platelet function, inhibits platelet aggregation irreversibly

617
Q

What affect does Plavix has? I

A

nhibits platelet aggregation

Given to patients allergic to aspirin àno ulcer side effect, given to patients with past ulcer history

618
Q

Prostaglandin inhibitor on gastric acid secretion?

A

PG decrease gastric acid and increase gastric mucous. Inhibiting PG will increase gastric acid and decrease mucosa. That’s why people
taking too much aspirin can get stomach bleeding cause more acidic and no protection

619
Q

What makes prostaglandin:

A

Arachidonic acid

620
Q

ginseng drug interactions

A

Ginseng is an antiplatelet (interferes with coagulation – not given with aspirin). pt on warfarin, aspirin

621
Q

Pt takes ginseng for energy, but it will interfere with

A

ASPIRIN (not digitalis)

- Ginseng = antiplatelet

622
Q

Pt is taking ginseng, what do you want to avoid?

A

Warfarin, NSAIDS, and Aspirin

623
Q

Pt taking ginseng. Which med should be avoided?

  • Penicillin
  • Aspirin
  • Digitoxin
A

Aspirin

624
Q

Pt. is taking saw palmetto, what do you want to avoid?

A

Aspirin

- Saw palmetto enhances anti-coagulants

625
Q

HERBAL supplement that potentiates anti-coagulation

a. St. John’s Wort
b. Saw Palmetto
c. Chamomile
d. Licorice

A

b. Saw Palmetto

626
Q

normal hematocrit

A

45 for males, 40 for females

if lower, consult physician for extractions

627
Q

Extractions for a pt with an INR of 2. what should you do?

a. extract, use sutures, hemostatic agents
b. gets pt off Coumadin for 2 days before extraction
c. Continue treatment

A

c. Continue treatment

- Mosby’s states that normal INR of people on anticoagulants is 2.5-3.0.

628
Q

Patient had extraction and socket is still bleeding 5 hours later?

A

Refer for INR

629
Q

Tooth extraction. 3 days later, area starts to hemorrhage, what is the cause?

A

Fibrinolysis

630
Q

Factor VIII is hemophilia

A

A

631
Q

The drug contraindicated in pt taking gingko biloba:

A

HEPARIN

632
Q

Overweight patient that has to piss twice (x2) at night. What condition?

A

Diabetes

633
Q

Diabetes is more common:

A

black men

634
Q

Hb1Ac: measuring

A

glucose level over extended period

635
Q

glucose in urine monitored?

A

no

636
Q

Pt who took too much insulin will have all except?

A

Hyperglycemia

637
Q

Decrease in ⬇ glycogenosis in the liver would be expected with

A

insulin

638
Q

Sign of hypoglycemia –

A

bradycardia, mydriasis (pupil dilation), diaphoresis (sweating), mental confusion

639
Q

Pt appears disorientated & hypoglycemic

A

administer glucose

640
Q
A

alcohol
risk factors for the development of hypoglycemia: exercise, alcohol, older age, renal dysfunction, infection, decreased intake of energy,
and mental health issues, including dementia, depression, and psychiatric illnesses.

641
Q

Ketone breath –>

A

Diabetes type 1/hyperglycemic

642
Q

Ketone breath & altered state of consciousness?

A

Hyperglycemia

643
Q

Type I Diabetes leads: a) Aphasia b) Ataxia c) Blindness d) Deafness

A

c) Blindness

644
Q

Common complication of Type I diabetes:

A

blindness (retinopathy)

645
Q

2nd generation Sulfonylureas:

A

enhance insulin secretion (glyburide, glipizide, glimepiride)

646
Q

Biguanides:

A

produces less glucose in the liver (metformin)

647
Q

Alpha glucosidase inhibitors:

A

slow carbohydrate digestion (acarbose, miglitol)

648
Q

Insulins by time:

A

Rapid acting (5-15min); Short acting (30-60min); Intermediate (2-4h); Long acting (6-10h); Premixed (30-60min)

649
Q

Glucocorticoids are contraindicated in:

A

Diabetes
- steroids can raise blood sugar & increase the need for more medication to control sugar levels. Diabetics on steroids may have to raise
their insulin dose dramatically.

650
Q

Oral hypoglycemic drug for diabetes?

A

sulfonylurea & metformin

651
Q

Why don’t you give sulfonylureas to Type I diabetic patients?

A

They do not have beta cells for insulin & Sulfonylureas MoA is to
stimulate those cells

652
Q

How do Sulfonylureas work? Stimulate insulin release from Beta cells in the pancreas, stimulate binding, decrease glucagon levels.

A

Stimulate insulin release from Beta cells in the pancreas

653
Q

MOA of sulfonylurea-

A

increase insulin PRODUCTION and SENSITIVITY by Beta cells stimulation by binding to ATP-dependent K
channels
- stimulating pancreatic insulin release by bind to ATP K+ channels & causing depolarization, which stimulates calcium ion influx & induces
insulin secretion.

654
Q

Metformin MOA

A

suppresses glucose production in liver (decreasing hepatic gluconeogenesis) à decreases glucagon levels) – bind to
AMP protein kinase receptors

655
Q

Proposed modes of action for the oral antidiabetic agents include each of the following EXCEPT one. Which one is the EXCEPTION?
A. Blockade of glucagon release from pancreas
B. Blockade of catecholamine release from adrenal medulla
C. Stimulation of insulin release from pancreatic beta cells
D. Action as direct receptor agonists for the insulin receptor
E. Increase affinity of tissues for utilization of available plasma glucose

A

D. Action as direct receptor agonists for the insulin receptor

656
Q

How are the various preparations for diabetic drugs classified by? by duration of action, MOA, etc

A

duration of action

657
Q

Patient with orthopnea (shortness of breath-dyspnea-while lying flat), dyspnea, pedal edema

a. Emphysema
b. Pulmonary edema
c. COPD
d. Congestive heart failure

A

d. Congestive heart failure

658
Q

What is common symptom of CHF?

A

Orthopnea

- other symptoms: dyspnea, fatigue, paroxysmal nocturnal dyspnea, edema

659
Q

Most common reason for cardiac arrest of kid? Respiratory distress, congestive heart failure, cyanotic heart disease, etc

A

Respiratory distress

660
Q

What is the most common heart condition in children?

A

Ventricular septal defects

- communications between the bottom chambers, structural heart defects

661
Q

Peripheral edema, increase systole

A

à congestive heart failure

662
Q

Patient has distended jugulars, pitting edema and dyspnea?

A

Congestive heart failure

663
Q

Cardiac referred pain is not consistent with?

A

Pain that goes away with LA

664
Q

MI and arrhythmia difference? Thrombosis, arthrosclerosis

A

Thrombosis

665
Q

Patient has chest pain in heart region when sleeping or at rest, what kind of angina is it?

a. Pseudo-angina
b. Unstable angina
c. Infarction

A

b. Unstable angina

666
Q

Why is pt taking ACE inhibitor?

A

Hypertension / CHF

667
Q

Pt taking cardiac glycosides. What is it used for? hypertension, congestive heart failure, etc

A

congestive heart failure

668
Q

What do cardiac glycosides (ex. digitalis) do?

A

Inhibit Na/K ATPase & Increase Na and Ca in cell to increases the refractory period.

669
Q

How does digitalis works? blocks Na/K ATPase =

A

increase influx more Ca

670
Q

Digitalis -

A
glycoside
Increase Ionotropic (contractions) effect of the heart
671
Q

Use of digitalis:

A

Post myocardial infarction, Supraventricular arrhythmia

- digitalis/cardiac glycoside = common indications for use is for atrial fibrillation

672
Q

How does Digoxin work?

A

Inhibits Na/K ATPase of cardiac cell membranes resulting in increase of Na concentration intracellularly,
cardiac glycoside, increases intracellular Ca++

673
Q

Garlic contraindications:

A

contraceptives and anti-virals (HIV), caution with bleeding

674
Q

Pt has history of cardiovascular disease and now, pt is taking aspirin. Pt needs ext. What should dentist do?

  • Med consult with physician
  • Normal extraction
  • Stop aspirin 3 days before and 2 days after surgery
A

• Med consult with physician

675
Q

Mechanism of most drugs that tx arrhythmias?

A

Decreases repolarization rate, prolongs refractory period

676
Q

When you have atrial arrhythmia, what’s the mech of action for the drug for it?

A
  • you can give Quinidine, Verapamil, and Digitalis for atrial and the side mechanism of Quinidine is it increases the refractory period
677
Q

Side effect of nitroglycerin:

A

orthostatic hypotension and headache

678
Q

Transient ischemic attack (TIA), what is false? Better chance to get stroke, patient should take nitroglycerin

A

patient should take nitroglycerin FALSE-give for

angina to prevent heart attacks.

679
Q

Nitrates and nitriles have what systematic effect?

A

Vasodilation of arteries à decreased BP à tachycardia
- Nitroglycerin is a nitrovasodilator. It produces nitric oxide, which activates guanylyl cyclase which, in turn, catalyzes the production of ⬆
cGMP.

680
Q

Nitrates/Nitriles, how do they respond to angina?

A

through blood vessels (dilate blood vessels)

681
Q

How do nitrates work on the heart?

A

relaxing and widening the blood vessels in the body, allowing more blood and oxygen to flow to
the heart. Since the arteries are wider, it is easier for the heart to pump blood, so it does not require as much blood and oxygen.

682
Q

You give the nitroglycerin to the pt with angina and heart rate goes up, what’s the reason?

A

natural reflex to the decrease in blood

pressure

683
Q

Diuresis (excessive urine production) after tx of angina w/ a glycoside?

A

b/c of increased blood flow caused increased blood flow to

kidney

684
Q

Why give hydralazine with chloral hydrate?

A

Decreases nausea

Hydralazine (Apresoline) is a direct-acting smooth muscle relaxant used to treat HTN by acting as a vasodilator primarily in arteries and
arterioles to decrease peripheral resistance, thereby lowering blood pressure and decreasing afterload.

685
Q

Main prophylactic treatment for angina?

A

Propranolol

686
Q

Which is not used in tx of angina? Nitroglycerin, Ca+ blocker, propranolol, thiazide

A

thiazide

those are diuretics

687
Q

Quinidine treats?

A

SV arrhythmias

688
Q

Asthma causes constriction on bronchioles, constriction of smooth muscles & inflammation of bronchioles?

A

Beta 2 receptor for

lungs, Beta 1 receptor for heart

689
Q

What do asthmatic patients have problem with?

A

Wheezing when exhaling

- Wheezing à exhale with high pitch

690
Q

Child makes a wheezing sound before injection?

A

Asthma (induced by stress)

691
Q

COPD vs Asthma?

A

Asthma have problem breathing in (but wheeze when exhaling), COPD has problem exhaling

692
Q

What is the most common cause for breathing difficulty in the dental chair?
Hyperventilation
COPD
Asthma

A

Hyperventilation

693
Q

Most common respiratory emergency in dental office?

A

Hyperventilation

694
Q

Hyperventilation causes –

A

tachycardia and tachypnea (rapid breathing)

695
Q

Face swelling after air spray in perio pocket:

A

soft tissue emphysema (sudden painless swelling)

- Emphysema: constriction of air sacks

696
Q

Perio surgery, air into sulcus. What occurs?

A

subcutaneous emphysema

697
Q

Pt has emphysema. What are his symptoms?

A

Dyspnea, wheezing, cough, chest tightness. Air sacks are all destroyed (narrowing of
distal airways)

698
Q
Crowing sound when breathing (Stridor)?
• asthma attack
• COPD
• Pneumothorax
• laryngospasms
A

laryngospasms

699
Q

Stridor-

A

laryngospasm - blockage of UPPER resp. tract

700
Q

QUESTION: Epi for laryngiospasm, what does it do?

A

bronchodilater, increase HR,

increase BP

701
Q

Theophylline is used to

A

prevent and treat wheezing, SOB (shortness of breath), and difficulty breathing caused by asthma, chronic
bronchitis, emphysema, and other lung diseases. It relaxes and opens air passages in the lungs, making it easier to breathe.

702
Q

Most effective during acute asthma attack?

A

albuterol- generic name is Salbutamol. It is a beta-2 agonist, which causes
bronchodilation

703
Q

Pt has asthmatic attach, took albuterol, and it didn’t work. What’s next step?

  • Epinephrine
  • atropine
  • something else
A

Epinephrine

704
Q

Medication for severe asthma attack

A

Aminophiline (bronchodilator)

705
Q

What drug cause asthma?

A

Aspirin (NSAID) so NO NSAIDS for asthmatic patients

706
Q

Which of the following drugs is can trigger asthma?

a) narcotic analgesic
b) NSAID
c) corticosteroid
d) sympatolytic amine.

A

NSAID

707
Q

What is used for a severe bronchial asthma attack?

A

Albuterol, corticosteroids, aminophylline

708
Q

Long term asthma, give

A

corticosteroid

709
Q

Patient begins to wheeze, what do you not do?

o Beta-2 blocker inhaler
o sit pt up & make them more comfortable
o corticosteroid inhaler
o Give oxygen

A

o Give oxygen

710
Q

albuterol on mouth

A

dry

711
Q
5-month old pregnant woman with syncope, what position do you put her in?
supine with legs raise
reversed trendelburg
on her Right
on her left
A

on her left - to avoid compression of inferior vena cava

712
Q

QUESTION: If a 3rd trimester pt all of a sudden feels a drop in BP, what do you do?

A

Have pt lay on left side

713
Q

Prego question – syncope, which side you put pt?

A

Raise right hip up

- Baby crushing IVC so lay on left hip & raise right hip UP

714
Q

What causes pregnant woman to syncope?

A

Beware of compression to inferior vena ceva

715
Q
Pregnant in supine position, what gets too much pressure?
Fetus
Placenta
Inferior Vena Cava
Superior Vena Cava
A

Inferior Vena Cava

716
Q

Most important thing to do when patient syncope – maintain airway, loosen up buttons, place head below heart, supine

A

maintain airway

717
Q

Crown disappears down patient’s throat, what position do you put them in? Supine, Upright, Trendelberg

A

Upright

718
Q

Want to determine patient physiologic rest position, place in – supine, upright/standing, tredenlburg

A

upright/standing

719
Q

Purpose of the Trendelberg position is to?

A

maint circulation so that the most vital organs are never hypoxic.

720
Q

What position you place the Pt when is having syncope?

A

TRENDELENBURG POSITION

SUPINE WITH FEET ELEVATED SLIGHTLY
- The most common early sign of syncope is PALLOR (paleness).

721
Q

earliest sign of syncope

A

pallor

722
Q
A

Hydralazine (Apresoline) is a direct-acting smooth muscle relaxant used to treat HTN by acting as a vasodilator primarily in arteries and arterioles to decrease peripheral resistance, thereby lowering blood pressure and decreasing afterload.

723
Q

You walk to office, pt is unconscious. What position do you place the patient in? Supine, Tendenberg, upright

A

Tendenberg

724
Q

All forms of shock have?
• Hypovolemia
• Decreased perfusion to tissue
• Sepsis

A

Decreased perfusion to tissue

725
Q

Vasovagal syncope is a common cause of

A

transient loss of consciousness

726
Q

Syncope?

A

Inhale ammonia to irritates es. trigeminal nerve sensory, 100% oxygen works except with hyperventilation syndrome

727
Q

High-flow 100% 02 is indicated for treating each of the following types of syncope EXCEPT one. Which one is this EXCEPTION? A. Vasovagal
B. Neurogenic
C. Orthostatic
D. Hyperventilation syndrome

A

Hyperventilation syndrome

728
Q

Most common dental complication/emergency in office?

A

Syncope

729
Q

You gave local anesthetic, BP went up to 200/100 and HR went up too, what could be due to?

A

Due to vasoconstrictor injected into venous system.

730
Q

After receiving one cartridge of a local anesthetic, a healthy adult patient became unconscious in the dental chair. The occurrence of a brief convulsion is
A. pathognomonic of grand mal epilepsy.
B. consistent with a diagnosis of syncope.
C. usually caused by the epinephrine in the local anesthetic.
D. pathognomonic of intravascular injection of a local anesthetic.

A

consistent with a diagnosis of syncope.

731
Q

Signs of syncope:

A

blood pressure falls

732
Q

Signs of epi overdose:

A

blood pressure and heart rate rises

733
Q
A

Hyperventilation

Carpopedal spasms are severely painful cramps of the hand/feet muscles. - May be caused by low blood calcium levels or by tetanus.

734
Q

Most adverse effect of benzos

A

Respiratory depression

735
Q

CONTRAINDICATIONS for benzos

A

Pregnant, myasthenia gravis, acute narrow glaucoma, COPD,
emphysema

736
Q

Grand mal seizure med

A

Phenytoin/ Dilantin

737
Q

Status epilepticus

A

Valium / diazepam

738
Q

QUESTION: Most common seizure in children –

A

Febrile seizures, which occur in young children & are provoked by fever, are the most common type of provoked seizures in childhood.
Then, generalized tonic-clonic (grand mal)

739
Q
A

Hydralazine (Apresoline) is a direct-acting smooth muscle relaxant used to treat HTN by acting as a vasodilator primarily in arteries and arterioles to decrease peripheral resistance, thereby lowering blood pressure and decreasing afterload.

740
Q

You walk to office, pt is unconscious. What position do you place the patient in? Supine, Tendenberg, upright

A

Tendenberg

741
Q

All forms of shock have?
• Hypovolemia
• Decreased perfusion to tissue
• Sepsis

A

Decreased perfusion to tissue

742
Q

Vasovagal syncope is a common cause of

A

transient loss of consciousness

743
Q

Syncope?

A

Inhale ammonia to irritates es. trigeminal nerve sensory, 100% oxygen works except with hyperventilation syndrome

744
Q

High-flow 100% 02 is indicated for treating each of the following types of syncope EXCEPT one. Which one is this EXCEPTION? A. Vasovagal
B. Neurogenic
C. Orthostatic
D. Hyperventilation syndrome

A

Hyperventilation syndrome

745
Q

Most common dental complication/emergency in office?

A

Syncope

746
Q

You gave local anesthetic, BP went up to 200/100 and HR went up too, what could be due to?

A

Due to vasoconstrictor injected into venous system.

747
Q

After receiving one cartridge of a local anesthetic, a healthy adult patient became unconscious in the dental chair. The occurrence of a brief convulsion is
A. pathognomonic of grand mal epilepsy.
B. consistent with a diagnosis of syncope.
C. usually caused by the epinephrine in the local anesthetic.
D. pathognomonic of intravascular injection of a local anesthetic.

A

consistent with a diagnosis of syncope.

748
Q

Signs of syncope:

A

blood pressure falls

749
Q

Signs of epi overdose:

A

blood pressure and heart rate rises

750
Q
A

Hyperventilation

Carpopedal spasms are severely painful cramps of the hand/feet muscles. - May be caused by low blood calcium levels or by tetanus.

751
Q

Most adverse effect of benzos

A

Respiratory depression

752
Q

CONTRAINDICATIONS for benzos

A

Pregnant, myasthenia gravis, acute narrow glaucoma, COPD,
emphysema

753
Q

Grand mal seizure med

A

Phenytoin/ Dilantin

754
Q

Status epilepticus

A

Valium / diazepam

755
Q

QUESTION: Most common seizure in children –

A

grand mal seizures- Febrile seizures, which occur in young children & are provoked by fever, are the most common type of provoked seizures in childhood. Then, generalized tonic-clonic (grand mal)

756
Q
Which of the following is the current drug-of-choice for status epilepticus?
A. Diazepam (Valium®)
B. Phenytoin (Dilantin®)
C. Chlorpromazine (Thorazine®)
D. Carbamazepine (Tegretol®)
E. Chlordiazepoxide (Librium®)
A

A. Diazepam (Valium®)

757
Q

QUESTION: Drug of choice of status epilepticus (seizure that last for long period)?

A

Valium (diazapams) 5-10 mg IV / per minute

758
Q

QUESTION: Diazepam is contraindicated in the following patients?

A

CONTRAINDICATIONS: Pregnant, myasthenia gravis, acute narrow glaucoma, COPD,
emphysema

759
Q
Which of the following drugs, when administered intravenously, is LEAST likely to produce respiratory depression?
A. Fentanyl
B. Diazepam
C. Thiopental
D. Meperidine
E. Pentobarbital
A

B. Diazepam

760
Q

What drug for patient with petit mal seizures in dental office?

A

Ethosuximide
- Only 2 drugs for absence seizures (petit mal): ethosuximide (Zarontin) – only treats petit mal- and valproic acid (Depakene, Depacon) –
treats grand mal, petit, and myoclonic seizures.

761
Q

What causes/induce seizures?

a. Hyperkalemia
b. Hypophosphatasa
c. Hyponantremia (low sodium)
d. Hypernantremia.
e. Hypoglycemia

A

c. Hyponantremia (low sodium)

762
Q

Epileptic pt least likely to take:

a. Ethosuximide
b. Diazepam
c. Lasix (furosemide)

A

a. Ethosuximide – petit mal seizures
b. Diazepam - Status epilepticus
c. Lasix (furosemide) – HTN loop diuretic

763
Q

Each of the following is an advantage of midazolam over diazepam EXCEPT one. Which one is this EXCEPTION?
A. Less incident of thrombophlebitis
B. Shorter elimination half-life
C. No significant active metabolites
D. Less potential for respiratory depression
E. More rapid and predictable onset of action when given intramuscularly

A

D. Less potential for respiratory depression

  • midazolam has a milder effect, but more long lasting
764
Q
The clinical activity of a single intravenous dose (10 mg) of diazepam is most dependent on which of the following?
A. Alpha half-life
B. Betahalf-life
C. Renal excretion
D. Enzymatic degradation
E. Hepatic biotransformation
A

E. Hepatic biotransformation

765
Q
Each of the following are narcotics used in outpatient anesthesia EXCEPT one. Which one is this EXCEPTION?
A. Fentanyl
B. Sufentanil
C. Meperidine
D. Diazepam
E. Morphine
A

D. Diazepam

766
Q

Which of the following describes the titration of diazepam to Verrill’s sign for IV conscious sedation?
A. It is recommended as an end-point.
B. It is recommended only when supplemental 02 is used.
C. It is usually not attainable with diazepam alone.
D. It is not recommended since it can indicate a too-deeply sedated patient.
E. It is not recommended since few patients are adequately sedated at that level.

A

A. It is recommended as an end-point.

  • The most frequently used signs for IV diazepam sedation are ptosis, (“the Verrill sign”), altered speech and blurred vision.
767
Q
Which of the following is the treatment of choice for lidocaine-induced seizures?
Epinephrine (EpiPen ̈)
Naloxone (Narcan ̈)
Diazepam (Valium ̈)
Flumazenil (Romazicon ̈)
Succinylcholine (Anectine ̈)
A

Diazepam (Valium ̈)

768
Q

Benzodiazepines: ones not metabolized by the liver (safe to use in liver failure)

A

o LOT: Lorazepam, Oxazepam, Temazepam

769
Q

α-Hydroxylation is a rapid route of metabolism unique to these benzos”

A

triazolam, midazolam, and alprazolam à short sedative

770
Q

Long-acting barbiturate:

A

Phenobarbital is used to treat certain types of seizures

771
Q

reverse for zolpidem and zaleplon

A

these are for insomnia

reverse with flumazenil

772
Q

What’s the action of the Benzodiazepines?

A

Facilitates GABA receptor binding by Increasing the frequency of chloride channel
opening.

773
Q

Xanax MOA,

A

Mechanism of action of on GABA receptors: increasing the frequency of chloride channels by benzodiazepines
- Barbiturates increase the duration of chloride channel opening

774
Q

Benzodiazepines act on:

A

GABA receptors

775
Q

Which benzodiazepines is used for depression & anxiety for obsessive compulsive disorder?

A

Xanax (alprazolam)

776
Q

Diazepam (valium) use

A

Anti-convulsant & sedative

777
Q

Anticonvulsants can cause

A

cleft palate (teratogenic effect)

778
Q

valium for emesis?

A

no

779
Q

valium for insomnia?

A

no

780
Q

diazepam on respiration

A

no effect as opposed to other benzos

781
Q
Hypnosis affects what?
voluntary muscles
involuntary muscles
both voluntary and involuntary muscles
glands
A

voluntary muscles

782
Q

Which of the benzodiazepine don’t you give to elderly?

A

Long acting one (like diazepam)

- Short to intermediate-acting benzodiazepines are preferred in the elderly (ex. oxazepam, temazepam, midazolam)

783
Q

Benzodiazepines are great for dentistry due to an action of-

A

amnesia and little memory of the event.

784
Q

Best benzo for IV sedation –

A

MIDAZOLAM

785
Q

What does IV Midazolam do?

A

Amnesia

786
Q

Best benzodiazepine for pt with liver cirrhosis

A

Oxazepam

- LOT: Lorazepam, Oxazepam, Temazepam

787
Q

Which drug best reverses the effect of benzodiazepines?

A

Flumazenil

- Flumazenil: Benzodiazepine antagonist b/c competitive GABA receptor.

788
Q
The reversal for Versed? 
A. Narcon
B. Flumazenil
C. Naloxone 
D. Disulfuriam
A

(versed = midazolam)

Flumazenil

C. Naloxone (for opioids)
D. Disulfuriam (for alcoholics)

789
Q

Contraindication of lorazepam:

a) pregnancy
b) diabetes

A

a) pregnancy

790
Q

Benzodiazepines (diazepam, lorazepam) are contraindicated in

A

pregnancy

791
Q

How benzos are anxiolytic?

A

moderate doses ANTIANXIOLYTIC and high doses is SEDATIVE

792
Q

Sedative rebound –

A

Antipsychotic, part of withdrawal
- Several anxiolytics & hypnotics have a rebound effect, which cause severe anxiety and insomnia worse than the original insomnia or
anxiety disorder.

793
Q

Which of the barbiturates MOST readily penetrates the blood-brain barrier?

A

Thiopental

794
Q

Sodium Thiopental use:

A

rapid-onset short ultra-acting barbiturate(IV) for general anesthesia

795
Q

A patient has appointment next morning, he is anxious, and the night before he had hard time sleeping, which of the following tx
would you prescribe?

A

Ambien (sedative and makes patient sleep).

796
Q

Chief mechanism by which the body metabolizes short-acting barbiturates is?

a. oxidation
b. reduction.
c. hydroxylation and oxidation.
d. sequestration in the body fats.

A

a. oxidation (occurs in the liver)

797
Q
A patient's early recovery from an ultrashort-acting barbiturate is related primarily to
redistribution.
breakdown in the liver.
excretion in the urine.
breakdown in the blood.
binding to plasma proteins.
A

redistribution.

798
Q

Know the effects of histamine and that it is derived from

A

histidine

- Histidine decarboxylase (HDC) enzyme catalyzes the reaction that makes histamine from histidine w/ vitamin B6

799
Q

Benadryl (diphenhydramine) class -

A

1st generation anti-histamine - H1 blockers

800
Q

What is used for motion sickness?

A

Diphenhydramine (Benadryl)

801
Q

What does diphenhydramine (Benadryl) cause?

A

Xerostomia (anti-cholinergic, anti-histamine, sedative)

802
Q

What property of diphenhydramine causes xerostomia?

a. Anticholinergic
b. Antihistaminic
c. Antimuscarinic

A

Anticholinergic

803
Q

What property of topical diphenhydramine would alleviate pruritus (itching)?

A

Anti-histamine

  • antihistamine relieves itchy/watery eyes and itchy throat by blocking a substance (histamine) released by allergies.
  • anticholinergic dries up a runny nose & the fluid that runs down your throat causing itching/irritation.
804
Q

What anti-histaminic cause less drowsiness:

A

H1 blocker 2nd generation
- Allegra, Claritin (loratidine), Clarinex (Desloratidine) Certizine (Zyrtec) because they don’t cross BBB, poor CNS penetration

805
Q

Which one of these has the least sedative effect?
Diphenylhydramine/ Benadryl
chlorpheniramine
Tripelennamine

A

(2nd generation H1 blocker)

Diphenylhydramine/ Benadryl (Most)
chlorpheniramine (LEAST)
Tripelennamine
- Chlorphenamine, is a first-generation alkylamine antihistamine

806
Q

Which antihistamine is least likely to cause drowsiness?

A

Loratidine (Claritin)

807
Q

Claritin/loratidine class –

A

second generation H1 blocker/antihistamine

808
Q

Which of the following would have slowest onset after IV administration? Diphenyhydramine, loratadine,

A

loratadine

809
Q

What do you give to someone who is allergic to ester & amides LA?

A

DIPHENHYDRAMINE (BENADRYL)

810
Q

How does antihistamines work?

A

Competitive inhibition of histamine receptors

811
Q

Detailed mechanism questions on H1 (histamine) à

A

compete w/ histamine to bind at H1 receptor sites.

812
Q

Effects of H1 blocker EXCEPT:

a. CNS increase
b. CNS decrease
c. increase acid secretion
d. respiratory depression
e. local anesthesia

A

(causes CNS depression)

a. CNS increase

813
Q

Actions of H1 antagonist à

A

competitive inhibition of H1 receptors so block vasodilation, bronchoconstriction, and capillary
permeability à Vasoconstriction, bronchodilation, and decrease capillary permeability

814
Q

H2 antihistamine à

A

Cimetidine – decrease ulcers
- Cimetidine (Tagamet) is a histamine H2 receptor antagonist that inhibits stomach acid production & is used as an antacid.

815
Q

Histamine 2 blocker meds - f

A

or gastric reflux or GERD (gastric esophageal reflux disease) - Cimetidine & Ranitidine

816
Q

What do bradykinin do?

A

Dilate blood vessel & lower BP

817
Q
25 yo female breast feeding 12m old child and currently pregnant, which sedative would you give?
• Halcion
• Promethazine
• Nitrous
• Diazepam
• Phenobarbital
A

• Promethazine

818
Q

What anxiolytic to use for anxious 25-year-old pregnant woman who is breastfeeding? Chloral hydrate, nitrous,
benzo, promethazine

A

promethazine

others AVOID

819
Q
Which of these opioid analgesics is associated with a serious life threatening drug interaction when administered with an MAO
inhibitor?
Meperidine 
morphine
fentanyl propoxyphene
codeine
A

Meperidine (Pethidine, Demerol)

  • Can cause life-threatening hyperpyrexia reactions (fever)
820
Q

pt taking MAOis in case of head injury - opioids?

A

no!

821
Q

Opioids contraindicated in: severe head injury, renal insufficiency

A

severe head injury

822
Q

QUESTION: An opiate type MAA with both agonist and antagonist properties is-

A

pentazocine
Pentazocine is a synthetically-prepared prototypical mixed agonist–antagonist narcotic (opioid analgesic)
Another one is nalbuphine

823
Q

Pt is addicted to oxycodone which contra indicated? codeine, pentozocaine

A

pentozocaine

824
Q

Which of the following effects are common to pentobarbital, diazepam, and meperidine?
A. Anticonvulsant and hypnotic
B. Analgesia and relief of anxiety
C. Sedation and ability to produce dependence
D. Amnesia and skeletal muscle relaxation

A

C. Sedation and ability to produce dependence

825
Q

Absolute Contraindications to Opioid Prescribing:

A

Allergy to Codeine/Oxycodone/Hydrocodone

- give Methadone or Meperidine or Tramadol instead (Group 3 synthetic)

826
Q

synthetic opioids

A

tramadol, methadone, meperidine (demerol), propozyphene, fentanyl

827
Q

Miosis seen in opioid abuse - except with

A

meperidine (an exception)

828
Q

Use for sedation of children -

A

Secobarbitol or pentobarbital (good for pre-op/anxious kids)

  • Ketamine is used in emergency situations (good anxiolytic and analgesic at low doses)
  • Meperidine should not be used in kids
829
Q

Which is not done by opiates?

o Diuresis
o Constipation
o Bronchiolar constriction
o Vomiting

A

Diuresis (opiates cause urinary retention)

830
Q

Opioid usage shows all except: xerostomia, chronic cough, diarrhea, miosis, constipation

A

diarrhea

831
Q

Opioid overdose side effect –

A

constipation, respiratory depression, euphoria, miosis, coma

832
Q

Opioid OD symptoms – . irritability (restlessness), hypertension, insomnia, hypotension

A

hypotension

others are withdrawal

833
Q

Symptom seen in oral opioid overdose: hypothermia, headache, insomnia, irritability

A

hypothermia

rest are withdrawal symptoms

834
Q

Symptoms if too much codeine? Insomnia, Cold and Clammy skin, irritable.

A

Cold and Clammy skin

835
Q

What is the most significant side effect of morphine/opioids?

A

Respiratory depression

836
Q

Major disadvantage of opioids is

A

respiratory depression.

837
Q
Which of the following symptoms is the most distinct characteristic of morphine poisoning?
A. Comatose sleep
B. Pin-point pupils 
C. Depressed respiration
D. Deep, rapid respiration
E. Widely dilated, non-responsive pupils
A

B. Pin-point pupils

838
Q

If you give too much of an opioid (but it’s not an overdose!), what’s the first sign you would see?

A

constricted pupils and absent/slow breathing

839
Q

Opioid Receptors-

A

brain, spinal cord and digestive GI tract.

840
Q

Opioid cause stomach upset by acting on the

A

brain, not on stomach receptors!

841
Q

Opioid agonists act by:

a. stimulating GABAergic neuron
b. increase pain threshold
c. acting as Mu receptor agonists

A

c. acting as Mu receptor agonists

842
Q

Naloxone: use

A

for Opioid overdose.
- Naloxone blocks or reverses the effects of opioid medication, including extreme drowsiness, slowed breathing, or loss of consciousness

843
Q

Opioid (Fentanyl, Morphine, Meperidine, Methadone, Sulfentanil, Codeine, Heroin, Dextromethorphan) reversal drug?

A

Naloxone

844
Q

Antidote for Percodone overdose

A

(Oxycodone + aspirin)? all opiate antidote is Nalaxone.

845
Q

How does an antagonist work?

A

No intrinsic activity, High affinity
- Intrinsic activity (IA) or efficacy refers to the relative ability of a drug-receptor complex to produce a maximum functional response.

846
Q

Antagonist:

A

Binds to receptor but lacks intrinsic activity (doesn’t activate receptor)

847
Q

What opiate is part of the intradermal system?

A

Fentanyl

- Fentanyl = opioid analgesic given via transdermally patch

848
Q

Methadone helps

A

alleviate withdrawal from heroine (opiates).
- Methadone for detoxification of opioid addicts, give to heroin addicts to decrease withdrawal symptoms
- Methadone is a synthetic opioid, analgesic, antitussive, anti-addictive, acts on MU receptors so produces similar effects of opioids but
without addictive qualities, receptor antagonist to glutamate. Long 1⁄2 life.

849
Q

Why is nalbuphine (NUBAIN) contraindicated in previous heroin addict?

A

It’s a mixed agonist-antagonist which may

potentiate/increase withdrawal symptoms.

850
Q

Sedative drug such as hydroxyzine, meperidine and diazepam are carried in the blood as?

a. serum
b. white blood cells
c. red blood cells
d. hemoglobin

A

serum

851
Q

Where do opioids act?

A

Medulla (bind to opioid receptors in CNS)

852
Q

Where are mu receptors?

A

Medulla

853
Q

Codeines produces nausea because?
Activates vasodilator cascade
Works on the medulla

A

(stimulates medullary chemoreceptor trigger zone)

854
Q

QUESTION Mechanism of how codeine causing nausea?

A

Chemotactic receptor zone (CRZ)

855
Q

How does morphine cause emesis (vomiting) in the body?

A

via central action (medulla)

856
Q

CODEINE is

A

analgesic, antitussive (suppress coughing), anti-diarrheal, anti-hypertensive, anxiolytic, anti-depressant, sedative and
hypnotic properties. IT IS ADDICTIVE.

857
Q

If you had an allergy to codeine, what do you take for pain? Tylenol #3, hydrocodone, acetaminophen with aspirin

A

acetaminophen with aspirin
- ALLERGY TO CODEINE: can prescribe another opioid from different class: Meperidine or fentanyl for moderate to severe pain or
acetaminophen or NSAID for mild pain.

858
Q

Patient is allergic to codeine when you look at their medical history tab, (this is the trick about the exam, look up stuff before you
answer questions), what do you prescribe him for pain?
a. Hydrocodone with Acetaminophen
b. Tylenol 3
c. Hydrocodone with Aspirin
d. Acetaminophen + aspirin

A

d. Acetaminophen + aspirin

859
Q

What give to pt allergy to codeine?

A

Propoxyphene

860
Q

Allergic to Codeine, what can you give? Demerol (meperidine), Pentazocine

A

use synthetic opioids (meperidine, tramadol) à Demerol (meperidine)

861
Q
Pt has hx of codeine allergy. What drug to give?
• Tylenol #3
• Vicodin
• Naproxen
• Hydrocodone
A

Tylenol #3 has codeine

Naproxen

862
Q

Pt is allergic to aspirin, what can you give?

A

Tylenol #3 (acetaminophen & codeine)

863
Q

What does acetametaphine do with codeine

A

(Tylenol #3)? Increase its activity & increase how long it’s around due to clearance

864
Q

Why opioid analgesic containing both acetaminophen and hydrocodone so effective?

A

acetaminophen & hydrocodone works
differently, and combining these effects makes it stronger
- acetaminophen blocks the binding of protein w/ hydrocodone, so hydrocodone level higher in blood à stronger response
- Narcotics work in brain (CNS) while NSAIDS/acetaminophen work in peripheral tissues (PNS) – 2 diff mechanisms complement each other
for effective pain reduction

865
Q
Pt has mild pain from ortho tx. What med NOT to give?
• Aspirin
• Ibuprofen
• Hydrocodone
• Naproxen
A

• Hydrocodone

866
Q

What med to give for moderate post-op extraction pain? Ibuprofen, Acetaminophen, NSAID, Opioid

A

Opioid

867
Q

QUESTION: Breastfeeding mother, don’t give her what?

A

Codeine, tetracycline, benzos
- Codeine medication may be harmful to an unborn baby, and could cause breathing problems or addiction/withdrawal symptoms in a
newborn.

868
Q

Pt taking narcotic for long term what causes:

A

headache due to increase intracranial pressure.

869
Q

DEA schedules their drugs by

A

ABUSE POTENTIAL or dependency potential (addiction)

870
Q

can dentist write prescriptions for class2 for back pain

A

no

871
Q

What is not true of drugs? -

  • Schedule II drugs can get refill without prescription.
  • Schedule 3, 4, 5 drugs can be filled over the phone.
  • Scripts must have patients name and address
  • DEA number must be on each script.
A

o Schedule II drugs cannot get a refill. A new prescription must be written!

872
Q

Oxycodone, Hydrocodone schedule

A

2

873
Q

What can be combined with Tylenol to make it a level 2?

A

oxycodone, codeine etc.

874
Q

percocet schedule

A

2: oxycodone + acetaminophen

875
Q

vicodin schedule

A

2

acetaminophen + hydrocodone

876
Q

schedule 2 combination products contain hydrocodone how much

A

less than 15 milligrams of hydrocodone per dosage unit (Vicodin®)

877
Q

schedule 3 contain ho w much codeine

A

less than 90 milligrams of codeine per dosage unit.

878
Q

propoxyphene schedule

A

4

879
Q

-zepam schedule

A

4

880
Q

what NSAID is in schdule 2 and 3 drugs

A

all acetaminophen with opioid except for one that was hydrocodone with ibuprofen (vicoprofen)

881
Q

dfference in action between aspirin and NSAIDs

A

Aspirin and NSAIDs inhibit platelet cyclooxygenase, thereby blocking the formation of thromboxane A2.

  • Aspirin irreversibly blocks cyclooxygenase & it’s actions persist for circulating lifetime of platelet.
  • NSAIDS inhibit cyclooxygenase reversibly & duration of action depends on drug dose/serum levels/half-life.
882
Q

selective NSAID

A

celebrex, targets just COX-2

all others both COX-1 and 2

883
Q

How do you treat acetaminophen overdose?

A

N-acetylcysteine

884
Q

Reversal of acetaminophen:

A

NAC, N-acethylcysteine-liposome.

885
Q

Tylenol - can cause

A

hepatotoxicity

886
Q

Pt has hepatic dysfunction; which pain medication can prescribe?

a. Oxycodone
b. naproxen
c. acetaminophen

A

naproxen

887
Q

What is relationship between Tylenol and aspirin?

A

Anti-pyretic and analgesic

- Difference: aspirin is anti-inflammatory, common: anti pyretic

888
Q

Similarity between Advil and Tylenol:

A

Anti-pyretic and analgesic

889
Q

which NSAID does not have anti-inflammatory action

A

acetaminophen

890
Q

Aspirin causes -

A

Reyes fever and adults, GI problems. If liver problems, give aspirin.

891
Q

Aspirin MOA

A

inhibits platelet aggregation

892
Q

NSAIDs – mech of action of suppressing platelets –

A

inactivate cyclooxygenase à decreased prostaglandin synthesis

893
Q

Aspirin stops pain by:
a. stopping the upward transduction of pain signal in the spinal cord
b. intefere with signal interpretation in the CNS
c. stopping the local signal production and transduction
stopping the signal transduction in the cortex

A

stopping the local signal production and transduction

894
Q

NSAID that is least likely to affect stomach –

A

CELEBREX (Selective NSAID - Cox 2 inhibitor only)

- Cox 2 does not increase bleeding time and less platelet adhesion.

895
Q

What does not have an effect on platelets on this list of NSAIDS?

A

Celebrex (Celecoxib)

896
Q

side effect of celebrex

A

bleeding!

897
Q

CASE: Patient is taking baby aspirin (81 mg).

a) How long before should you stop before surgery?
b) Is it necessary to stop?
c) For long will the platelets be inhibited?

A

b) Is it necessary to stop? No
c) For long will the platelets be inhibited? 5-7days
- aspirin stays in body for 7 days.

898
Q

For how long will a single dose of aspirin will have effect on the platelets? 2h, 12h, 1 day, 10 days, 1 month

A

10 days

899
Q

After one effective dose of aspirin, how long must you wait before there is not effect on bleeding time?

A

1 week

900
Q

Aspirin works on which pathway for pain?

A

Cyclo-ox pathway

901
Q

How does aspirin work to inhibit bleeding?

A

Inhibits thromboxane A2, preventing platelet synthesis

- aspirin affects bleeding time

902
Q

Allergic to Aspirin?

A

Take acetaminophen. DO NOT take ibuprofen.

903
Q

If someone can’t take Ibuprofen, what can you give them?

a. aspirin
b. Demerol
c. Pentazocine

A

b. Demerol - narcotic w/out aspirin

pentazocine is with aspirin

904
Q

Which statement is correct for Ibuprofen?
• ceiling analgesia at 400mg
• safe use for pt w/ peptic ulcer

A

• ceiling analgesia at 400mg

905
Q

Asthma patient: NSAID

A

contraindication

906
Q

Patient is on 3-5g acetylsalicylic acid per day for 3 months, what are you the most likely to see in this patient?
Increased PT and Bleeding time
Increased PT and PTT
Acidosis and increased bleeding time

A

Acidosis and increased bleeding time

- Causes acidosis since acetylsalicylic acid is aspirin. It’s an acid & 3g daily is a lot!

907
Q

Ketorolac is an

A

NSAID that inhibits prostaglandin synthesis (competitive non-selective cox inhibit).

908
Q

What would you prefer for a patient with renal vascular disease & why?

a. acetaminophen
b. aspirin
c. ketorolac
d. ibuprofen

A

acetaminophen
- The other drugs are NSAIDS and they affect the kidney in a more negative way. This drug affects the liver and causes liver toxicity.

909
Q

QUESTION: Methotrexate (immune suppressive drug) toxicity

A

increases with use of NSAIDS or penicillin

910
Q

3rd trimester pregnant patient needs pain medication – options: Tylenol 325mg, Tylenol 3m aspirin, or ibuprofen 600mg?

A

Tylenol 325mg

911
Q
Patient is pregnant and needs 2 teeth extracted. Pt needs post-op analgesic. What do you rx?
ibuprofen
aspirin
Tylenol 3
Tylenol (acetaminophen)
A
  • For pregnant- only give Tylenol- NOTHING WITH CODEINE
912
Q

What is a safe pain killer to give a woman who is pregnant?

A

Tylenol

913
Q

What analgesic do you give a child with Asthma?

A

Tylenol

- Asthmatic only use Tylenol (not aspirin bc of hypervent)

914
Q

5-year-old patient with fever and pain. What do you Rx?

  • Codeine
  • Tylenol
  • Aspirin
  • NSAID
A

Tylenol

915
Q

If a patient had some teeth extracted & asked what drug he can take that’ll provide at least 8 hours of relief, what would you Rx?

a. Tylenol
b. Ibuprofen
c. Naproxen

A

Naproxen

916
Q

Pt wants to be able to sleep through the night following extraction what should you prescribe?

A

Naproxen — a nonselective COX

inhibitor—NSAID

917
Q

Naproxen duration of action

A

8 hours

918
Q

What are you worried about when a patient is on Naproxen?

A

DDI w/ aspirin antiplatelet activity

919
Q
Which of the following has least effect on platelets/bleeding?
• Aspirin
• Ibuprofen
• Naproxen
• Difluzole
A

• Difluzole (vaginal candidiasis medication)

920
Q

Epinephrine = _____ to histamine & nitroglycerin

A

physiological antagonist of

  • Doesn’t act on same mechanism (epi = α vasoconstriction vs nitro = smooth muscle dilatator) but opposing action
  • Same mechanism = competive antagonist; physiological antagonist = competing physicological effects
921
Q

What best describes biotransformation? I

A

Increase in polarity, more ionized and more water soluble

  • Whatever helps its excretion – polar and more water soluble
922
Q

In relation to their parent drug, conjugated metabolites are what?

A

more ionized in plasma (more water soluble)

923
Q

QUESTION: What happens to a drug after conjugation? more ionic, more hydrophilic, more active

A

more ionic

924
Q

What do you use sodium bicarbonate for?

A

All drugs or alcohol (Phenobarbitals)

- Excretion of acidic drugs is accelerated with sodium bicarbonate

925
Q

Excretion of an acidic drug will be enhanced if the patient is given which of the following?

A

Sodium bicarbonate

926
Q

After drug goes through liver?

A

More water soluble and less lipid soluble.

927
Q

First pass metabolism:

A

enzymatic degradation in the liver prior to drug reaching its site of action

928
Q

First pass refers to:

A

enterohepatic circulation, metabolism in liver